Tag Archives: Lop8

Bài toán hàm số trong kì thi tuyển sinh vào 10

Trong các kì thi tuyển sinh vào 10 có dạng toán liên quan đến hàm số, chủ yếu là hàm bậc hai dạng $y = ax^2$ (1) và đường thẳng $y = mx + n$ (2)Trong bài viết này chủ yếu xét các bài toán tương giao giữa đồ thị hàm số (1) và (2).

Nếu hàm số $y =ax^2$ có đồ thị là parabol $(P)$ và hàm số $y = mx + n$ có đồ thị là đường thẳng $d$, thì phương trình hoành độ giao điểm của $(P)$ và $(d)$ là

$$ax^2 = mx + n \Leftrightarrow ax^2 – m x – n =0 (*)$$

$(*)$ là một phương trình bậc hai, nên có 3 trường hợp xảy ra:

  • TH1: Nếu $(*)$ vô nghiệm thì $(d)$ và $(P)$ không có giao điểm.
  • TH2: Nếu $(*)$ có 1 nghiệm thì $(d)$ và $(P)$ có 1 giao điểm, ta nói $d$ tiếp xúc với $(P)$.
  • TH3: Nếu $(*)$ có hai nghiệm phân biệt thì ta nói $(d)$ cắt $(P)$, và nghiệm của $(*)$ là hoành độ của hai giao điểm, từ hoành độ ta có thể tính tung độ của giao điểm dựa vào phương trình của $(d)$ hoặc của $(P)$.

Ta xét một vài ví dụ sau:

Bài 1. (Thi vào lớp 10 trường PTNK năm 2018) Gọi $(P),(d)$ lần lượt là đồ thị của các hàm số $y=x^2$ và $y=2 m x+3$.
a) Chứng minh đường thẳng $(d)$ luôn cắt $(P)$ tại hai điểm phân biệt $A\left(x_1 ; y_1\right), B\left(x_2 ; y_2\right)$ và tính $y_1+y_2$ theo $m$.
b) Tìm $m$ sao cho $y_1-4 y_2=x_1-4 x_2+3 x_1 x_2$.

Lời giải bài 1.
a) Phương trình hoành độ giao điểm của $(P)$ và $(d)$ là:
$$
x^2=2 m x+3 \Leftrightarrow x^2-2 m x-3=0 \quad(1)
$$

Xét phương trình (1), ta có: $\Delta^{\prime}=m^2+3>0$ với mọi $m \in \mathbb{R}$
Suy ra phương trình (1) luôn có hai nghiệm phân biệt $x_1, x_2$ với mọi $m$ hay $(d)$ luôn cắt $(P)$ tại hai điểm phân biệt $A\left(x_1 ; y_1\right), B\left(x_2 ; y_2\right)$.
Theo định lý Viete, ta có: $\left\{\begin{array}{l}x_1+x_2=2 m \\\ x_1 x_2=-3\end{array}\right.$
Khi đó $y_1=2 m x_1+3, y_2=2 m x_2+3$
$y_1+y_2=2 m x_1+3+2 m x_2+3=2 m\left(x_1+x_2\right)+6=4 m^2+6$
b) Ta có:
$y_1-4 y_2=x_1-4 x_2+3 x_1 x_2 $
$\Leftrightarrow 2 m x_1+3-8 m x_2-12=x_1-4 x_2-9 $
$ \Leftrightarrow 2 m\left(x_1-4 x_2\right)=x_1-4 x_2 $
$ \Leftrightarrow\left(x_1-4 x_2\right)(2 m-1)=0 $
$ \Leftrightarrow\left[\begin{array}{l}
x_1=4 x_2 \\\
m=\frac{1}{2} \quad(n)
\end{array}\right. $
Với $x_1=4 x_2 $ lại có $x_1 x_2=-3 \Rightarrow 4 x_2^2=-3 $ (vô lý)
Vậy $m=\frac{1}{2} $

Bài 2. (Đề thi vào 10 trường PTNK năm 2019) Cho $(P),(d)$ lần lượt là đồ thị hàm số $y=x^2$ và $y=2 x+m$.
a) Tìm $m$ sao cho $(P)$ cắt $(d)$ tại hai điểm phân biệt $A\left(x_1 ; y_1\right), B\left(x_2 ; y_2\right)$.
b) Tìm $m$ sao cho $\left(x_1-x_2\right)^2+\left(y_1-y_2\right)^2=5$.

Lời giải bài 2.
a) Phương trình hoành độ giao điểm của $(P)$ và $(d)$
$$
x^2=2 x+m \Leftrightarrow x^2-2 x-m=0 \quad(1)
$$
$(P)$ cắt $(d)$ tại 2 điểm phân biệt $A, B \Leftrightarrow (1)$ có 2 nghiệm phân biệt

$\Leftrightarrow \Delta^{\prime}>0 \Leftrightarrow 1+m>0 $
$ \Leftrightarrow m>-1(*)$
Vậy $m>-1$ thì $(P)$ cắt $(d)$ tại hai điểm phân biệt.


b) Với điều kiện $(*)$ theo Viete ta có: $S=x_1+x_2=2, P=x_1 \cdot x_2=-m$

Ta có: $A\left(x_1 ; y_1\right) \in(d) \Leftrightarrow y_1=2 x_1+m ; B\left(x_2 ; y_2\right) \in(d) \Leftrightarrow y_2=2 x_2+m$

Ta có: $\left(x_1-x_2\right)^2+\left(y_1-y_2\right)^2=5 $

$\Leftrightarrow\left(x_1-x_2\right)^2+\left(2 x_1-2 x_2\right)^2=5$

$\Leftrightarrow\left(x_1-x_2\right)^2+4\left(x_1-x_2\right)^2=5 $

$\Leftrightarrow\left(x_1-x_2\right)^2=1 \Leftrightarrow\left(x_1+x_2\right)^2-4 x_1 x_2=1$

$\Leftrightarrow 4+4 m=1 \Leftrightarrow m=\frac{-3}{4} $ thỏa (*)
Vậy $m = \dfrac{-3}{4}$.

Bài 3. Đồ thị của hàm số $f(x)=a x^2$ và $g(x)=-a x+b(a ; b$ là các số thực), điểm chung thứ nhất có hoành độ bằng 1 và tung độ điểm chung thứ 2 là 8 . Tìm hoành độ của điểm chung thứ hai của hai đồ thị và tính $a, b$.

Lời giải bài 3.

  • Phương trình hoành độ giao điểm $a x^2=-a x+b \Leftrightarrow a x^2+a x-b=0$ thì phương trình nhận 1 là nghiệm nên $a 1^2+a \cdot 1-b=0 \Rightarrow b=2 a$.
  • Khi đó gọi nghiệm còn lại là $x_2$ ta có $1 \cdot x_2=\frac{-b}{a}=-2$
  • Do đó tung độ $a(-2)^2=8$, suy ra $a=2$ và $b=4$.

Bài 4. (TS chuyên Đăk Lăk 2020 – 2021) Trong mặt phẳng $O x y$, cho parabol $(P): y=x^2$ và đường thẳng $(d): y=2(m+1) x+3$ với $m$ là tham số. Tìm tất cả các giá trị của tham số $m$ để đường thẳng $(d)$ cắt parabol tại hai điểm phân biệt có hoành độ $x_1, x_2$ thoả mãn điều kiện $x_1^2-2 m x_1+2 x_2-x_1 x_2=2$.

Lời giải bài 4.

  • Phương trình hoành độ giao điểm $x^2-2(m+1) x-3=0\left(^*\right)$ $\Delta^{\prime}=(m+1)^2+3>0$ với mọi $m$.
  • Theo định lý Viete ta có $x_1+x_2=2(m+1), x_1 x_2=-3$.
    Ta có $x_1^2-2(m+1) x_1-3=0$, suy ra $x_1^2-2 m x_1=2 x_1+3$ $x_1^2-2 m x_1+2 x_2-x_1 x_2=2 \Leftrightarrow 2 x_1+3+2 x_2-(-3)=2 \Leftrightarrow m=-2$.
  • Vậy $m=-2$.

Bài 5. (TS chuyên Khánh Hoà 2020 – 2021) Trên mặt phẳng toạ độ $O x y$, cho parabol $(P)$ có phương trình $y=2 x^2$ và đường thẳng $(d): y=-2 m x+m+1$ với $m$ là tham số.
a) Chứng minh đường thẳng $(d)$ luôn cắt Parabol $(P)$ tại hai điểm phân biệt.
b) Gọi $x_1, x_2$ lần lượt là hoành độ giao điểm của đường thẳng $(d)$ và parabol $(P)$, tìm $m$ thoả mãn đẳng thức $\frac{1}{\left(2 x_1-1\right)^2}+\frac{1}{\left(2 x_2-1\right)^2}=66$.

Lời giải bài 5 .
a) Phương trình hoành độ giao điểm của $d$ và $P$ là
$$
2 x^2+2 m x-m-1=0
$$
$\Delta^{\prime}=m^2-2(-m-1)=(m+1)^2+1>0$ với mọi $m$, do đó $d$ cắt $P$ tại hai điểm phân biệt với mọi $m$.
b) Theo định lý Viete ta có $x_1+x_2=-m, x_1 x_2=\frac{-m-1}{2}$.
Suy ra $x_1^2+x_2^2=\left(x_1+x_2\right)^2-2 x_1 x_2=m^2+m+1$
Ta có $66=\frac{1}{\left(2 x_1-1\right)^2}+\frac{1}{\left(2 x_2-1\right)^2}=\frac{\left(2 x_1-1\right)^2+\left(2 x_2-1\right)^2}{\left(2 x_1-1\right)^2\left(2 x_2-1\right)^2}=\frac{4\left(x_1^2+x_2^2\right)-4\left(x_1+x_2\right)+2}{\left(4 x_1 x_2-2\left(x_1+x_2\right)+1\right)^2}$
$$
=\frac{4\left(m^2+m+1\right)-4(-m)+2}{(-2 m-2-2(-m)+1)^2}=\frac{4 m^2+8 m+6}{1}
$$

Giải ra được $m=-5, m=3$.

Bài 6. (TS chuyên Thái Bình 2020 – 2021) Trong mặt phẳng toạ độ $O x y$, cho parabol $(P): y=\frac{x^2}{2}$ và hai đường thẳng $\left(d_1\right): y=5 x+2,\left(d_2\right): y=\left(m^2+1\right) x+m$ với $m$ là tham số.
a) Tìm $m$ để $\left(d_1\right)$ song song với $\left(d_2\right)$.
b) Tìm $m$ để $\left(d_2\right)$ cắt parabol $(P)$ tại hai điểm phân biệt có hoành độ $x_1, x_2$ sao cho $Q=x_1+x_2-4 x_1 x_2$ đạt giá trị nhỏ nhất.

Lời giải bài 6 .
a) Điều kiện để $d_1 || d_2$ là $m^2+1=5, m \neq 2$, giải ra được $m=-2$.
b) Phương trình hoành độ giao điểm của $d_2$ và $P$ là
$$
\frac{x^2}{2}=\left(m^2+1\right) x+m \Leftrightarrow x^2-2\left(m^2+1\right) x-2 m=0
$$

Điều kiện $\Delta^{\prime}=\left(m^2+1\right)^2-(-2 m)>0 \Leftrightarrow m^4+2 m^2+1+2 m>0 \Leftrightarrow m^4+m^2+(m+1)^2>0$ (Đúng với mọi $m)$

Theo định lý Viete ta có $x_1+x_2=2\left(m^2+1\right), x_1 x_2=-2 m$

Ta có $P=x_1+x_2-4 x_1 x_2=$ $2\left(m^2+1\right)-4(-2 m)=2\left(m^2+1+4 m\right)=2(m+2)^2-6 \geq-6$, đẳng thức xảy ra khi $m=-2$.

Bài 8. Trong mặt phẳng tọa độ $O x y$, cho parabol $(P): y=x^2$ và đường thẳng $(d): y=2 x-m-2$. Tìm tất cả các giá trị của tham số $m$ để $(d)$ cắt $(P)$ tại hai điểm phân biệt lần lượt có hoành độ $x_1, x_2$ thỏa mãn $x_1^2+1=2 x_2$.

Lời giải bài 8 .

  • Phương trình hoành độ giao điểm
    $$
    x^2=2 x-m-2 \Leftrightarrow x^2-2 x+m+2=0
    $$
  • Điều kiện $\Delta^{\prime}=1-(m+2)>0 \Leftrightarrow m<-1$.
  • Theo định lý Viete ta có $x_1+x_2=2, x_1 x_2=m+2$.

Ta có $x_1^2=2 x_1-m-2$, suy ra $x_1^2+1=2 x_2 \Leftrightarrow 2 x_1-m-2+1=2 x^2 \Leftrightarrow 2\left(x_1-x_2\right)=m+1$ Kết hợp với Viete ta có $x_1=\frac{m+5}{4}, x_2=\frac{3-m}{4}$
Khi đó $x_1 x_2=m+2 \Leftrightarrow \frac{m+5}{4} \frac{3-m}{4}=m+2 \Leftrightarrow m=-1(l), m=-17(n)$.

  • Vậy $m=-17$.

Bài 9. Cho $(P): y=x^2$ và đường thẳng $(d): y=(m+2) x-2 m$.
a) Tìm $m$ để $d$ cắt $(P)$ tại hai điểm phân biệt $A\left(x_1 ; y_1\right), B\left(x_2 ; y_2\right)$.
b) Tìm $m$ để $x_1+2 y_2=7$.

Lời giải bài 9 .
a) Phương trình hoành độ giao điểm

$\quad x^2-(m+2) x+2 m=0 $
$\Delta=(m+2)^2-8 m=(m-2)^2>0 \Leftrightarrow m \neq 2 .$

b) Khi đó phương trình có nghiệm $x=2, x=m$.
3

  • TH1: $x_1=2, x_2=m$ suy ra $y_1=4, y_2=m^2$. Ta có $2+2 m^2=7$ giải ra được $m=\sqrt{2,5}, m=$ $-\sqrt{2,5}$.
  • TH2: $x_1=m, x_2=2$, suy ra $y_1=m^2, y_2=4$. Ta có $m+2.4=7 \Leftrightarrow m=-1$.
  • Vậy có 3 giá trị $m$ thỏa đề bài $m=\sqrt{2,5}, m=-\sqrt{2,5}, m=-1$.

Bài 10. Trong mặt phẳng tọa độ $O x y$, cho parabol $(P)$ có phương trình $y=x^2$ và đường thẳng $(d)$ có phương trình $y=2 m x-m^2-m-2$ (với $m$ là tham số).
a) Tìm tọa độ điểm $M$ thuộc $(P)$ biết điểm $M$ có hoành độ bằng -3 .
b) Tìm điều kiện của $m$ để đường thẳng $(d)$ cắt parabol $(P)$ tại hai điểm phân biệt. Gọi $A\left(x_1 ; y_1\right), B\left(x_2 ; y_2\right)$ là hai giao điểm của đường thẳng $(d)$ và parabol $(P)$, xác định $m$ để $x_1 y_2+x_2 y_1=2 m^3+6$.

Lời giải bài 10.

b) Tìm điều kiện của $m$ để đường thẳng $(d)$ cắt parabol $(P)$ tại hai đie biệt. Gọi $A\left(x_1 ; y_1\right), B\left(x_2 ; y_2\right)$ là hai giao điểm của đường thẳng $(d)$ và $(P)$, xác định $m$ để $x_1 y_2+x_2 y_1=2 m^3+6$. Ta có phương trình hoành độ giao điểm của $(d)$ và $(P)$ là

$ x^2=2 m x-m^2-m-2 \Leftrightarrow x^2-2 m x+m^2+m+2=0(1) $
$ \Delta^{\prime}=(-m)^2-\left(m^2+m+2\right)=-m-2$

$(d)$ cắt parabol $(P)$ tại 2 điểm phân biệt khi và chỉ khi phương trình (1) có hai nghiệm phân biệt $\Leftrightarrow \Delta^{\prime}>$ $0 \Leftrightarrow-m-2>0 \Leftrightarrow m<-2(*)$

$ \text { Ta có } x_1+x_2=2 m, x_1 x_2=m^2+m+2 $
$x_1 y_2+x_2 y_1=x_1 \cdot x_2^2+x_2 \cdot x_1^2=x_1 \cdot x_2\left(x_1+x_2\right)=2$ $m\left(m^2+m+2\right) $
$=2 m^3+2 m^2+4 m $
$2 m^3+2 m^2+4 m=2 m^3+6 \Leftrightarrow 2 m^2+4 m-6=0 $

$\Leftrightarrow\left[\begin{array}{l}
m=1 \\\
m=-3
\end{array}\right.$

Đối chiếu (*) vậy $m=-3$.

Cực trị hình học (Lớp 8)

Cực trị hình học là bài toán tìm giá trị lớn nhất, giá trị nhỏ nhất của các đối tượng hình học như độ dài, chu vi, diện tích, …

Các bước cho một bài toán tìm giá trị lớn nhất, giá trị nhỏ nhất gồm các bước sau:

  • Đánh giá bất đẳng thức
  • Tìm điều kiện, vị trí để đẳng thức xảy ra
  • Kết luận

Một số tính chất cần nhớ trong các bài toán cực trị

Tính chất 1. Bất đẳng thức trong tam giác: Cho 3 điểm $A, B, C$ thì $AB + BC \geq AC$. Đẳng thức xảy ra khi $B$ nằm giữa $A, C$. Tính chất này có thể tổng quát cho trường hợp nhiều hơn 3 điểm.

Từ tính chất này ta có thể thấy rằng, con đường ngắn nhất để đi từ $A$ đến $B$ là con đường thẳng.

Tính chất 2. Đường xiên và hình chiếu: Cho điểm $A$ và đường thẳng $d$, khi đó $M$ thay đổi trên $d$ thì $AM$ nhỏ nhất khi và chỉ khi $M$ là hình chiếu vuông góc của $A$ trên $d$.

Một số bất đẳng thức cần dùng: Cho $a, b \geq 0$.

  • $a^2 + b^2 \geq \dfrac{1}{2} (a+b)^2 \geq 2ab$
  • $\dfrac{1}{a} + \dfrac{1}{b} \geq \dfrac{4}{a+b}$
  • $a^2+b^2+c^2 \geq \dfrac{1}{3}(a+b+c)^2 \geq ab+bc+ac$.

Chú ý trong các bài toán cực trị thì đẳng thức phải xảy ra, do đó việc đánh giá bất đẳng thức cần chặt chẽ để xảy ra dấu bằng, nếu tìm không được vị trí dấu bằng xảy ra thì đánh giá đó chưa hợp lý.

Kinh nghiệm làm bài, nếu bài toán có nhiều giá trị thay đổi ta có thể

  • Tính toán biến đổi để đưa về biểu thức ít yếu tố thay đổi hơn.
  • Tìm mối liên hệ giữa các biến mà không đổi như: tích không đổi, tổng không đổi,… và từ đó áp dụng các bất đẳng thức đại số để đánh giá.

Sau đây là một số ví dụ.

Ví dụ 1. Cho hai điểm $A, B$ và đường thẳng $d$. Tìm vị trí của $M$ thuộc $d$ sao cho $MA + MB$ nhỏ nhất trong hai trường hợp.

a) $A, B$ cùng phía với $d$.

b) $A,B$ khác phía đối với đường thẳng $d$.

Phân tích và Lời giải

a) Với bài toán này ta nhận thấy rằng ta có thể áp dụng ngay tính chất 1, ta có $MA + MB \geq AB$

Đẳng thức xảy ra khi $M$ là điểm nằm giữa $A,B$, mà $M$ thuộc $d$ nên $M$ là giao điểm của đoạn thẳng $AB$ và đường thẳng $d$. Rõ ràng giao điểm này tồn tại vì $A, B$ là khác phía đối với $d$.

b) Đối với ý này, nếu vội vàng áp dụng như câu a thì ta thấy do $A,B$ cùng phía nên giao điểm của đoạn thẳng $AB$ và $d$ không tồn tại. Do đó cách làm như câu a, cũng không đúng.

Vậy ta sẽ làm thế nào? Ta có thể đưa về trường hợp ở câu a hay không? nếu đưa về câu a thì ta sẽ làm gì?

Ở đây có một kĩ thuật, là sử dụng đối xứng trục, để thay đổi vị trí điểm $A$ và vẫn tạo ra một đoạn thẳng bằng với $MA$. Tạo ra điểm phụ sẽ giúp ta giải được bài toán này.

Gọi $A’$ là đối xứng của $A$ qua $d$, khi đó $A’, B$ khác phía đối với $d$ và $MA = MA’$, ta đưa về trường hợp của câu $a$.

Ta có $MA + MB = MA’ + MB \geq A’B$, đẳng thức xảy ra khi $M$ là giao điểm của $A’B$ và $d$.

Vậy $MA + MB$ nhỏ nhất khi $M$ là giao điểm của $A’B$ với $d$.

Ví dụ 2. Cho tam giác $ABC$ đều cạnh $a$. $M$ là một điểm nằm trong tam giác. Gọi $D, E, F$ là hình chiếu của $M$ trên các cạnh $BC, AC, AB$.

a) Chứng minh $MD + ME + MF$ không đổi và tìm giá trị nhỏ nhất của $MD^2 +ME^2 +MF^2$.

b) Tìm giá trị nhỏ nhất của $BD^2+CE^2+AF^2$.

Phân tích và lời giải

a) Với bài này việc chứng minh ý đầu có thể còn khó hơn ý sau, việc chứng minh tổng này không đổi thì nhiều khi ta phải dự đoán được tổng này giá trị không đổi là bao nhiêu, phụ thuộc vào $a$ thế nào. Ta có thể đoán bằng cách cho $M$ trùng với một đỉnh nào đó, hoặc điểm đặc biệt như tâm của tam giác đều, khi cho trùng đỉnh $A$ thì ta có $E, F \equiv A$, $D \equiv H$, chân đường cao từ $A$, do đó ta có $MD + ME +MF = AH$ độ dài đường cao. Việc chứng minh tổng này bằng $AH$ ta có thể sử dụng phương pháp diện tích, rất hữu hiệu trong các bài có độ dài đường vuông góc.

a) $S_{A B C}=S_{M B C}+S_{M A C}+S_{M A B}$
$$
\begin{aligned}
\frac{1}{2} A H \cdot B C= & \frac{1}{2} M D \cdot B C+\frac{1}{2} M E \cdot A C \
& +\frac{1}{2} M F \cdot A C
\end{aligned}
$$
$\frac{1}{2} A H \cdot a=\frac{1}{2} a(M D+M E+M E)$
$$
\Rightarrow M D+M E+M F=A I+\text { (Ehongdon!) }
$$
$$
=\frac{a \sqrt{3}}{2}
$$
Áp dụng bất đẳng thức $x^2+y^2+z^2 \geqslant \frac{1}{3}(x+y+z)^2$
$$
\begin{array}{ll}
\Rightarrow M D^2+M E^2+M F^2 \geqslant \frac{1}{3}(M D+M E+M F)^2 & =\frac{1}{2} \cdot\left(\frac{a \sqrt{3}}{2}\right)^2 \
& =\frac{a^2}{4}
\end{array}
$$
Đẳng thức xảy ra khi và chỉ khi $M D=M E=M F$, tức là $M$ là giao điểm 3 đường phân giác của tam giác $ABC$, do tam giác $ABC$ đều nên $M$ cũng là trọng tâm tam giác.
$$
(M D^2+M E^2+M F^2)_{\min }=\dfrac{a^2}{4}
$$

b) Với câu này mình không có gợi ý như câu b, tìm min của đại lượng $P = BD^2+CE^2+AF^2$ ta có thể suy nghĩ tới việc tính tổng hay tích các số hạng, tuy vậy các giá trị này thay đổi theo $M$. Và quan sát thêm một chút là vị trí của các đoạn thẳng $DB, CE, AF$ trên các cạnh $BC, AC, AB$ có vẻ là cùng một hướng, và ta lại xem các đoạn thẳng còn lại thế nào? tức là $CD, BF, AE$ vai trò như nhau với các đoạn trên không? Liệu $BD^2+CE^2+AF^2 = CD^2+BF^2+AE^2?

Và khi đi vào kiểm tra thì rõ ràng ta chứng minh được $BD^2+CE^2+AF^2 = CD^2 + BF^2+AE^2 (1)$ và từ đó ta có lời giải như sau.

Trước hết ta chứng minh (1), theo định lý Pitago ta có $BD^2 – CD^2 = MB^2-MD^2 – (MC^2-MD^2) = MB^2-MC^2$, tương tự ta cũng có các đẳng thức khác.

Khi đó $BD^2+CE^2+AF^2 – CD^2-AE^2-BF^2 = MB^2 – MC^2 + MC^2-MA^2 +MA^2-MB^2 = 0$

Suy ra $BD^2+CE^2+AF^2 = CD^2+BF^2+AE^2 = \dfrac{1}{2} (BD^2+CD^2+AF^2+BF^2+CE^2+AE^2$.

Mà $CD^2+BD^2 \geq \dfrac{1}{2}(CD+BD)^2 = \dfrac{1}{2}a^2$

Tương tự thì $AF^2+BF^2 \geq \dfrac{1}{2}a^2, AE^2+CE^2 \geq \dfrac{3}{2}a^2$

Từ đó $BD^2+CE^2+AD^2 \geq \dfrac{3}{4}a^2$, đẳng thức xảy ra khi $M$ là giao điểm 3 đường trung trực của tam giác $ABC$.

Vậy $(BD^2+CE^2+AF^2)_{\max} = \dfrac{3}{4}a^2$.

Ví dụ 3. (PTNK 1999) Cho tam giác $A B C$ có diện tích $\mathrm{S}$ và một điểm $P$ nằm trong tam giác.
(a) Gọi $S_1, S_2, S_3$ lần lượt là diện tích của tam giác $P B C, P C A, P A B$. Hãy tìm giá trị nhỏ nhất của $S_1^2+S_2^2+S_3^2$.
(b) Gọi $P_1, P_2, P_3$ lần lượt là các điểm đối xứng của $P$ qua $B C, C A$ và $A B$. Đường thẳng qua $P_1$ song song với $B C$ cắt $A B$ và $A C$ tại $B_1$ và $C_1$. Đường thẳng qua $P_2$ song song với $A C$ cắt $B C, B A$ tại $C_2, A_2$, đường thẳng qua $P_3$ và song song với $A B$ cắt $C A, C B$ tại $A_3, B_3$. Hãy xác định vị trí của điểm $P$ dể tổng diện tích ba hình thang $B C C_1 B_1, C A A_2 C_2$ và $A B B_3 A_3$ đạt giá trị nhỏ nhất và tính giá trị đó.

Phân tích và lời giải

a) Bài này ta làm tương tự câu a ví dụ 2, cũng áp dụng bdt $x^2+y^2+z^2 \geq \dfrac{1}{3} (x+y+z)^2$ để suy ra cực trị.

b) Với bài toán này, để tìm cực trị của tổng diện tích các hình thang, ta phải tính diện tích các hình thang này thông qua một đại lượng trung gian, trong bài này thì đó là diện tích tam giác $ABC$, (S). Việc các đường thẳng song song gợi ta nghĩa tới tam giác đồng dạng và tính chất “tỉ số diện tích bằng bình phương tỉ số đồng dạng”, từ đó ta có cách giải sau:

b) Gọi độ dài các đường cao của tam giác $A B C$ là $h_a, h_b, h_c$ và khoảng cách từ $P$ đến $B C, A C, A B$ là $x, y, z$. Ta có $\frac{S}{S_{A B_1 C_1}}=\frac{h_a^2}{\left(h_a+x\right)^2}$.
Suy ra $S_{A B_1 C_1}=\left(1+\frac{x}{h_a}\right)^2 S$.
Tương tự ta có $S_{B A_2 C_2}=\left(1+\frac{y}{h_b}\right)^2 . S, S_{C A_3 B_3}=\left(1+\frac{z}{h_c}\right)^2 S$.
Đặt $a=\frac{x}{h_a}, b=\frac{y}{h_b}, c=\frac{z}{h_c}$ thì $a+b+c=1$.
Ta có $S_{B C C_1 B_1}+S_{A C C_2 A_2}+S_{A B B_3 A_3}=S\left((1+a)^2+(1+b)^2+(1+c)^2-3\right)=$ $S\left(2+a^2+b^2+c^2\right)$.
Ta có $a^2+b^2+c^2 \geq \frac{1}{3}(a+b+c)^2=\frac{1}{3}$. Do đó $S_{B C C_1 B_1}+S_{A C C_2 A_2}+S_{A B B_3 A_3} \geq \frac{7}{3} S$.
Đẳng thức xảy ra khi $P$ là trọng tâm tam giác $A B C$.

Ví dụ 4. (PTNK 2008) Cho góc $x A y$ vuông và hai điểm $B, C$ lần lượt trên các tia $A y, A y$. Hình vuông $M N P Q$ có các đỉnh $M$ thuộc cạnh $A B$, dỉnh $N$ thuộc cạnh $A C$ và các đỉnh $P, Q$ thuộc cạnh $B C$.
(a) Tính cạnh hình vuông $M N P Q$ theo cạnh $B C=a$ và đường cao $A H=h$ của tam giác $A B C$.
(b) Cho $B, C$ thay đổi lần lượt trên các tia $A x, A y$ sao cho tích $A B \cdot A C=k^2$ ( $k$ không đổi). Tìm giá trị lớn nhất của diện tích hình vuông $M N P Q$.

Phân tích và lời giải

a)

a) Đặt $x$ là độ dài hình vuông. Gọi $K$ là giao điểm của $A H$ và $M N$.
Ta có $M K H Q$ là hình chữ nhật, suy ra $K H=M Q=x, A E=A H-E H=$ $h-x$.
Ta có $M N \parallel B C$, suy ra $\frac{M N}{B C}=\frac{A N}{A C}$.
Và $N K \parallel C H$ nên ta có $\frac{A N}{A C}=\frac{A K}{A H}$.
Do đó ta có $\frac{M N}{B C}=\frac{A K}{A H}$ hay $\frac{x}{a}=\frac{h-x}{h}$, suy ra $x=\frac{a h}{a+h}$.
b) Ta có $b c=a h=k^2$ và $a^2=b^2+c^2 \geq 2 b c=2 a h$. Suy ra $a \geq 2 h$.
Ta có $S_{M N P Q}=M N^2=\frac{(a h)^2}{(a+h)^2}=\frac{k^4}{(a+h)^2}$.
Ta có $(a+h)^2=a^2+h^2+2 a h=h^2+\frac{1}{4} a^2+\frac{3}{4} a^2+2 a h$.
Mà $h^2+\frac{1}{4} a^2 \geq a h=k^2, \frac{3}{4} a^2 \geq \frac{3}{2} a h=\frac{3}{2} k^2, a h=k^2$.
Suy ra $(a+h)^2 \geq \frac{9}{2} k^2$.
Do đó $S \leq \frac{2}{9} k^2$. Đẳng thức xảy ra khi $a=2 h$ hay tam giác $A B C$ cân.
Vậy giá trị lớn nhất của diện tích hình vuông MNPQ là $\frac{2}{9} k^2$ khi $A B=A C=k$.

Chú ý, nếu ta áp dụng Cauchy ngay chỗ $(a+h)^2 \geq 4ah$ thì đẳng thức không xảy ra, do đó đánh giá chưa đủ chặt chẽ.

Bài tập rèn luyện.

Bài 1. Cho tam giác $ABC$ nhọn tìm vị trí điểm $M$ trong tam giác sao cho $MA + MB + MC$ nhỏ nhất.

Bài 2. Cho hình vuông $A B C D . M, N, P, Q$ là các đỉnh của tứ giác $M N P Q$ lần Iượt thuộc các cạnh $\mathrm{AB}, \mathrm{BC}, \mathrm{CD}, \mathrm{DA}$ (MNPQ gọi là tứ giác nội tiếp hình vuông). Tìm điều kiện để tứ giác MNPQ có chu vi nhỏ nhất.

Bài 3. Cho tam giác $ABC$ nhọn. Tìm vị trí của $M$ bên trong tam giác sao cho $MA \cdot BC + MB \cdot AC + MC \cdot AB$ đạt giá trị nhỏ nhất.

Bài 4. Cho tam giác $ABC$ vuông tại $A$ có $BC$ không đổi $BC = 2a$. Vẽ đường cao $AH$. Tìm giá trị lớn nhất của $BH + AH$.

Bài 5. Cho hình bình hành $ABCD$, một đường thẳng $d$ qua $A$ không cắt các cạnh của hình bình hành. Tìm vị trí của $d$ để tổng khoảng cách từ các đỉnh $B, C, D$ đến $d$ là lớn nhất.

Bài 6. Cho đoạn thẳng $A B=a$. $C$ là điểm trên đoạn thẳng $A B$. Vẽ các hình vuông $A C D E$ và $C B F G$. Xác định vị trí điểm $C$ để $S_{A C D E}+S_{C B F G}$ đạt giá trị nhỏ nhất.

CHUYÊN ĐỀ SỐ HỌC : PHÉP CHIA HẾT VÀ PHÉP CHIA CÓ DƯ

MỘT SỐ VÍ DỤ

 

Ví dụ 1

Cho $a$ là một số nguyên. Tìm UCLN $(2 a+3,3 a+4)$.

Lời giải

Gọi $d=(2 a+3,3 a+4)$, ta có $d \backslash 2 a+3$ và $d \backslash 3 a+4$.

Vì $3(2 a+3)-2(3 a+4)=1$ nên $d$ là ước của 1 hay $d=1$.

Ví dụ 2

Cho $\mathrm{a}, b$ là các số nguyên dương sao cho $a^2+b^2$ chia hết cho tích $a . b$. Hãy tính giá trị của biểu thức

$A=\frac{a^2+b^2}{a b} .$

(Thi học sinh giỏi Toán 9 – Thành phố Hà Nội, năm 2002).

Lời giải

Gọi $d=(a, b)$ thì $a=d . a_1$ và $b=d . b_1$ với $\left(a_1, b_1\right)=1$. Ta có :

$a^2+b^2=d^2\left(a_1^2+b_1^2\right) \text { và } a b=d^2 a_1 b_1 .$

  • Vì $a^2+b^2$ chia hết cho $a b$ nên $a_1^2+b_1^2$ chia hết cho $a_l b_1$. Suy ra $a_1^2+b_1^2$ chia hết cho $a_l$ và $b_l$. Suy ra $a_1^2$ chia hết cho $b_l$ và $b_1^2$ chia hết cho $a_l$.
  • Vì $\left(a_1, b_1\right)=1$ nên $\mathrm{a}_1$ chia hết cho $\mathrm{b}_1$ và $\mathrm{b}_1$ chia hết cho $\mathrm{a}_1$.

Suy ra $a_l=b_1=1$. Vậy,

$A=\frac{d^2\left(a_1^2+b_1^2\right)}{d^2 c_1 b_1}=\frac{2 d^2 a_1^2}{d^2 c_1^2}=2$

Ví dụ 3

Chứng minh rằng với mọi số nguyên dương $n$ ta đều có $n^3+5 n$ chia hết cho 6 .

(Thi vào lớp 10 chuyên, DHKHTN ĐHQGHN năm 1996).

Lời giải

Ta có $n^3+5 n=\left(n^3-n\right)+6 n$. Để chứng minh $n^3+5 n$ chia hết cho 6 ta chứng minh $n^3-n$ chia hết cho 6 .

Do $n^3-n=n(n-1)(n+1)$ là tích của ba số nguyên liên tiếp nên chia hết cho 2 và 3 .

Vì $(2,3)=1$ nên $n^3-n$ chia hết cho tích $2 \times 3=6$.

Ví dụ 4

Cho $a, b, c$ là các số nguyên. Chứng minh rằng $a^3+b^3+c^3$ chia hết cho 6 khi và chỉ khi $a+b+c$ chia hết cho 6 .

Lời giải

Xét $A=a^3+b^3+c^3-a-b-c=\left(a^3-a\right)+\left(b^3-b\right)+\left(c^3-c\right)$.

Theo ví dụ 3 thì $a^3-a \cdot b^3-b$ và $c^3-c$ đều chia hết cho 6 . Suy ra $A$ chia hết cho 6. Vậy. $a^3+b^3+c^3$ chia hết cho 6 khi và chỉ khi $a+b+c$ chia hết cho 6 .

Ví dụ 5

Chứng minh $S=n^2+3 n-38$ không chia hết cho 49 , với mọi số tự nhiên $n$.

Lời giải

Giả sử tồn tại $n$ sao cho $S=n^2+3 n-38$ chia hết cho $+9$. Vì

$n^2-4 n+4=n^2+3 n-38-7(n-6)$

nên $n^2-t n+4$ chia hết cho 7 hay $(n-2)^2$ chia hết cho 7 . Suy ra $n-2$ chia hết cho 7 hay $n=2+7 t$.

Thay vào $S$ ta được : $S=49\left(t^2+t\right)-28$. Suy ra $S$ không chia hết cho 49 , trái với điều giả sử.

Vậy $S$ không chia hết cho 49 với mọi số tự nhiên $n$.

Ví dụ 6

Chứng minh rằng với mọi số tự nhiên $n$ ta luôn có

$A=2005^n+60^n-1897^n-168^n \text { chia hết cho } 2004 \text {. }$

Lời giải

Ta có $2004-12 \times 167$. Vì $(12,167)=1$ nên để chứng minh $A$ chia hết cho 2004 ta chứng minh $A$ chia hết cho 12 và 167 .

Ta có: $A=\left(2005^n-1897^n\right)-\left(168^n-60^n\right)$.

Áp dụng tính chất $a^{\prime \prime}-b^n$ chia hết cho $a-b$ với mọi $n$ tự nhiên và $a-b \neq 0$. ta suy ra $2005^n-1897^n$ chia hết cho $2005-1897=108=12 \times 9$.

Suy ra $2005^n-1897^n$ chia hết cho 12 . Mặt khác, 168 và 60 đều chia hết cho 12 nên $168^n-60^n$ chia hết cho 12 . Vậy $A$ chia hết cho 12 .

Tương tự như trên, ta có

$A=\left(2005^n-168^n\right)-\left(1897^n-60^n\right) .$

Cũng lập luận tương tự như trên, ta có $2005^n-168^n$ chia hết cho $2005-168=1837$; $1897^n-60^n$ chia hết cho $1897-60=1837$ và $1837=11 \times 167$ nên $2005^n-168^n$ và $1897^n-60^n$ chia hết cho 167 . Suy ra $A$ chia hết cho 167 .

Vậy ta có điều phải chứng minh.

BÀI TẬP

1. Chứng minh $a+2 \mathrm{~b}$ chia hết cho 3 khi và chỉ khi $b+2 a$ chia hết cho 3 .

2. Giả sử $a-c$ là ước của $a b+c d$. Chứng minh rằng $a-c$ cũng là ước của $a d+b c$.

3. Cho $a, b \in \mathbb{N}$. Chứng minh $\frac{11 a+2 b}{19} \in \mathbb{Z}$ khi và chỉ khi $\frac{18 a+5 b}{19} \in \mathbb{Z}$.

4. Cho $n$ nguyên dương. Chứng minh rằng

$(n !+1,(n+1) !+1)=1 .$

5. Cho $a, b$ là các số nguyên. Chứng minh rằng

$(5 a+3 b, 13 a+8 b)=(a, b) \text {. }$

6. Cho các số nguyên $m, n, p, q$ thỏa mãn $|p \cdot m-q \cdot n|=1$. Chứng minh rằng với mọi cặp số nguyên $a, b$ ta đều có

$(m a+n b, p a+q b)=(a, b) .$

7. Giả sử $(a, n)=p$ và $(b, n)=q$. Chứng minh rằng $(a b, n)=(p q, n)$.

8. Cho $a \leq b \leq c$ và $b=a q_1+r_1, c=a \cdot q_2+r_2$. Chứng minh rằng

$(a, b, c)=\left(a, r_1, r_2\right) .$

9. Chứng minh rằng với mọi số tự nhiên $n$, các phân số sau là phân số tối giản

(a) $\frac{21 n+4}{14 n+3}$;

(b) $\frac{15 n^2+8 n+6}{30 n^2+21 n+13}$;

(c) $\frac{n^3+2 n}{n^4+3 n^2+1}$.

$(I M O-1959)$.

10. Xác định các giá trị của $n$ để các phân số sau đây là phân số tối giản

(a) $\frac{n+22}{n+3}$

(b) $\frac{3 n+2}{2 n+3}$

(c) $\frac{18 n+3}{21 n+7}$.

11. Xét phân số

$A=\frac{n^2+4}{n+5} .$

Hỏi có bao nhiêu số tự nhiên $n$ trong khoảng từ 1 đến 2005 sao cho phân số $A$ chưa tối giản?

12. Chứng minh rằng với mọi bộ ba số lẻ $a, b, c$ ta đều có

$\left(\frac{a+b}{2}, \frac{b+c}{2}, \frac{c+a}{2}\right)=(a, b, c) .$

13. Cho $\mathrm{a}, \mathrm{b}, c$ là các số nguyên dương. Chứng minh

a) $(a, b, c)=\frac{(a, b, c) a b c}{(a, b)(b, c)(c, a)}$;

b) $[a, b, c]=\frac{(a, b, c)[a, b][b, c][c, a]}{a b c \ldots}$.

14. Cho $a_1, a_2, \ldots, a_n$ là các số nguyên dương và $n>1$. Đặt

$A=a_1 a_2 \ldots a_n, A_i=\frac{A}{a_i}(i=\overline{1, n}) .$

Chứng minh các đẳng thức sau :

a) $\left(a_1, a_2, \ldots, a_n\right)\left[A_1, A_2, \ldots, A_n\right]=A$;

b) $\quad\left[a_1, a_2, \ldots, a_n \mid\left(A_1, A_2, \ldots, A_n\right): A\right.$.

15. Cho $m, n$ là hai số tự nhiên nguyên tố cùng nhau. Hãy tìm ước số chung lớn nhất của hai số $A=m+n$ và $B=m^2+n^2$.

(Thi học sinh gioi Toán toàn quốc lớp 9 năm 1979).

16. Xác định ước số chung lớn nhất của hai số sau :

a) $(7 a+1,8 a+3)$

b) $(11 a+2,18 a+5)$

trong đó $a$ là một số nguyên cho trước.

17. Cho $n$ là một số nguyên dương. Hãy tính bội số chung nhỏ nhất của các số

$n, n+1, n+2 \text {. }$

18. Chứng minh rằng với mọi số nguyên dương $n$ ta có

$[1,2, \ldots, 2 n]=[n+1, n+2, \ldots, n+n]$

19. Cho số nguyên $a$ không chia hết cho 2 và 3 . Chứng minh rằng :

$A: 4 a^2+3 a+5 \text { chia hết cho } 6 \text {. }$

20. Chứng minh rằng $\frac{a}{3}+\frac{a^2}{2}+\frac{a^3}{6} \in \mathbb{Z}, \forall u \in \mathbb{Z}$.

21. Chứng minh rằng $\mathrm{A}(\mathrm{n})=\mathrm{n}^4+6 \mathrm{n}^3+11 \mathrm{n}^2+6 n$ chia hết cho 24 .

(Thi học sinh giỏi Toán toàn quốc – lớp 9 năm 1975)

22. Chứng minh rằng $n^5-n$ chia hết cho 30 , với mọi $n$.

23. Chứng minh rằng $m^3+3 m^2-m-3$ chia hết cho 48 , với mọi $m$ lẻ.

24. Chứng minh rằng $n^{12}-n^8-n^4+1$ chia hết cho 512 , với mọi $n$ lẻ.

25. Chứng minh rằng $A(n)=n^4 \cdots 14 n^3+71 n^2-154 n+120$ chia hết cho 24 , với mọi số tự nhiên $n$.

26. Chứng minh rằng $n^4-4 n^3-4 n^2+16 n$ chia hết cho 384 , với mọi số tự nhiên $n$ chẵn.

(Thi học sinh giỏi toàn quốc – lớp 9 năm 1970)

27. Tìm tất cả các số nguyên dương $n$ sao cho $n^2+9 n-2$ chia hết cho 11 .

28. Tìm tất cả các số nguyên $x$ sao cho : $\left(x^3-8 x^2+2 x\right)$ chia hết cho $x^2+1$.

(Thi vô địch Bun-ga-ri năm 1977)

29. Cho $f(x)=a x^2+b x+c$ thoả mãn : $f(x) \in \mathbb{Z}, \forall x \in \mathbb{Z}$. Hỏi $a, b, c$ có nhất thiết phải là các số nguyên hay không? Tại sao?

(Thi vào lớp 10 chuyên, $Đ H K H T N$ – ĐHQGHN năm 2001)

30. Chứng minh $n^2+n+2$ không chia hết cho 15 , với mọi $n$ thuộc $\mathbb{Z}$.

31. Chứng minh $n^2+3 n+5$ không chia hết cho 121 , với mọi $n$ thuộc $\mathbb{N}$.

32. Chứng minh $9 n^3+9 n^2+3 n-16$ không chia hết cho 343 , với mọi $n$ thuộc $\mathbb{N}$.

33. Chứng minh $4 n^3-6 n^2+3 n+37$ không chia hết cho 125 , với mọi $n$ thuộc $\mathbb{N}$.

34. Cho $a$ và $b$ thuộc $\mathbb{N}$. Chứng minh rằng $5 a^2+15 a b-b^2$ chia hết cho $49 \mathrm{khi}$ và chỉ khi $3 a+b$ chia hết cho 7 .

35. Cho $a, b \in \mathbb{N}$. Chứng minh rằng $2 a+b$ chia hết cho 7 khi và chỉ khi $3 a^2+10 a b-8 b^2$ chia hết cho 49 .

36. Cho $n \in \mathbb{N}$. Chứng minh rằng số $A=5^n\left(5^n+1\right)-6^n\left(3^n+2^n\right)$ chia hết cho 91 .

(Thi vào lớp 10 chuyên, ĐHSPHN năm 1998).

37. Cho $n \in \mathbb{N}$. Chứng minh $6^{2 n}+19^n-2^{n+1}$ chia hết cho 17 .

38. Chứng minh $2^{8 n} \cdot 5^{6 n}-1980^n-441^n+1$ chia hết cho 1979 , với mọi $n$ thuộc $\mathbb{N}$.

39. Chứng minh $118^n-101^n-16^n-1$ chia hết cho 234 , với mọi $n$ lẻ.

40. Chứng minh $11^{n+2}+12^{2 n+1}$ chia hết cho 133 , với mọi $n$ thuộc $\mathbb{N}$.

41. Chứng minh $5^{2 n-1} \cdot 2^{n+1}+3^{n+1} \cdot 2^{2 n-1}$ chia hết cho 38 , với mọi $n$ thuộc $\mathbb{N}^*$.

42. Chứng minh $5^{n+2}+26.5^n+8^{2 n+1}$ chia hết cho 59. với mọi $n$ thuộc $\mathbb{N}$.

43. Tìm số tự nhiên $n$ lớn nhất sao cho $: 29^n$ là ước của 2003 !.

44. Tìm số tự nhiên $k$ lớn nhất sao cho : $(1994 \text { ! })^{1995} \quad \vdots 1995^k$.

(Thi học sinh giỏi Toán toàn quốc – lớp 9. năm 199t).

45. Cho $n$ thuộc $\mathbb{N}$ và $n>3$. Chứng minh rằng nếu $2^n=10 a+b(0<b<10)$ thì tích $a \cdot b$ chia hết cho 6 .

(Thi học sinh giỏi Toán toàn quốc lớp 9 năm 1983).

46. Cho $n$ thuộc $\mathbb{N}, n \geq 1$. Chứng minh $T_n=1^5+2^5+\ldots+n^5$ chia hết cho tổng của $n$ số tự nhiên đầu tiên $S_n=1+2+\ldots+n$.

(Thi vào lớp 10 chuyên $Đ H S P H N$ năm 2001).

47. Tìm $n$ nguyên dương sao cho : $(n-1)$ ! chia hết cho $n$.

(Thi vô địch Hungari năm 1951).

48. Xác định $n$ nguyên dương $(\mathrm{n} \geq 3$ ) sao cho số $A=1.2 .3 \ldots \mathrm{n}$ (tích của $n$ số nguyên dương đầu tiên) chia hết cho $B=1+2+\ldots+n$.

(Thi vào lớp 10 chuyên ĐHKHTN – ĐHQGHN năm 1994).

49. Cho $a$ và $m$ là các số nguyên dương và $a>1$. Chứng minh rằng

$\left(\frac{a^m-1}{a-1}, a-1\right)=(m, a-1) .$

50. Cho $a, m, n$ là các số nguyên dương và $a \neq 1$. Chứng minh rằng $a^n-1 \backslash a^m-1$ khi và chỉ khi $n \backslash m$.

51. Cho $a, m, n$ là các số nguyên dương và $a>1$. Chứng minh rằng

$\left(a^m-1, a^n-1\right)=a^{(m, n)}-1 .$

52. Cho $a, b$ là hai số nguyên dương không nhỏ hơn 2 và nguyên tố cùng nhau. Chứng minh rằng nếu $m, n$ là hai số nguyên dương thỏa mãn $a^n+b^n \backslash a^m+b^m$ thì ta cũng có $n . \mid m$.

53. Cho $a, b, n$ là các số nguyên dương. Biết rằng với mọi số tự nhiên $k \neq b$ ta đều có $k^n-a$ chia hết cho $k-b$. Chứng minh $a=b^n$.

54. Chứng minh rằng tồn tại vô hạn số tự nhiên $n$ sao cho : $4 n^2+1$ chia hết cho cả 5 và 13 .

55. Giả sử $1-\frac{1}{2}+\frac{1}{3}-\ldots+\frac{1}{1319}=\frac{p}{q}$, trong đó $p, q$ là các số nguyên. Chứng minh rằng $p$ chia hết cho 1979.

56. Cho $a_1, a_2, \ldots, a_n \in{1,-1}, n \in \mathbb{N}^*$ và thoả mãn :

$a_1 a_2+a_2 a_3+\ldots+a_n a_1=0 \text {. }$

Chứng minh $n$ chia hết cho 4 .

57. Chứng minh rằng tổng bình phương của $p$ số nguyên liên tiếp ( $p$ là số nguyên tố, $p>3$ ) chia hết cho $p$.

58. Cho số nguyên $a$ không nhỏ hơn 2 . Hỏi có tồn tại hay không số tự nhiên $A$ sao cho

$a^{2001}<A<a^{2002}$

và $A$ có ít nhất 600 chữ số 0 ở tận cùng?

59. Có tồn tại hay không 4004 số nguyên dương sao cho tổng của 2003 số bất kì đều không chia hết cho 2003 .

(Balkan 2003).

60. Tìm một cặp số nguyên dương $(a, b)$ thoả mãn đồng thời các điều kiện sau :

a) $a b(a+b)$ không chia hết cho 7 .

b) $(a+b)^7-a^7-b^7$ chia hết cho $7^7$.

(IMO-198t).

61. Giả sử $a, b$ là hai số nguyên dương khác nhau. Chứng minh rằng tồn tại vô số số tự nhiên $n$ sao cho $a+n$ và $b+n$ là hai số nguyên tố cùng nhau.

 

LỜI GIẢI – HƯỚNG DẪN – ĐÁP SỐ

1. Suy ra từ đẳng thức : $(a+2 b)+(b+2 a)=3(a+b)$.

2. Suy ra từ đẳng thức : $(a b+c d)-(a d+b c)=(a-c)(b-d)$.

3. Suy ra từ đẳng thức : $5 \cdot \frac{11 a+2 b}{19}-2 \cdot \frac{18 a+5 b}{19}=a$.

4. Giả sử $d=(n !+1,(n+1) !+1)$.

Ta có $d \backslash n !+1$ và $d \backslash(n+1) !+1$ nên $d \backslash(n+1) !+1-n !-1=n ! . n\quad\quad(1)$.

Vì $d \backslash n !+1$ nên $(d, n)=(d, n !)=1$. Từ (1) suy ra $d=1$.

5. Giả sử $d=(a, b)$ và $d^{\prime}=(5 a+3 b, 13 a+8 b)$.

Vì $d \backslash a$ và $d \backslash b$ nên $d \backslash 5 a+3 b$ và $d \backslash 13 a+8 b$. Suy ra $d \backslash d\quad(1)$.

Vì $d^{\prime} \backslash 5 a+3 b$ và $d^{\prime} \backslash 13 a+8 b$ nên

$d^{\prime} \backslash 8(5 a+3 b)-3(13 a+8 b)=a$

và $\quad d^{\prime} \backslash 5(13 a+8 b)-13(5 a+3 b)=b$.

Suy ra $d^{\prime} \backslash d\quad\quad(2)$.

Từ (1) và (2) ta suy ra $d^{\prime}=d$.

6. Giải tương tự bài $1.5$.

7. Ta có $(a, n)=p$ nên $a=p \cdot a_1, n=p n_1$ với $\left(a_1, n_1\right)=1$. Suy ra

$(a b, n)=\left(p a_1 b, p n_1\right)=p \cdot\left(a_1 b, n_1\right)=p\left(b, n_1\right)=(p b, n)$

$\text { Vì }(b, n)=q \text { nên } b=q \cdot b_1 \text { và } n=q \cdot n_2 \text { với }\left(b_1, n_2\right)=1 . \text { Suy ra }$

$(p b, n)=\left(p \cdot q \cdot b_1, q \cdot n_2\right)=q\left(p b_1, n_2\right)=q\left(p, n_2\right)=(p q, n)$

8. Giải tương tự bài $1.5$.

9. a) Giả sử $(21 n+4,14 n+3)=d(d \geq 1)$.

Ta có $d \backslash 21 n+4$ và $d \backslash 14 n+3$ nên $d \backslash 3(14 n+3)-2(21 n+4)=1$.

Vậy $d=1$.

Các bạn tự giải các câu b) và c).

10. a) Ta có $\frac{n+22}{n+3}=1+\frac{19}{n+3}$. Phân số đã cho tói gian khi và chi khi $(n+3,19)=1$ hay $n \neq 19 m-3$.

b) Vì $(2 n+3,2)=1$ nên phân số đã cho tối gian khi và chi khi phân số sau tối gịản

$B=\frac{2(3 n+2)}{2 n+3}=3-\frac{5}{2 n+3} .$

Phân số $B$ tối giann khi và chi khi $(2 n+3,5)=1$.

Ta có $(2 n+3,5) \neq 1$ khi và chi khi $5 \backslash 2 n+3$ hay $2 n+3=5 a$.

Xét $2 n+3=5 a$, ta có $n=2 a+\frac{a-3}{2}$.

Vì $n$ và $a$ là các số nguyên nên $a-3=2 m$, từ đó có $n=5 m+6$.

Vậy phân số đã cho tối giản khi và chi khi $n \neq 5 m+6$.

c) Đáp số: $n \neq 7 m+1$.

11. Giả sử $A$ là phân số chưa tối giản. Đặt $d=\left(n^2+4, n+5\right)$ suy ra $d>1$. Ta có

$d \backslash(n+5)^2-\left(n^2+4\right)=10 n+21=10(n+5)-29$

nên $d \backslash 29$ suy ra $d=29$.

Ngược lại, nếu $n+5$ chia hết cho 29 thì có thể đặt

$n+5=29 . m\left(m \in \mathbb{N}^*\right)$

12. Giải tương tự bài 5 .

13. Giải tương tự bài 5 .

14. Giải tương tự bài 5 .

15. Giả sử $d=(A, B)(d \geq 1)$. Ta có $d \backslash A^2-B$ suy ra $d \backslash 2 m n\quad(1)$.

Vì $d \backslash A$ nên $d \backslash 2 n \cdot A$ hay $d \backslash 2 m n+2 n^2$. Suy ra $d \backslash 2 n^2\quad(2)$.

Tương tự ta cũng có $d \backslash 2 m^2\quad(3)$

Vì $(m, n)=1$ nên $m, n$ không cùng chẵn. Xét các trường hợp:

  • Nếu $m, n \cdot$ khác tính chã̃n lẻ thì $d$ lẻ. Từ (2) và (3) ta suy ra $d \backslash m^2$ và $d \backslash$ $n^2$. Vì $(m, n)=1$ nên $d=1$.

  • Nếu $m, n$ cùng lẻ thì $d$ chã̃n. Đặt $d=2 d$, từ (2) và (3) ta suy ra $d \backslash m^2$ và $d^n \backslash n^2$. Vì $(m, n)=1$ nên $d^n=1$. Suy ra $d=2$.

16. a) Đặt $d=(7 a+1,8 a+3)$.

Ta có $d \backslash 7(8 a+3)-8(7 a+1)=13$ nên $d=1$ hoặc $d=13$.

Để $d=13$ thì điều kiện cần và đủ là $13 \backslash 7 a+1$.

Xét phương trình: $7 a+1=13 x$.

Ta có $a=2 x-\frac{x+1}{7}$ là một số nguyên nên $7 \backslash x+1$.

Đặt $x+1=7 m$ ta được $a=13 m-2, m \in \mathbb{Z}$.

Vậy, khi $a=13 m-2, m \in \mathbb{Z}$ thì $(7 a+1,8 a+3)=13$,

$a \neq 13 m-2, m \in \mathbb{Z}$ thì $(7 a+1,8 a+3)=1$

b) Giải tương tự câu a).

Đáp Số:

  • Nếu $a=19 m-14, m \in \mathbb{Z}$ thì $(11 a+2,18 a+5)=19$

  • Nếu $a \neq 19 m-14, m \in \mathbb{Z}$ thì $(11 a+2,18 a+5)=1$.

18. Giả sử $m=[1,2, \ldots, 2 n]$ và $m^{\prime}=[n+1, \ldots, n+n](n \geq 2)$.

Để chứng minh $m=m^{\prime}$ ta chứng minh $m \backslash m^{\prime}$ và $m^{\prime} \backslash m$.

Vì $n+1, n+2, \ldots, n+n$ là ước của $m$ nên $m^{\prime} \backslash m$.

Ngược lại, xét số $a \in{1,2, \ldots, n}$ tùy ý.

Trong $a$ số nguyên liên tiếp $n+1, \ldots, n+a$ luôn có một số chia hết cho $a$ nên $a \backslash m^{\prime}$. Suy ra các số $1,2, \ldots, 2 n$ đều là ước của $m^{\prime}$ hay $m \backslash m^{\prime}$.

Vậy $m=m^{\prime}$.

19. Vì $a$ không chia hết cho 2 và 3 nên $a$ có dạng: $a=6 m \pm 1(m \in \mathbb{Z})$

  • Với $a=6 m+1$ ta có $A=4(6 m+1)^2+3(6 m+1)+5$

$=6\left(24 m^2+11 m+2\right) \vdots 6 \text {. }$

  • Với $a=6 m-1$ ta có $A=4(6 m-1)^2+3(6 m-1)+5$

$=6\left(24 m^2-5 m+1\right) \vdots 6 \text {. }$

Vậy $A$ chia hết cho 6 , với mọi $a$ không chia hết cho 2 và 3 .

20. Ta có $\frac{a}{3}+\frac{a^2}{2}+\frac{a^3}{6}=\frac{a(a+1)(a+2)}{6}$.

Vì $a(a+1)(a+2)$ là tích của ba số nguyên liên tiếp nên chia hết cho 6 từ đó suy ra đpcm.

21. Ta có $A(n)=n(n+1)(n+2)(n+3)$.

Vì tích của ba số nguyên liên tiếp chia hết cho 3 nên $A(n)$ chia hết cho 3 . Trong bốn số nguyên liên tiếp luôn có hai số chẵn liên tiếp, một trong hai số đó chia hết cho 4 nên $A(n)$ chia hết cho 8 .

Vì $(3,8)=1$ nên $A(n)$ chia hết cho $3 \times 8=24$.

22. Ta có $30=6 \times 5$. Vì $(6,5)=1$ nên để chứng minh $n^5-n$ chia hết cho 30 ta chứng minh $n^5-n$ chia hết cho 6 và 5 .

Ta có $n^5-n=(n-1) n(n+1)\left(n^2+1\right)$. Vì $(n-1) n(n+1)$ là tích ba số nguyên liên tiếp nên chia hết cho 2 và 3 .

Suy ra $n^5-n$ chia hết cho $2 \times 3=6$.

Mặt khác ta lại có

$n^5-n=(n-1) n(n+1)(n^2-4+5)$

$=(n-2)(n-1) n(n+1)(n+2)+5(n-1) n(n+1) .$

Vì $(n-2)(n-1) n(n+1)(n+2)$ là tích cua năm sổ nguyên liên tiếp nên chia hết cho 5 .

Suy ra $n^5-n$ chia hết cho 5 .

Vậy $n^5-n$ chia hết cho 30 .

23. Đặt $A=m^3+3 m^2-m-3$.

Ta có $A=(m+3)\left(m^2-1\right)=(m+3)(m+1)(m-1)$.

Vì $m$ lẻ nên $m=2 n+1(n \in \mathbb{Z})$, từ đó suy ra $A=8 .(n+2)(n+1) n \Rightarrow$ đpcm.

24. Đặt $A=n^{12}-n^8-n^4+1$. Ta có

$A=\left(n^4-1\right)\left(n^8-1\right)=\left[\left(n^2-1\right)\left(n^2+1\right)\right]^2\left(n^4+1\right) .$

Vì $n$ lẻ nên $n=2 m+1$, suy ra $A=64 \cdot[m(m+1)]^2\left(2 m^2+2 m+1\right)^2\left(n^4+1\right)$.

25. Ta có $24=3 \times 8$. Để chứng minh $A(n)$ chia hết cho 24 ta chứng minh $A(n)$ chia hết cho 3 và 8 .

Ta có $A(n)=(n-2)(n-3)(n-4)(n-5)$ (bạn đọc tự phân tích).

Vì $A(n)$ là tích của bốn số nguyên liên tiếp nên $A(n)$ chia hết cho 3 .

Trong bốn số nguyên liên tiếp $n-2, n-3, n-4, n-5$ luôn có hai số chã̃n liên tiếp. Một trong hai số đó chia hết cho 4 , số còn lại chia hết cho 2 nên $A(n)$ chia hết cho 8 . Vì $(3,8)=1$ nên $A(n)$ chia hết cho $3 \times 8=24$.

26. Đặt $A=n^4-4 n^3-4 n^2+16 n$. Ta có $A=n(n-4)\left(n^2-4\right)$.

Vì $n$ chẵn nên $n=2 m(m \in \mathbb{Z})$. Từ đó suy ra $A=16 .(m-2)(m-1) m(m+1)$.

Vì $(m-2)(m-1) m(m+1)$ là tích của 4 số nguyên liên tiếp nên chia hết cho 8 và 3 .

Từ đó có đpcm.

27. Đáp số: $n=11 m+6$ hoặc $n=11 m+7(m \in \mathbb{N})$.

Hướng dẫn :

$\text { Ta có } n^2+9 n-2 \vdots 11 \Leftrightarrow n^2-2 n-2 \vdots 11 \Leftrightarrow 4\left(n^2-2 n-2\right) \vdots 11$

$\Leftrightarrow 4 n^2-8 n+3 \vdots 11 \Leftrightarrow(2 n-1)(2 n-3) \vdots 11 .$

28. Đáp số: $x \in{-8,0,2}$.

Giả sử $\left(x^3-8 x^2+2 x\right) \vdots\left(x^2+1\right)$ suy ra

$x\left(x^2+1\right)-8\left(x^2+1\right)+x+8 \vdots\left(x^2+1\right) \text {. }$

hay $x+8 \vdots\left(x^2+1\right)\quad\quad( * )$

  • Nếu $x+8=0$ thì $x=-8$, thỏa mãn điều kiện đề bài.

  • Nếu $x \neq-8$ thì tù $\left(^*\right)$ ta phải có $|x+8| \geq x^2+1\quad\quad(1)$.

Bất phương trình (1) cho ta $x \in{-2,-1,0,1,2,3}$

Thử trực tiếp ta được $x=0$ và $x=2$ thỏa mãn.

Cách 2

$\text { Ta có } x+8 \vdots\left(x^2+1\right) \Rightarrow x^2+8 x \vdots\left(x^2+1\right) \Rightarrow 8 x-1 \vdots\left(x^2+1\right)$

$\Rightarrow  8(x+8)-(8 x-1) \vdots x^2+1 \Rightarrow 65 \vdots\left(x^2+1\right)$

$\Rightarrow x^2+1 \text { là ước dương của } 65$

$\Rightarrow x^2+1 \in{1,5,13,65} .$

29. Cho $x=0$ suy ra $f(0)=c \in \mathbb{Z}$. Các số $a, b$ không nhất thiết phải là các số nguyên.

Ví dụ, chọn $a=b=\frac{1}{2}$, ta có

$f(x)=\frac{x(x+1)}{2}+c \in \mathbb{Z}, \forall x \in \mathbb{Z} \text {. }$

30. Giả sử $n^2+n+2 \vdots 15$ ta có $n^2+n+2 \vdots 3\quad\quad(1)$.

Từ (1) suy ra $n$ không chia hết cho 3 .

Vậy $n$ có dạng $3 k+1$ hoặc $3 k-1(k \in \mathbb{Z})$, ta có

$n^2-1=(n-1)(n+1) \vdots 3$

$\Rightarrow n^2+n+2=\left(n^2-1\right)+n+3 \text { không chia hết cho } 3 \text {, mâu thuẫn với (1). }$

31. Giả sử $n^2+3 n+5 \vdots 121$ suy ra $n^2+3 n+5 \vdots 11$ hay $4 n^2+12 n+20 \vdots 11$ Vậy

$4 n^2+12 n+9 \vdots 11 \Rightarrow(2 n+3)^2 \vdots 11 \Rightarrow 2 n+3 \vdots 11$

Nhưng khi đó

$4\left(n^2+3 n+5\right)=(2 n+3)^2+11$ không chia hết cho 121 , mâu thuẫn với điều giả sử trên, từ đó suy ra đpcm.

32. Giải tương tự bài 31

33. Giải tương tự bài 31

34. $\Rightarrow \text { ) Giả sử } 5 a^2+15 a b-b^2 \vdots 49 \Rightarrow 5 a^2+15 a b-b^2 \vdots 7$

$\Rightarrow 9 a^2+6 a b+b^2 \vdots 7 \Rightarrow(3 a+b)^2 \vdots 7 \Rightarrow 3 a+b \vdots 7 \text {. }$

$\Leftrightarrow) \text { Giả sử } 3 a+b \vdots 7 \Rightarrow 3 a+b=7 c(c \in \mathbb{Z}) \Rightarrow b=7 c-3 a$

$\Rightarrow 5 a^2+15 a b-b^2=5 a^2+15 a(7 c-3 a)-(7 c-3 a)^2$

$=49\left(c^2+3 a c-a^2\right) \vdots 49 .$

35. Giải tương tự bài 34.

36. Ta có $91=7 \times 13$. Vì $(7,13)=1$ nên để chứng $\operatorname{minh} A \vdots 91$ ta chi cần chứng $\operatorname{minh} A \vdots 7$ và $A \vdots 13$.

  • Chứng $\operatorname{minh} A \vdots$ 7: Ta viết $A$ dưới dạng: $A=\left(25^n-18^n\right)-\left(12^n-5^n\right)$. Vì $\left(25^n-18^n\right) \vdots 25-18=7$ và $\left(12^n-5^n\right) \vdots 12-5=7$ nên $A \vdots 7$.

  • Chứng $\operatorname{minh} A \vdots$ 13: Ta viết $A$ dưới dạng: $A=\left(25^n-12^n\right)-\left(18^n-5^n\right)$ Vì $\left(25^n-12^n\right) \vdots 25-12=13$ và $\left(18^n-5^n\right) \vdots 18-5=13$ nên $A \vdots 13$. Vậy $A \vdots 91, \forall n \in \mathbb{N}$.

37. Đặt $A(n)=6^{2 n}+19^n-2^{n+1}$.

Ta có $A=36^n+19^n-2 \cdot 2^n=\left(36^n-2^n\right)+\left(19^n-2^n\right)$.

Vì $36^n-2^n \vdots 34(=36-2)$ nên $36^n-2^n \quad \vdots 17$ và $19^n-2^n \vdots 17(=19-2)$ nên $A(n) \vdots 17$.

38. Đặt $A(n)=2^{8 n} \cdot 5^{6 n}-1980^n-441^n+1$.

Ta có $A=\left(4000000^n-441^n\right)-(1980-1)^n$.

Vì $\left(4000000^n-441^n\right) \vdots 3999599(=4000000-441=2021)$ nên

$\left(4000000^n-441^n\right) \vdots 1979$

và $1980^n-1 \vdots 1979(=1980-1)$ nên $A(n) \vdots 1979$.

39. Giải tương tự bài $36$.

40. Giải tương tự bài $37$.

41. Giải tương tự bài 36.

42. Giải tương tự bài 37.

43. Các số chia hết cho 29 trong khoảng từ 1 đến 2003 là:

$29 \times 1,29 \times 2,29 \times 3, \ldots, 29 \times 69 \text {. }$

Suy ra $2003 !=29^{69} \cdot 69 ! . A$, trong đó $(A, 29)=1$.

Các số chia hết cho 29 trong khoảng từ 1 đến 69 là: $29 \times 1,29 \times 2$.

Suy ra: $69 !=29^2 \cdot 2 ! . B$, trong đó $(B, 29)=1$.

Vậy $2003 !=29^{71} \cdot 2 \cdot A \cdot B$, trong đó $(A \cdot B, 29)=1$.

Từ đó suy ra $n$ cần tìm là 71 .

44. Đáp số: $k=217455$.

$1995=3 \times 5 \times 7 \times 19 .$

Ta :cần tìm số mũ lớn nhất của 19 trong phân tích tiêu chuẩn của số $(1994 !)^{1995}$. Xem lại bài 43.

45. Ta có $2^n=10 a+b$ nên $b \vdots 2$ hay $a b \vdots 2$. Ta chứng minh $a b \vdots 3$ :

Từ đẳng thức $2^n=10 a+b$ suy ra $2^n$ có chữ số tận cùng là $b$.

Đặt $n=4 k+r(k, r \in \mathbb{N}, 0 \leq r \leq 3)$ ta có $2^n=16^k \cdot 2^r$.

Nếu $r=0$ thì $2^n=16^k$ có chữ số tận cùng là 6 suy ra $b=6$ hay $a b \vdots 6$.

Nếu $1 \leq r \leq 3$ thì $2^n-2^r=2^r\left(16^k-1\right) \vdots 10$ suy ra $2^n$ có tận cùng là $2^r$.

Vậy ta có $b=2^r$, từ đó suy ra

$10 a=2^n-2^r=2^r\left(16^k-1\right) \vdots 3 \Rightarrow a \vdots 3 \Rightarrow a b \vdots 3 .$

46. Ta có $2 S_n=n(n+1)$.

Mặt khác, sử dụng tính chất $a^n+b^n \vdots(a+b), \forall a, b \in \mathbb{N}^*$ và $n$ lẻ ta có

$2 T_n=\left(1^5+n^5\right)+\left(2^5+(n-1)^5\right)+\ldots+\left(n^5+1\right) \vdots(n+1)\quad\quad(1) .$

$2 T_n=\left(1^5+(n-1)^5\right)+\left(2^5+(n-2)^5\right)+\ldots+\left((n-1)^5+1\right)+2 n^5 \vdots n\quad\quad(2) .$

Do $(n, n+1)=1$, từ $(1)$ và $(2)$ ta suy ra

$2 T_n \vdots n(n+1)=2 S_n \Rightarrow T_n \vdots S_n .$

Tổng quát, ta có thể chứng minh được:

$1^k+2^k+\ldots+n^k$ chia hết cho $1+2+\ldots+n, \forall n, k \in \mathbb{N}, n \geq 1$ và $k$ lẻ.

47. Dễ thấy $n=1$ thỏa mãn và $n=4$ không thỏa mãn. Xét $n>1$ và $n \neq 4$ :

Từ giả thiết suy ra $n$ là hợp số, như vậy $n$ có thể viết được dưới dạng $n=p . q$, trong đó $p, q$ là các số nguyên dương thỏa mãn: $2 \leq p, q \leq\left[\frac{n}{2}\right]$.

  • Nếu $p \neq q$ thì trong tích $(n-1) !=1.2 \ldots n$ chứa cả hai số $p$ và $q$ nên $(n-1)$ ! chia hết cho $n$.

  • Nếu $p=q$ thì $p, q>2$ và trong tích $(n-1)$ ! chứa cả $p$ và $2 p$ nên $(n-1)$ ! chia hết cho $n$.

48. Xem lời giải bài 47.

49. Giả sử $d=\left(\frac{a^m-1}{a-1}, a-1\right)$ và $d^{\prime}=(m, a-1)$. Ta có

$\frac{a^m-1}{a-1}=a^{m-1}+a^{m-2}+\ldots+a+1$

$=\left(a^{m-1}-1\right)+\left(a^{m-2}-1\right)+\ldots+(a-1)+m .$

Vì $a^i-1$ chia hết cho $a-1$ và do đó chia hết cho $d$ với mọi $i \geq 1$ nên $d \backslash m$. Suy ra $d \backslash d$.

Ngược lại, vì $d^{\prime} \backslash a-1$ nên $d^{\prime} \backslash a^i-1$ với mọi $i \geq 1$. Hơn nữa, $d^{\prime} \backslash m$ nên $d^{\prime} \backslash$ $\frac{a^m-1}{a-1}$ và do đó $d^{\prime} \backslash d$.

Vậy $d=d$.

50. $\Rightarrow)$ Giả sử $a^n-1 \backslash a^m-1$. Ta chứng minh $n \backslash m$.

Đặt $m=q n+r, 0 \leq r<n$. Ta có

$a^m-1=a^{n q+r}-1=a^r\left(a^{n q}-1\right)+a^r-1$

$\text { Vì } a^n-1 \backslash a^m-1 \text { và } a^n-1 \backslash a^{n q}-1 \text { nên } a^n-1 \backslash a^r-1 .$

$\text { Mặt khác } 0 \leq a^r-1<a^n-1 \text { nên } a^r-1=0 \text { hay } r=0 .$

$\Leftarrow)$ Dễ dàng chứng minh được.

51. Giả sử $d=(m, n)$ và $d=\left(a^m-1, a^n-1\right)$. Đặt $m=d . m_1, n=d . n_1$ ta có $a^m-1=\left(a^d\right)^{m_1}-1$ chia hết cho $a^d-1$

và $\quad a^n-1=\left(a^d\right)^{n_1}-1$ chia hết cho $a^d-1$

nên d’ chia hết cho $a^d-1\quad\quad(1)$.

Mặt khác, vì $d=(m, n)$ nên tồn tại hai số nguyên dương $x, y$ sao cho $m x-n y=d$. Vì $d^n \backslash a^m-1$ và $d^{\prime} \backslash a^n-1$ nên $d^{\prime} \backslash a^{m x}-1$ và $d^n \backslash a^{n y}-1$.

Suy ra $d^n \backslash a^{m x}-a^{n y}=a^{n y}\left(a^d-1\right)$. Vì $\left(d^n, a^{n y}\right)=1$ nên $d^n \backslash a^d-1\quad\quad(2)$.

Từ (1) và (2) ta có đpcm.

52. Giả sử $m$ không chia hết cho $n$, tức là $m=q \cdot n+r$ với $0<r<n$. Ta có

$a^m+b^m=a^{m-n}\left(a^n+b^n\right)-b^n\left(a^{m-n}-b^{m-n}\right) .$

Suy ra $a^n+b^n \backslash a^{m-n}-b^{m-n}$.

Nếu $q-1>0$, lại làm tương tự như trên tai có

$a^{m-n}-b^{m-n}=a^{m-2 n}\left(a^n+b^n\right)-b^n\left(a^{m-2 n}+b^{m-2 n}\right) .$

Suy ra $a^n-b^n \backslash a^{m-2 n}+b^{m-2 n}$.

Cứ lặp lại cách làm trên ta suy ra $a^n+b^n \backslash a^{m-n \cdot k}+(-1)^k b^{m-n k}, \forall k \leq q$.

Đặc biệt với $k=q$ ta có $a^n+b^n \backslash a^r+(-1)^q b^r$. Điều này không xảy ra vì

$0<\left|a^r+(-1)^q b^r\right|<a^r+b^r<a^n+b^n \text {. }$

53. Ta có $k-b \backslash k^n-a=\left(k^n-b^n\right)+\left(b^n-a\right)$ và $k-b \backslash k^n-b^n$ nên $k-b \backslash b^n-a$. Vì điều này đúng với mọi $k$ nên chọn $k$ sao cho $k-b>\left|b^n-a\right|$. Vì $b^n-a$ chia hết cho $k-b$ nên $b^n-a=0$ hay $a=b^n$.

54. Cần tìm $n$ sao cho $4 n^2+1$ chia hết cho 65 . Đặt $n=65 k+r$, ta chọn $r$ sao cho $4 r^2+1=65$ hay $r=\pm 4$.

Khi đó, mọi số $n$ có dạng $65 k \pm 4$ đều thỏa mãn.

55. Ta có

$\text { Ta có } \frac{p}{q}=\left(1+\frac{1}{2}+\ldots+\frac{1}{1319}\right)-2\left(\frac{1}{2}+\frac{1}{4}+\ldots+\frac{1}{1318}\right)$

$=\left(1+\frac{1}{2}+\ldots+\frac{1}{1319}\right)-\left(1+\frac{1}{2}+\frac{1}{3}+\ldots+\frac{1}{659}\right)$

$=\frac{1}{660}+\ldots+\frac{1}{1319}$

$\Rightarrow 2 \cdot \frac{p}{q}=\left(\frac{1}{660}+\frac{1}{1319}\right)+\left(\frac{1}{661}+\frac{1}{1318}\right)+\ldots+\left(\frac{1}{1319}+\frac{1}{660}\right)=\frac{1979 . A}{B} \text {. }$

Từ đó suy ra $p$ chia hết cho 1979 .

56. Đặt $x_1=a_1 \cdot a_2, x_2=a_2 \cdot a_3, \ldots, x_n=a_n \cdot a_1$.

Vì $a_1, a_2, \ldots, a_n \in{-1,1}$ nên $x_1, x_2, \ldots, x_n \in{-1,1}$.

Ta có $x_1+x_2+\ldots+x_n=0$ suy ra trong các số $x_1, x_2, \ldots, x_n$ số các số 1 bằng số các số $-1$.

Giả sử số các số 1 là $m\left(\mathrm{~m} \in \mathbb{N}^*\right)$ thì $n=2 m$ và

$x_1 x_2 \ldots x_n=(-1)^m \text {. }\quad\quad(1)$

Mặt khác, $x_1 x_2 \ldots x_n=\left(a_1 a_2 \ldots a_n\right)^2=1\quad\quad(2)$.

Từ (1) và (2) suy ra $m$ chã̃n và điều đó có nghĩa là $n$ chia hết cho 4 .

57. Giả sử $p$ số nguyên liên tiếp đó là: $a+1, a+2, \ldots, a+p(\mathrm{a} \in \mathbb{Z})$.

Đặt $A=(a+1)^2+(a+2)^2+\ldots+(a+p)^2$. Ta có

$\mathrm{A}=p \cdot a^2+2(1+2+\ldots+p) \cdot a+\left(1^2+2^2+\ldots+p^2\right)$

Mặt khác: $1+2+\ldots+p=\frac{p(p+1)}{2}, 1^2+2^2+\ldots+p^2=\frac{p(p+1)(2 p+1)}{6}$.

Suy ra $6 A=p\left[6 a^2+6(p+1) a+(p+1)(2 p+1)\right]$ chia hết cho $p$.

Do $p$ là số nguyên tố và $p>3$ nên $(p, 6)=1$. Vậy $A$ chia hết cho $p$.

58. Vì $a^{2001}-a^{2000}=a^{2000}(a-1) \geq 2^{2000}=1024^{200}>10^{600}$ nên giữa $a^{2000}$ và $a^{2001}$ có ít nhất $10^{600}$ số nguyên dương liên tiếp. Trong số đó, tồn tại một số chia hết cho $10^{600}$, đó chính là số $A$ cần tìm.

59. Đáp số: 

Tồn tại. Có thể chọn 2002 số chia hết cho 2003 và 2002 số chia cho 2003 dư 1 .

60. Ta có $(a+b)^7-a^7-b^7=7 a b(a+b)\left(a^2+a b+b^2\right)^2$.

Chọn $b=1$ và $a^2+a+1=7^3$ (bạn đọc tự tính $a$ ) ta có đpcm.

61. Không mất tính tổng quát, giả sử $c=a-b>0$.

Ta có $b=q c+r$, với $q \geq 0,0 \leq r<c, q$ và $r$ không đồng thời bằng 0 .

Các số $n$ có dạng $n=c+1-r+k c, k \in \mathbb{Z}$ đều thoả mãn.

 

 

 

 

 

 

 

 

 

 

 

 

 

 

 

 

 

 

 

 

 

 

 

 

 

 

 

 

 

 

 

 

 

 

 

 

 

 

 

 

 

 

 

 

 

 

 

 

 

 

 

 

 

 

 

 

 

 

 

 

 

 

 

 

 

 

 

 

 

 

 

 

 

 

 

 

 

 

 

 

 

 

CHUYÊN ĐỀ: TÍNH CHIA HẾT ĐỐI VỚI SỐ NGUYÊN

CHỨNG MINH QUAN HỆ CHIA HẾT

Gọi $\mathrm{A}(\mathrm{n})$ là một biểu thức phụ thuộc vào $\mathrm{n}(\mathrm{n} \in \mathbf{N}$ hoặc $\mathrm{n} \in \mathbf{Z})$.

Chú ý 1 : Để chứng minh biểu thức $\mathrm{A}(\mathrm{n})$ chia hết cho một số $\mathrm{m}$, ta thường phân tích biểu thức $\mathrm{A}(\mathrm{n})$ thành thừa số, trong đó có một thừa số là $\mathrm{m}$. Nếu $\mathrm{m}$ là hợp số, ta phân tích nó thành một tích các thừa số đôi một nguyên tố cùng nhau, rồi chứng minh $\mathrm{A}(\mathrm{n})$ chia hết cho tất cả các số đó. Nên lưu ý đến nhận xét : Trong $\mathrm{k}$ số nguyên liên tiếp, bao giờ cũng tồn tại một bội số của k.

Ví dụ 1. Chứng minh rằng $A=n^3\left(n^2-7\right)^2-36 n$ chia hết cho 5040 với mọi số tự nhiên $n$.

Giải : Phân tích ra thừa số : $5040=2^4 \cdot 3^2 \cdot 5 \cdot 7$.

Phân tích $A=n\left[n^2\left(n^2-7\right)^2-36\right]=n\left[\left(n^3-7 n\right)^2-6^2\right]$

$=n\left(n^3-7 n-6\right)\left(n^3-7 n+6\right) \text {. }$

Ta lại có $\quad \mathrm{n}^3-7 \mathrm{n}-6=(\mathrm{n}+1)(\mathrm{n}+2)(\mathrm{n}-3)$,

$n^3-7 n+6=(n-1)(n-2)(n+3) \text {. }$

Do đó $\mathrm{A}=(\mathrm{n}-3)(\mathrm{n}-2)(\mathrm{n}-1) \mathrm{n}(\mathrm{n}+1)(\mathrm{n}+2)(\mathrm{n}+3)$.

Đây là tích của bảy số nguyên liên tiếp. Trong bảy số nguyên liên tiếp :

  • Tồn tại một bội số của 5 (nên $\mathrm{A}$ chia hết cho 5) ;

  • Tồn tại một bội số của 7 (nên $\mathrm{A}$ chia hết cho 7) ;

  • Tồn tại hai bội số của 3 (nên A chia hết cho 9) ;

  • Tồn tại ba bội số của 2, trong đó cọ́ một bội số của 4 (nên $\mathrm{A}$ chia hết cho 16).

$\mathrm{A}$ chia hết cho các số $5,7,9,16$ đôi một nguyên tố cùng nhau nên $\mathrm{A}$ chia hết cho $5.7 .9 .16=5040$.

Chú ý : Khi chứng minh $\mathrm{A}(\mathrm{n})$ chia hết cho $\mathrm{m}$, ta có thể xét mọi trường hợp về số dư khi chia n cho m.

Ví dụ 2. Chứng minh rằng với mọi số nguyên a thì

a) $\mathrm{a}^2-\mathrm{a}$ chia hết cho 2 ;

b) $\mathrm{a}^3-\mathrm{a}$ chia hết cho 3 ;

c) $\mathrm{a}^5-$ a chia hết cho 5 ;

d) $\mathrm{a}^7-\mathrm{a}$ chia chết cho 7 .

Giải :

a) $a^2-a=a(a-1)$, chia hết cho 2 .

b) $\mathrm{a}^3-\mathrm{a}=\mathrm{a}\left(\mathrm{a}^2-1\right)=(\mathrm{a}-1) \mathrm{a}(\mathrm{a}+1)$, tích này chia hết cho 3 vì tồn tại một bội của 3 .

c) Cách 1. $\mathrm{A}=\mathrm{a}^5-\mathrm{a}=\mathrm{a}\left(\mathrm{a}^2+1\right)\left(\mathrm{a}^2-1\right)$.

Nếu a $=5 \mathrm{k}(\mathrm{k} \in \mathbb{Z})$ thì a chia hết cho 5 .

Nếu $\mathrm{a}=5 \mathrm{k} \pm 1(\mathrm{k} \in \mathbf{Z})$ thì $\mathrm{a}^2-1$ chia hết cho 5 .

Nếu $\mathrm{a}=5 \mathrm{k} \pm 2(\mathrm{k} \in \mathrm{Z})$ thì $\mathrm{a}^2+1$ chia hết cho 5 .

Trường hợp nào cũng có một thừa số của $\mathrm{A}$ chia hết cho $5 .$

Cách 2. Phân tích a $a^5$ – a thành một tổng của hai số hạng chia hết cho 5 :

Một số hạng là tích của năm số nguyên liên tiếp, một số hạng chứa thừa số 5 .

$a^5-a =a\left(a^2-1\right)\left(a^2+1\right) $

$=a\left(a^2-1\right)\left(a^2-4+5\right) $

$=a\left(a^2-1\right)\left(a^2-4\right)+5 a\left(a^2-1\right) $

$=(a-2)(a-1) a(a+1)(a+2)+5 a\left(a^2-1\right)$

Số hạng thứ nhất là tích của năm số nguyên liên tiếp nên chia hết cho 5 , số hạng thứ hai cũng chia hết cho 5 . Do đó $\mathrm{a}^5-\mathrm{a}$ chia hết cho 5 .

Cách 3. Giải tương tự như cách 2 : Xét hiệu giữa a ${ }^5-$ a và tích năm số nguyên liên tiếp $(\mathrm{a}-2)(\mathrm{a}-1) \mathrm{a}(\mathrm{a}+1)(\mathrm{a}+2)$, được $5 \mathrm{a}\left(\mathrm{a}^2-1\right)$. Do đó $\mathrm{a}^5-\mathrm{a}$ chia hết cho 5 .

Ví dụ 3.
a) Chứng minh rằng một số chính phương chia cho 3 chỉ có thể có số dư bằng 0 hoặc 1 .

b) Chứng minh rằng một số chính phương chia cho 4 chỉ có thể có số dư bằng 0 hoặc 1 .

c) Các số sau có là số chính phương không ?

$\mathrm{M}=1992^2+1993^2+1994^2 $

$\mathrm{~N}=1992^2+1993^2+1994^2+1995^2 $

$\mathrm{P}=1+9^{100}+94^{100}+1994^{100}$

d) Trong dãy sau có tồn tại số nào là số chính phương không ?

$11,111,1111,11111, \ldots$

Giải : Gọi A là số chính phương $\mathrm{A}=\mathrm{n}^2(\mathrm{n} \in \mathrm{N})$.

a) Xét các trường hợp :

$\mathrm{n}=3 \mathrm{k}(\mathrm{k} \in \mathbf{N}) \Rightarrow \mathrm{A}=9 \mathrm{k}^2$, chia hết cho 3 .

$\mathrm{n}=3 \mathrm{k} \pm 1(\mathrm{k} \in \mathbf{N}) \Rightarrow \mathrm{A}=9 \mathrm{k}^2 \pm 6 \mathrm{k}+1$, chia cho 3 dư 1 .

Vậy số chính phương chia cho 3 chỉ có thể có số dư bằng 0 hoặc 1 .

b) Xét các trường hợp :

$\mathrm{n}=2 \mathrm{k}(\mathrm{k} \in \mathrm{N}) \Rightarrow \mathrm{A}=4 \mathrm{k}^2$, chia hết cho $4 .$

$\mathrm{n}=2 \mathrm{k}+1(\mathrm{k} \in \mathbf{N}) \Rightarrow \mathrm{A}=4 \mathrm{k}^2+4 \mathrm{k}+1=4 \mathrm{k}(\mathrm{k}+1)+1$, chia cho 4 dư 1

(chia cho 8 cũng dư 1).

Vậy số chính phương chia cho 4 chỉ có thể có số dư bằng 0 hoặc $1 .$

Chú ý : Từ bài toán trên ta thấy :

  • Số chính phương chẵn thì chia hết cho $4 .$

  • Số chính phương lẻ thì chia cho 4 dư 1 (hơn nữa, chia cho 8 cũng dư 1).

c) Các số $1993^2, 1994^2$ là số chính phương không chia hết cho 3 nên chia cho 3 dư 1 , còn $1992^2$ chịa hết cho 3 .Số M là số chia cho 3 dư 2 , không là số chính phương.

Các số $1992^2, 1994^2$ là số chính phương chẵn nên chia hết cho 4. Các số $1993^2, 1995^2$ là số chính phương lẻ nên chia cho 4 dư 1. Số $\mathrm{N}$ là số chia cho 4 . dư 2, không là số chính phương.

Các số $94^{100}, 1994^{100}$ là số chính phương chẵn nên chia hết cho 4 . Còn $9^{100}$ là số chính phưong lẻ nên chia cho 4 đư 1 . Số P là số chia cho 4 dư 2 , không là số chính phương.

d) Mọi số của dãy đều tận cùng bởi 11 nên là số chia cho 4 dư 3. Mặt khác, số chính phương lẻ thì chia cho 4 dư $1 .$

Vậy không có số nào của dãy là số chính phương.

Chú ý : Khi chứng minh về tính chia hết của các luỹ thừa, ta còn sử dụng đến các hằng đẳng thức 8,9 ở $\S 2$ và công thức Niu-tơn sau đây :

$(a+b)^n=a^n+c_1 a^{n-1} b+c_2 a^{n-2} b^2+\ldots+c_{n-1} a b^{n-1}+b^n .$

Trong công thức trên, vế phải là một đa thức có $\mathrm{n}+1$ hạng tử, bậc của mỗi hạng tử đối với tập hợp các biến $\mathrm{a}, \mathrm{b}$ là $\mathrm{n}$ (phần biến số của mỗi hạng tử có dạng $\mathrm{a}^{\mathrm{i}} \mathrm{b}^{\mathrm{k}}$, trong đó $\mathrm{i}+\mathrm{k}=\mathrm{n}$ với $0 \leq \mathrm{i} \leq \mathrm{n}, 0 \leq \mathrm{k} \leq \mathrm{n}$ ). Các hệ số $c_1$, $c_2$, $\ldots$, $c_n-1$ được xác định bởi bảng tam giác Pa-xcan (h.1) :

$\quad\quad\quad\quad\quad\quad\quad\quad\quad\quad\quad\quad Hình 1\quad\quad\quad\quad\quad\quad\quad\quad\quad\quad\quad\quad Hình 2$

Trong hình 1 , các số dọc theo một cạnh góc vuông bằng 1 , các số dọc theo cạnh huyền bằng 1. Cộng mỗi số với số liền sau bên phải thì được số đứng ở hàng dưới của số liền sau ấy, chẳng hạn ở hình $2 .$

Áp dụng các hằng đẳng thức đó vào tính chia hết, ta có với mọi số nguyên a, b và số tự nhiên $\mathrm{n}$ :

$a^n-b^n$ chia hết cho $a-b(a \neq b)$;

$a^{2 n+1}+b^{2 n+1}$ chia hết cho $a+b(a \neq-b)$;

$(a+b)^n=B S a+b^n(B S$ a là bội của $a)$.

Đặc biệt nên lưu ý đến :

$(a+1)^n=B S a+1 $

$(a-1)^{2 n}=B S a+1 $

$(a-1)^{2 n+1}=B S a-1$

Ví dụ 4. Chứng minh rằng với mọi số tự nhiên $\mathrm{n}$, biểu thức $16^{\mathrm{n}}-1$ chia hết cho 17 khi và chỉ khi $\mathrm{n}$ là số chẵn.

Giải :

Cách 1. Nếu n chã̃n $(\mathrm{n}=2 \mathrm{k}, \mathrm{k} \in \mathrm{N})$ thì $\mathrm{A}=16^{2 \mathrm{k}}-1=\left(16^2\right)^{\mathrm{k}}-1$. chia hết cho $16^2-1$ theo hằng đẳng thức 8 , mà $16^2-1=255$, chia hết cho 17 . Vậy $\mathrm{A}$ chia hết cho 17 .

Nếu $\mathrm{n}$ lẻ thì $\mathrm{A}=16^{\mathrm{n}}+1-2$, mà $16^{\mathrm{n}}+1$ chia hết cho 17 theo hằng đẳng thức 9 , nên $\mathrm{A}$ không chia hết cho $17 .$

Vậy $\mathrm{A}$ chia hết cho $17 \Leftrightarrow \mathrm{n}$ chẵn.

Cách 2. $\mathrm{A}=16^{\mathrm{n}}-1=(17-1)^{\mathrm{n}}-1=\mathrm{BS} 17+(-1)^{\mathrm{n}}-1$ (theo công thức Niu-tơn).

Nếu n chã̃n thì $\mathrm{A}=\mathrm{BS} 17+1-1=\mathrm{BS} 17$.

Nếu n lẻ thì $\mathrm{A}=\mathrm{BS} 17-1-1$, không chia hết cho 17 .

Chú ý : Người ta còn dùng phương pháp phản chứng, nguyên lí Đi-rích-lê để chứng minh quan hệ chia hết.

Ví dụ 5. Chứng minh rằng tồn tại một bội của 2003 có dạng

$\quad\quad\quad\quad\quad\quad\quad\quad2004\quad2004 \ldots 2004 .$

Giải : Xét 2004 số :

$a_1=2004 $

$a_2=2004\quad2004$

$\mathrm{a}_{2004}=2004\quad2004 \ldots 2004$ (nhóm 2004 có mặt 2004 lần).

Theo nguyên lí Đi-rích-lế, tồn tại hai số có cùng số dư khi phép chia cho $2003 .$

Gọi hai số đó là $a_m$ và $a_n(1 \leq \mathrm{n}<\mathrm{m} \leq 2004)$ thì $a_m-a_n\vdots 2003$. Ta có

$a_m-a_n=2004 \ldots 20040000 \ldots 0000=\underbrace{2004 \ldots 2004}_{m-n \text { nhóm 2004 }}\text{.} 10^{4 n} .$

Do $10^{4 \mathrm{n}}$ và 2003 nguyên tố cùng nhau nên $\underbrace{2004 \ldots 2004}_{\mathrm{m}-\mathrm{n} \text { nhóm } 2004}$ chia hết cho $2003 .$

 

TÌM SỐ DƯ

VÍ dụ 6. Tìm số dư khi chia $2^{100}$ :

a) Cho 9 ;

b) Cho 25 ;

c) Cho 125 .

Giải : a) Luỹ thừa của 2 sát với một bội số của 9 là $2^3=8=9-1$.

Ta có $2^{100}=2\left(2^3\right)^{33}=2(9-1)^{33}=2(\mathrm{BS}\quad 9-1)=\mathrm{BS}\quad 9-2=\mathrm{BS}\quad 9+7$.

Số dư khi chia $2^{100}$ cho 9 là 7 .

b) Luỹ thừa của 2 sát với một bội số của 25 là $2^{10}=1024=\mathrm{BS}\quad 25-1$.

Ta có $\quad 2^{100}=\left(2^{10}\right)^{10}=(\mathrm{BS}\quad 25-1)^{10}=\mathrm{BS}\quad 25+1$.

c) Dùng công thức Niu-tơn :

$2^{100}=(5-1)^{50}=5^{50}-50.5^{49}+\ldots+\frac{50.49}{2} \cdot 5^2-50: 5+1 .$

Không kể phần hệ số của khai triển Niu-tơn thì 48 số hạng đầu đã chứa luỹ thừa của 5 với số mũ lớn hơn hoặc bằng 3 nên chia hết cho 125 . Hai số hạng tiếp theo cũng chia hết cho 125 , số hạng cuối cùng là 1 . Vậy $2^{100}=\mathrm{BS}\quad 125+1$.

Chú ý : Tổng quát hơn, ta chứng minh được rằng nếu một số tự nhiên $\mathrm{n}$ không chia hết cho 5 thì chia $\mathrm{n}^{100}$ cho 125 ta được số dư là 1 .

Thật vậy, $n$ có dạng $5 \mathrm{k} \pm 1$ hoặc $5 \mathrm{k} \pm 2$. Ta có

$(5 \mathrm{k} \pm 1)^{100}=(5 \mathrm{k})^{100} \pm \ldots+\frac{100.99}{2}(5 \mathrm{k})^2 \pm 100.5 \mathrm{k}+1=\mathrm{BS}\quad 125+1$

$(5 \mathrm{k} \pm 2)^{100} =(5 \mathrm{k})^{100} \pm \ldots+\frac{100 \cdot 99}{2}(5 \mathrm{k})^2 \cdot 2^{98} \pm 100 \cdot 5 \mathrm{k} \cdot 2^{99}+2^{100} $

$=\mathrm{BS}\quad 125+2^{100}$

Ta lại có $2^{100}=\mathrm{BS}\quad 125+1$ (câu c). Do đó $(5 \mathrm{k} \pm 2)^{100}=\mathrm{BS}\quad 125+1$.

Ví dụ 7. Tìm ba chữ số tận cùng của $2^{100}$ khi viết trong hệ thập phân.

Giải : Tìm ba chữ số tận cùng của $2^{100}$ là tìm số dư khi chia $2^{100}$ cho 1000 . Trước hết tìm số dư khi chia $2^{100}$ cho 125 . Theo ví dụ 43 ta có $2^{100}=\mathrm{BS} 125+1$, mà $2^{100}$ là số chẵn, nên ba chữ số tân cùng của nó chỉ có thể là 126, 376, 626 hoặc 876 .

Hiển nhiên $2^{100}$ chia hết cho 8 nên ba chữ số tận cùng của nó phải chia hết cho 8. Trong bốn số trên chỉ có 376 thoả mãn điều kiện này.

Vậy ba chữ số tận cùng của $2^{100}$ là 376 .

Chú ý : Bạn đọc tự chứng minh rằng nếu n là số chẵn không chia hết cho 5 thì ba chữ số tận cùng của $\mathrm{n}^{100}$ là 376 .

Ví dụ 8. Tìm bốn chữ số tận cùng của $5^{1994}$ khi viết trong hệ thập phân.

Giải :

Cách 1. $5^4=625$. Ta thấy số tận cùng bằng 0625 nâng lên luỹ thừa nguyên dương bất kì vẫn tận cùng bằng 0625 (chỉ cần kiểm tra : … $0625 \times \ldots 0625=\ldots 0625$ ). Do đó :

$5^{1994}=5^{4 \mathrm{k}+2}=25\left(5^4\right)^{\mathrm{k}}=25(0625)^{\mathrm{k}}=25(\ldots 0625)=\ldots 5625 .$

Cách 2. Tìm số dư khi chia $5^{1994}$ cho $10000=2^4 \cdot 5^4$.

Nhận xét $: 5^{4 \mathrm{k}}-1$ chia hết cho $5^4-1=\left(5^2+1\right)\left(5^2-1\right)$ nên chia hết cho 16 . Ta có $: 5^{1994}=5^6\left(5^{1988}-1\right)+5^6$.

Do $5^6$ chia hết cho $5^4$, còn $5^{1988}-1$ chia hết cho 16 (theo nhận xét trên) nên $5^6\left(5^{1988}-1\right)$ chia hết cho 10000 . Tính $5^6$, ta được 15625 . Vậy bốn chữ số tận cùng của $5^{1994}$ là 5625 .

Chú ý: Nếu viết $5^{1994}=5^2\left(5^{1992}-1\right)+5^2$ thì ta có $5^{1992}-1$ chia hết cho 16 , nhưng $5^2$ không chia hết cho $5^4$.

Như thế trong bài toán này, ta cần viết $5^{1994}$ dưới dạng $5^{\mathrm{n}}\left(5^{1994-\mathrm{n}}-1\right)+5^{\mathrm{n}}$ sao cho $n^{\prime} \geq 4$ và $1994-n$ chia hết cho 4 .

TÌM ĐIỀU KIỆN ĐỂ CHIA HẾT

 

Ví dụ 9. Tìm số nguyên $\mathrm{n}$ để giá trị của biểu thức $\mathrm{A}$ chia hết cho giá trị của biểu thức $\mathrm{B}$ :

$A=n^3+2 n^2-3 n+2, \quad B=n^2-n .$

Giải : Đặt tính chia

Muốn chia hết, ta phải có 2 chia hết cho $\mathrm{n}(\mathrm{n}-1)$, do đó 2 chia hết cho $\mathrm{n}$. Ta có :

Đáp số : $\mathrm{n}=-1 ; \mathrm{n}=2$.

Chú ý:

a) Không thể nói đa thức $\mathrm{A}$ chia hết cho đa thức $\mathrm{B}$. Ỏ đây chỉ tồn tại những giá trị nguyên của n để giá trị của biểu thức $\mathrm{A}$ chia hết cho giá trị của biểu thức $\mathrm{B}$.

b) Có thể thay việc đặt phép chia bằng cách biến đổi :

$n^3+2 n^2-3 n+2=n\left(n^2-n\right)+3\left(n^2-n\right)+2 .$

Ví dụ 10. Tìm số nguyên dương $\mathrm{n}$ để $\mathrm{n}^5+1$ chia hết cho $\mathrm{n}^3+1$.

Giải : Biến đổi

$\mathrm{n}^5+1 \vdots \mathrm{n}^3+1 \Leftrightarrow \mathrm{n}^2\left(\mathrm{n}^3+1\right)-\left(\mathrm{n}^2-1\right) \vdots \mathrm{n}^3+1 $

$ \Leftrightarrow(\mathrm{n}+1)(\mathrm{n}-1) \vdots(\mathrm{n}+1)\left(\mathrm{n}^2-\mathrm{n}+1\right) $

$ \Leftrightarrow \mathrm{n}-1 \vdots \mathrm{n}^2-\mathrm{n}+1(\mathrm{vì} \mathrm{n}+1 \neq 0)$

Nếu $\mathrm{n}=1$ thì ta được 0 chia hết cho 1 .

Nếu $\mathrm{n}>1$ thì $\mathrm{n}-1<\mathrm{n}(\mathrm{n}-1)+1=\mathrm{n}^2-\mathrm{n}+1$, do đó $\mathrm{n}-1$ không thể chia hết cho $\mathrm{n}^2-\mathrm{n}+1$

Vậy giá trị duy nhất của n tìm được là 1 .

Ví dụ 11. Tìm số nguyên $\mathrm{n}$ để $\mathrm{n}^5+1$ chia hết cho $\mathrm{n}^3+1$.

Giải : Cũng biến đổi như ở ví dụ 47 , ta có $\mathrm{n}-1 \vdots \mathrm{n}^2-\mathrm{n}+1$

$\mathrm{n}-1 \vdots \mathrm{n}^2-\mathrm{n}+1 \Rightarrow \mathrm{n}(\mathrm{n}-1) \vdots \mathrm{n}^2-\mathrm{n}+1 \Rightarrow \mathrm{n}^2-\mathrm{n} \vdots \mathrm{n}^2-\mathrm{n}+1$

$\Rightarrow\left(n^2-n+1\right)-1 \vdots n^2-n+1 \Rightarrow 1 \vdots n^2-n+1$

Có hai trường hợp :

$\mathrm{n}^2-\mathrm{n}+1=1 \Leftrightarrow \mathrm{n}(\mathrm{n}-1)=0 \Leftrightarrow \mathrm{n}=0 ; \mathrm{n}=1$. Các giá trị này thoả mãn đề bài.

$\mathrm{n}^2-\mathrm{n}+1=-1 \Leftrightarrow \mathrm{n}^2-\mathrm{n}+2=0$, vô nghiệm.

Vậy $n=0, n=1$ là hai số phải tìm.

Chú ý: Từ $\mathrm{n}-1 \vdots \mathrm{n}^2-\mathrm{n}+1$ suy ra $\mathrm{n}(\mathrm{n}-1) \vdots \mathrm{n}^2-\mathrm{n}+1$ là phép kéo theo chứ không là phép biến đổi tương đương. Do đó sau khi tìm được $\mathrm{n}=0, \mathrm{n}=1$, ta phải thử lại.

Ví dụ 12. Tîm số tự nhiên $n$ sao cho $2^n-1$ chia hết cho 7 .

Giải : Nếu $\mathrm{n}=3 \mathrm{k} \cdot(\mathrm{k} \in \mathbf{N})$ thì $2^{\mathrm{n}}-1=2^{3 \mathrm{k}}-1=8^{\mathrm{k}}-1$ chia hết cho 7 .

Nếu $\mathrm{n}=3 \mathrm{k}+1(\mathrm{k} \in \mathrm{N})$ thì $2^{\mathrm{n}}-1=2^{3 \mathrm{k}+1}-1=2\left(2^{3 \mathrm{k}}-1\right)+1=\mathrm{BS} 7+1$.

Nếu $\mathrm{n}=3 \mathrm{k}+2(\mathrm{k} \in \mathbf{N})$ thì $2^{\mathrm{n}}-1=2^{3 \mathrm{k}+2}-1=4\left(2^{3 \mathrm{k}}-1\right)+3=\mathrm{BS} 7+3$.

Vậy $2^{\mathrm{n}}-1$ chia hết cho $7 \Leftrightarrow \mathrm{n}=3 \mathrm{k}(\mathrm{k} \in \mathrm{N})$.

 

BÀI TẬP

 

$1.$ Chứng minh rằng với mọi số nguyên $\mathrm{n}$, ta có :

a) $\mathrm{n}^3+3 \mathrm{n}^2+2 \mathrm{n}$ chia hết cho 6 ;

b) $\left(\mathrm{n}^2+\mathrm{n}-1\right)^2-1$ chia hết cho 24 .

$2.$ Chứng minh rằng :

a) $\mathrm{n}^3+6 \mathrm{n}^2+8 \mathrm{n}$ chia hết cho 48 với mọi số chẵn $\mathrm{n}$;

b) $n^4-10 n^2+9$ chia hết cho 384 với mọi số lẻ $n$.

$3.$ Chứng minh rằng $n^6+n^4-2 n^2$ chia hết cho 72 với mọi số nguyên $n$.

$4.$ Chứngminh rằng $3^{2 \mathrm{n}}-9$ chia hết cho 72 với mọi số nguyên dương $\mathrm{n}$. 190(3). Chứng minh rằng với mọi số tự nhiên a và $\mathrm{n}$ :

a) $7^{\mathrm{n}}$ và $7^{\mathrm{n}+4}$ có hai chữ số tận cùng như nhau ;

b) a và a ${ }^5$ có chữ số tận cùng như nhau ;

c) $\mathrm{a}^{\mathrm{n}}$ và $\mathrm{a}^{\mathrm{n}+4}$ có chữ số tận cùng như nhau $(\mathrm{n} \geq 1)$.

$5.$ Tìm điều kiện của số tự nhiên $\mathrm{a}$ để a $\mathrm{a}^2+3 \mathrm{a}+2$ chia hết cho 6 .

$6.$ a) Cho a là số nguyên tố lớn hơn 3. Chứng minh rằng $\mathrm{a}^2-1$ chia hết cho 24 .

b) Chứng minh rằng nếu $a$ và $\mathrm{b}$ là các số nguyên tố lớn hơn 3 thì $\mathrm{a}^2-\mathrm{b}^2$ chia hết cho 24 .

c) Tìm điều kiện của số tự nhiên a để $a^4-1$ chia hết cho 240 .

$7.$ Tìm ba số nguyên tố liên tiếp $a, b, c$ sao cho $a^2+b^2+c^2$ cũng là số nguyên tố.

$8.$ Cho bốn số nguyên dương $\mathrm{a}, \mathrm{b}, \mathrm{c}, \mathrm{d}$ thoả mãn $\mathrm{a}^2+\mathrm{b}^2=\mathrm{c}^2+\mathrm{d}^2$. Chứng minh rằng $\mathrm{a}+\mathrm{b}+\mathrm{c}+\mathrm{d}$ là hợp số.

$9.$ Cho bốn số nguyên dương $\mathrm{a}, \mathrm{b}, \mathrm{c}, \mathrm{d}$ thoả mãn $\mathrm{ab}=\mathrm{cd}$. Chứng minh rằng $a^5+b^5+c^5+d^5$ là hợp số.

$10.$ Cho các số nguyên a, b, c. Chứng minh rằng :

a) Nếu $a+b+c$ chia hết cho 6 thì $a^3+b^3+c^3$ chia hết cho 6 .

b) Nếu $\mathrm{a}+\mathrm{b}+\mathrm{c}$ chia hết cho 30 thì $\mathrm{a}^5+\mathrm{b}^5+\mathrm{c}^5$ chia hết cho 30 .

$11.$ Cho các số nguyên $\mathrm{a}, \mathrm{b}, \mathrm{c}$ thoả mãn $\mathrm{a}+\mathrm{b}+\mathrm{c}=0$. Chứng minh rằng :

a) $a^3+b^3+c^3$ chia hết cho $3 a b c$;

b) $a^5+b^5+c^5$ chia hết cho $5 a b c$.

$12.$ a) Viết số 1998 thành tổng của ba số tự nhiên tuỳ ý. Chứng minh rằng tổng các lập phương của ba số tự nhiên đó chia hết cho 6 .

b)* Viết số $1995^{1995}$ thành tổng của nhiều số tự nhiên. Tổng các lập phương của các số tự nhiên đó chia cho 6 dư bao nhiêu?

$13.$ Chứng minh rằng với mọi số nguyên $\mathrm{a}$ và $\mathrm{b}$ :

a) $\mathrm{a}^3 \mathrm{~b}-\mathrm{ab}{ }^3$ chia hết cho 6 ;

b) $\mathrm{a}^5 \mathrm{~b}-\mathrm{ab}{ }^5$ chia hết cho 30 .

$14.$ Chứng minh rằng mọi số tự nhiên đều viết được dưới dạng $b^3+6 c$ trong đó b và c là các số nguyên.

$15*$. Chứng minh rằng nếu các số tự nhiên $\mathrm{a}, \mathrm{b}, \mathrm{c}$ thoả mãn điều kiện $a^2+b^2=c^2$ thì abc chia hết cho 60 .

$16.$ Chứng minh rằng tổng các lập phương của ba số nguyên liên tiếp thì chia hết cho $9 .$

$17.$ Chứng minh rằng nếu tổng các lập phương của ba số nguyên chia hết cho 9 thì tồn tạii một trong ba số đó là bội số của 3 .

$18.$ Cho dãy số $7,13,25, \ldots, 3 \mathrm{n}(\mathrm{n}-1)+7(\mathrm{n} \in \mathrm{N})$. Chứng minh rằng :

a) Trong năm số hạng liên tiếp của dạ̃y, bao giờ cũng tồn tại một bội số của 25 .

b) Không có số hạng nào của dãy là lập phương của một số nguyên.

$19.$ a) Chứng minh rằng nếu số tự nhiên a không chia hết cho 7 thì $\mathrm{a}^6-1$ chia hết cho 7 .

b) Chứng minh rằng nếu n là lập phương của một số tự nhiên thì $(n-1) n(n+1)$ chia hết cho 504 .

$20.$ Chứng minh rằng $\mathrm{A}$ chia hết cho $\mathrm{B}$ với :

a) $A=1^3+2^3+3^3+\ldots+99^3+100^3$,

$\mathrm{B}=1+2+3+\ldots+99+100$

b) $A=1^3+2^3+3^3+\ldots+98^3+99^3$,

$\mathrm{B}=1+2+3+\ldots+98+99$

$21.$ Các số sau có là số chính phương không ?

a) $\mathrm{A}=22 \ldots 24$ (có 50 chữ số 2 ) ;

b) $\mathrm{B}=44 \ldots 4$ (có 100 chữ số 4);

c) $\mathrm{A}=1994^7+7$;

d)* $B=144$… 4 (có 99 chữ số 4).

$22.$ Có thể dùng cả năm chữ số $2,3,4,5,6$ lập thành số chính phương có năm chữ số được không ?

$23.$ Chứng minh rằng tổng của hai số chính phương lẻ không là số chính phương.

$24.$ Chứng minh rằng mọi số lẻ đều viết được dưới dạng hiệu của hai số chính phương.

$25*.$ Chứng minh rằng :

a) $A=1^2+2^2+3^2+4^2+\ldots+100^2$ không là số chính phương ;

b) $\mathrm{B}=1^2+2^2+3^2+4^2+\ldots+56^2$ không là số chính phương ;

c) $\mathrm{C}=1+3+5+7+\ldots+\mathrm{n}$ là số chính phương ( $\mathrm{n}$ lẻ).

$26.$ Chứng minh rằng :

a) Một số chî́nh phương tận cùng bằng 9 thì chữ số hàng chục là chữ số chẵn

b) Một số chính phương lẻ thì chữ số hàng chục là chữ số chẵn.

c) Một số chính phương tận cùng bằng 6 thì chữ số hàng chục là chữ số lẻ.

d) Một số chính phương tận cùng bằng 5 thì chữ số hàng chục bằng 2 và chữ số hàng trăm là chữ số chẵn.

$27.$ a) Một số chính phương có chữ số hàng chục bằng 5. Tìm chữ số hàng đơn vị.

b) Một số chính phương có chữ số hàng chục là chữ số lẻ. Tìm chữ số hàng đơn vị.

c) Có bao nhiêu số tự nhiên $\mathrm{n}$ từ 1 đến 100 mà chữ số hàng chục của $\mathrm{n}^2$ là chữ số lẻ ?

$28.$ Chứng minh rằng :

a) Tích của hai số nguyên dương liên tiếp không là số chính phương.

b)* Tích của ba số nguyên dương liên tiếp không là số chính phương.

c)* Tích của bốn số nguyên dương liên tiếp không là số chính phương.

$29.$ Cho hai số tự nhiên a và $\mathrm{b}$, trong đó $\mathrm{a}=\mathrm{b}-2$.

Chứng minh rằng $\mathrm{b}^3-\mathrm{a}^3$ viết được dưới dạng tổng của ba số chính phương.

$30.$ Tìm số nguyên dương $\mathrm{n}$ để biểu thức sau là số chính phương :

a) $n^2-n+2$;

b) $n^4-n+2$

c) $n^3-n+2$;

d) ${ }^* n^5-n+2$.

$31.$ Tìm số nguyên tố $\mathrm{p}$ để $4 \mathrm{p}+1$ là số chính phương.

$32*.$ Chứng minh rằng nếu $\mathrm{n}+1$ và $2 \mathrm{n}+1(\mathrm{n} \in \mathrm{N})$ đều là số chính phương thì $\mathrm{n}$ chia hết cho 24 .

$33*.$ Chứng minh rằng nếu $2 n+1$ và $3 n+1(n \in N)$ đều là số chính phương thì n chia hết cho $40 .$

$34.$ Tìm số nguyên tố $\mathrm{p}$ để :

a) $2 \mathrm{p}^2+1$ cũng là số nguyên tố ;

b) $4 \mathrm{p}^2+1$ và $6 \mathrm{p}^2+1$ cũng là những số nguyên tố.

$35.$ Tìm số tự nhiên $\mathrm{n}$ để giá trị của biểu thức là số nguyên tố :

a) $12 n^2-5 n-25$

b) $8 n^2+10 n+3$;

c) $\frac{n^2+3 n}{4}$.

$36.$ Chứng minh rằng với mọi số nguyên $\mathrm{n}$ :

a) $n^2+7 n+22$ không chia hết cho 9 ;

b) $n^2-5 n-49$ không chia hết cho 169 .

$37.$ Các số tự nhiên $\mathrm{n}$ và $\mathrm{n}^2$ có tổng các chữ số bằng nhau. Tìm số dư của $\mathrm{n}$ khi chia cho $9 .$

$38*.$ a) Cho chín số tự nhiên từ 1 đến 9 xếp theo thứ tự tuỳ ý. Lấy số thứ nhất trừ 1, lấy số thứ hai trừ 2 , lấy số thứ ba trừ $3, \ldots$, lấy số thứ chín trừ 9 . Chứng minh rằng tích của chín số mới lập được là một số chẵn.

b) Cho hai dãy số $a_1, a_2, a_3, \ldots, a_9$ và $b_1, b_2, b_3, \ldots, b_9$, trong đó $a_1, a_2, \ldots, a_9$ là các số nguyên và $b_1, b_2, \ldots, b_9$ cũng là chín số nguyên trên nhưng lấy theo thứ tự khác. Chứng minh rằng tích $\left(\mathrm{a}_1-\mathrm{b}_1\right)\left(\mathrm{a}_2-\mathrm{b}_2\right) \ldots\left(\mathrm{a}_9-\mathrm{b}_9\right)$ là số chẵn.

$39.$ Tìm số nguyên $\mathrm{n}$ sao cho :

a) $n^2+2 n-4$ chia hết cho 11 ;

b) $2 n^3+n^2+7 n+1$ chia hết cho $2 n-1$;

c) $\mathrm{n}^3-2$ chia hết cho $\mathrm{n}-2$;

d) $n^3-3 n^2-3 n-1$ chia hết cho $n^2+n+1$;

e) $n^4-2 n^3+2 n^2-2 n+1$ chia hết cho $n^4-1$;

g) ${ }^* n^3-n^2+2 n+7$ chia hết cho $n^2+1$.

$40.$ Đố vui : Năm sinh của hai bạn

Một ngày của thập kỉ cuối cùng của thế kỉ XX, một người khách đến thăm trường gặp hai học sinh. Người khách hỏi :

  • Có lẽ hai em bằng tuổi nhau ?

Bạn Mai trả lời :

  • Không; em hơn bạn em một tuổi. Nhưng tổng các chữ số của năm sinh mỗi chúng em đều là số chẵn.

  • Vậy thì các em sinh năm 1979 và 1980, đúng không ?

Người khách đã suy luận thế nào?

$41.$ Tìm số nguyên dương $\mathrm{n}$ để $2^{\mathrm{n}}$ là số nằm giữa hai số nguyên tố sinh đôi ${ }^{(*)}$ (hai số nguyên tố gọi là sinh đôi nếu chúng hơn kém nhau 2 đơn vị).

$42*.$ Cho các số nguyên $\mathrm{a}, \mathrm{b}, \mathrm{c}, \mathrm{d}, \mathrm{e}, \mathrm{g}$ thoả mãn $\mathrm{a}^2+\mathrm{b}^2+\mathrm{c}^2+\mathrm{d}^2+\mathrm{e}^2=\mathrm{g}^2$.

Chứng minh rằng tích abcdeg là số chẵn.

$43.$ Chứng minh rằng với mọi số nguyên $\mathrm{a}, \mathrm{b}, \mathrm{c}, \mathrm{d}$, tích

$(\mathrm{a}-\mathrm{b})(\mathrm{a}-\mathrm{c})(\mathrm{a}-\mathrm{d})(\mathrm{b}-\mathrm{c})(\mathrm{b}-\mathrm{d})(\mathrm{c}-\mathrm{d}) \text { chia hết cho } 12 \text {. }$

$44*$. Chứng minh rằng có thể có đến 33 số nguyên dương khác nhau, không quá 50, trong đó không tồn tại hai số nào mà một số gấp đôi số còn lại.

$45.$ Chứng minh rằng tồn tại vô số bội của 2003 mà trong biểu diễn thập phân của chúng không có các chữ số $0,1,2,3$.

$46.$ Chứng minh rằng tồn tại số tự nhiên $\mathrm{k}$ sao cho $2003^{\mathrm{k}}$ – 1 chia hết cho 51 .

Các bài toán sủ dụng các hằng đẳng thúc 8,9 và công thức Niu-tơn.

$47.$ Chứng minh rằng $2^{51}-1$ chia hết cho 7 .

$48.$ Chứng minh rằng $2^{70}+3^{70}$ chia hết cho $13 .$

$49.$ Chứng minh rằng $17^{19}+19^{17}$ chia hết cho 18 .

$50.$ Chứng minh rằng $36^{63}-1$ chia hết cho 7 , nhưng không chia hết cho 37 .

$51.$ Chứng minh rằng các số sau là hợp số :

a) $4^{20}-1$;

b) 1000001 .

c) $2^{50}+1$.

$52.$ Chứng minh rằng $1 \cdot 4+2 \cdot 4^2+3 \cdot 4^3+4 \cdot 4^4+5 \cdot 4^5+6 \cdot 4^6$ chia hết cho 3 .

$53.$ Chứng minh rằng biểu thức $\mathrm{A}=31^{\mathrm{n}}-15^{\mathrm{n}}-24^{\mathrm{n}}+8^{\mathrm{n}}$ chia hết cho 112 với mọi số tự nhiên $\mathrm{n}$.

$54.$ Tìm số tự nhiên $\mathrm{n}$ để $3^{\mathrm{n}}-1$ chia hết cho 8 .

$55.$ Tìm số tự nhiên $\mathrm{n}$ để $3^{2 \mathrm{n}+3}+2^{4 \mathrm{n}+1}$ chia hết cho 25 .

$56.$ Tìm số tự nhiên $\mathrm{n}$ để $5^{\mathrm{n}}-2^{\mathrm{n}}$ chia hết cho 9 .

$57.$ Tìm số tự nhiên $\mathrm{n}$ để $5^{\mathrm{n}}-2^{\mathrm{n}}$ chia hết cho 63 .

$58.$ Tìm số tự nhiên $\mathrm{n}$ để $1^{\mathrm{n}}+2^{\mathrm{n}}+3^{\mathrm{n}}+4^{\mathrm{n}}$ chia hết cho 5

$59.$ Tìm số dư khi chia $22^{22}+55^{55}$ cho 7 .

$60.$ Tìm số dư khi chia $2^{1994}$ cho 7 .

$61.$ Tìm số dư khi chia $3^{1993}$ cho 7 .

$62.$ Tìm số dư khi chia $1992^{1993}+1994^{1995}$ cho 7 .

$63 *.$ Tìm số dư khi chia $9^{10^{11}}-5^{9^{10}}$ cho 13 .

$64*.$ Chứng minh rằng số $\mathrm{A}=2^{2^{2 \mathrm{n}+1}}+3$ là hợp số với mọi số nguyên dương $\mathrm{n}$.

$65.$ Tìm số dư khi chia các số sau cho 7 :

a) $2^{9^{1945}}$;

b) $3^{2^{1930}}$.

$66.$ Tìm số dư khi chia $\left(\mathrm{n}^3-1\right)^{111} \cdot\left(\mathrm{n}^2-1\right)^{333}$ cho $\mathrm{n}(\mathrm{n} \in \mathrm{N})$.

$67.$ Cho $\mathrm{ab}=455^{12}$. Tìm số dư trong phép chia $\mathrm{a}+\mathrm{b}$ cho $4 .$

$68.$ Tìm hai chữ số tận cùng của :

a) $3^{999}$

b) $7^{7^7}$.

$69.$ Tìm ba chữ số tận cùng của $3^{100}$.

$70 *.$ Thay các dấu * bởi các chữ số thích hợp :

$89^6=4969 * * 290961$

 

 

 

 

 

 

 

 

 

 

 

 

 

 

 

 

 

 

 

PHÂN THỨC ĐẠI SỐ – P.2

 CÁC PHÉP TÍNH VỀ PHÂN THỨC

 

Muốn cộng các phân thức, ta quy đồng mẫu thức, cộng các tử thức với nhau, giữ nguyên mẫu thức chung, rồi rút gọn phân thức vừa tìm được.

Muốn trừ đi một phân thức, ta lấy phân thức bị trừ cộng với phân thức đối của phân thức trừ.

Muốn nhân các phân thức, ta nhân các tử thức với nhau, các mẫu thức với nhau, rồi rút gọn phân thức vừa tìm được. Muốn chia cho một phân thức khác 0 , ta lấy phân thức bị chia nhân với phân thức nghịch đảo của phân thức chia.

Ví dụ 1.

Cho $\mathrm{a}+\mathrm{b}+\mathrm{c}=0$ và $\mathrm{a}, \mathrm{b}, \mathrm{c}$ đều khạ́c 0 . Rút gọn biểu thức

$A=\frac{a b}{a^2+b^2-c^2}+\frac{b c}{b^2+c^2-a^2}+\frac{c a}{c^2+a^2-b^2} \text {. }$

Giải : Từ $\mathrm{a}+\mathrm{b}+\mathrm{c}=0$ suy ra $\mathrm{a}+\mathrm{b}=-\mathrm{c}$.

Bình phương hai vế, ta được $\mathrm{a}^2+\mathrm{b}^2+2 \mathrm{ab}=\mathrm{c}^2$ nên $\mathrm{a}^2+\mathrm{b}^2-\mathrm{c}^2=-2 \mathrm{ab}$.

Tương tự, $\mathrm{b}^2+\mathrm{c}^2-\mathrm{a}^2=-2 \mathrm{bc}$ và $\mathrm{c}^2+\mathrm{a}^2-\mathrm{b}^2=-2 \mathrm{ca}$.

Do đó $\mathrm{A}=\frac{\mathrm{ab}}{-2 \mathrm{ab}}+\frac{\mathrm{bc}}{-2 \mathrm{bc}}+\frac{\mathrm{ca}}{-2 \mathrm{ca}}=-\frac{1}{2}-\frac{1}{2}-\frac{1}{2}=-\frac{3}{2}$.

Ví dụ 2. Rút gọn biểu thức

$A=\frac{1}{1-x}+\frac{1}{1+x}+\frac{2}{1+x^2}+\frac{4}{1+x^4}+\frac{8}{1+x^8} .$

Giải : Do đặc điểm của bài toán, ta không quy đồng mẫu tất cả các phân thức mà cộng lần lượt từng phân thức.

$A =\frac{2}{1-x^2}+\frac{2}{1+x^2}+\frac{4}{1+x^4}+\frac{8}{1+x^8} $

$=\frac{4}{1-x^4}+\frac{4}{1+x^4}+\frac{8}{1+x^8}=\frac{8}{1-x^8}+\frac{8}{1+x^8}=\frac{16}{1-x^{16}}$

Ví dụ 3. Rút gọn biểu thức

$\mathrm{B}=\frac{3}{(1.2)^2}+\frac{5}{(2.3)^2}+\ldots+\frac{2 n+1}{[n(n+1)]^2}$

Giải : Đương nhiên không thể quy đồng mẫu tất cả các phân thức. Ta tìm cách tách mỗi phân thức thành hiệu của hai phân thức rồi dùng phương pháp khử liên tiếp. Ta có :

$\frac{2 \mathrm{k}+1}{\mathrm{k}^2(\mathrm{k}+1)^2}=\frac{(\mathrm{k}+1)^2-\mathrm{k}^2}{\mathrm{k}^2(\mathrm{k}+1)^2}=\frac{1}{\mathrm{k}^2}-\frac{1}{(\mathrm{k}+1)^2}$

Do đó : $\quad \mathrm{B}=\frac{1}{1^2}-\frac{1}{2^2}+\frac{1}{2^2}-\frac{1}{3^2}+\ldots+\frac{1}{\mathrm{n}^2}-\frac{1}{(\mathrm{n}+1)^2}=$

$=1-\frac{1}{(n+1)^2}=\frac{n(n+2)}{(n+1)^2}$

Ví dụ 4. Xác định các ‘số a, b, c sao cho

$\frac{1}{\left(x^2+1\right)(x-1)}=\frac{a x+b}{x^2+1}+\frac{c}{x-1} \text {. }\quad\quad(1)$

Giải : Thực hiện phép cộng ở vế phải của (1) :

$\frac{(a x+b)(x-1)+c\left(x^2+1\right)}{\left(x^2+1\right)(x-1)}=\frac{a x^2-a x+b x-b+c x^2+c}{\left(x^2+1\right)(x-1)}=$

$=\frac{(a+c) x^2+(b-a) x+(c-b)}{\left(x^2+1\right)(x-1)} \text {. }$

Đồng nhất phân thức trên với phân thức $\frac{1}{\left(x^2+1\right)(x-1)}$, ta được :

$\left\{\begin{array} { l }{ \mathrm { a } + \mathrm { c } = 0 } \\ { \mathrm { b } – \mathrm { a } = 0 } \\ { \mathrm { c } – \mathrm { b } = 1 }\end{array} \Rightarrow \left\{\begin{array}{l}\mathrm{c}+\mathrm{b}=0 \\ \mathrm{c}-\mathrm{b}=1\end{array} \Rightarrow \mathrm{c}=\frac{1}{2} ; \mathrm{b}=-\frac{1}{2} .\right.\right.$

Do đó $a=-\frac{1}{2}$. Như vậy : $\frac{1}{\left(x^2+1\right)(x-1)}=\frac{-\frac{1}{2} x-\frac{1}{2}}{x^2+1}+\frac{\frac{1}{2}}{x-1}$.

Ví dụ 5. Cho $\quad A=\frac{1}{(x+y)^3}\left(\frac{1}{x^4}-\frac{1}{y^4}\right)$, $B=\frac{2}{(x+y)^4}\left(\frac{1}{x^3}-\frac{1}{y^3}\right), \quad C=\frac{2}{(x+y)^5}\left(\frac{1}{x^2}-\frac{1}{y^2}\right)$.

Thực hiện phép tính $\mathrm{A}+\mathrm{B}+\mathrm{C}$.

Giải : Ta có

$A =\frac{y^4-x^4}{x^4 y^4(x+y)^3}=\frac{\left(y^2+x^2\right)\left(y^2-x^2\right)}{x^4 y^4(x+y)^3}=\frac{\left(y^2+x^2\right)(y-x)}{x^4 y^4(x+y)^2} $

$B+C =\frac{2}{(x+y)^4}\left(\frac{1}{x^3}-\frac{1}{y^3}+\frac{1}{x+y} \cdot \frac{y^2-x^2}{x^2 y^2}\right) $

$=\frac{2}{(x+y)^4}\left(\frac{1}{x^3}-\frac{1}{y^3}+\frac{y-x}{x^2 y^2}\right)=\frac{2}{(x+y)^4} \cdot \frac{y^3-x^3+x y(y-x)}{x^3 y^3}$

$=\frac{2}{(x+y)^4} \cdot \frac{(y-x)\left(y^2+2 x y+x^2\right)}{x^3 y^3}=\frac{2(y-x)}{(x+y)^2 x^3 y^3}$

Do đó $A+B+C=\frac{\left(y^2+x^2\right)(y-x)}{x^4 y^4(x+y)^2}+\frac{2(y-x)}{x^3 y^3(x+y)^2}=$

$=\frac{\left(y^2+x^2\right)(y-x)+2 x y(y-x)}{x^4 y^4(x+y)^2}=\frac{(y-x)\left(y^2+x^2+2 x y\right)}{x^4 y^4(x+y)^2}=\frac{y-x}{x^4 y^4}$

 

BÀI TẬP

19. Thực hiện phép tính :
a) $\frac{x+3}{x+1}-\frac{2 x-1}{x-1}-\frac{x-3}{x^2-1}$
b) $\frac{1}{x(x+y)}+\frac{1}{y(x+y)}+\frac{1}{x(x-y)}+\frac{1}{y(y-x)}$.
10. Thực hiện phép tính :
a) $A=\frac{1}{(a-b)(a-c)}+\frac{1}{(b-a)(b-c)}+\frac{1}{(c-a)(c-b)}$;
b) $B=\frac{1}{a(a-b)(a-c)}+\frac{1}{b(b-a)(b-c)}+\frac{1}{c(c-a)(c-b)}$;
c) $\mathrm{C}=\frac{\mathrm{bc}}{(\mathrm{a}-\mathrm{b})(\mathrm{a}-\mathrm{c})}+\frac{\mathrm{ac}}{(\mathrm{b}-\mathrm{a})(\mathrm{b}-\mathrm{c})}+\frac{\mathrm{ab}}{(\mathrm{c}-\mathrm{a})(\mathrm{c}-\mathrm{b})}$;
d) $D=\frac{a^2}{(a-b)(a-c)}+\frac{b^2}{(b-a)(b-c)}+\frac{c^2}{(c-a)(c-b)}$.
11. Cho $\mathrm{a}, \mathrm{b}, \mathrm{c}$ là các số nguyên khác nhau đôi một. Chứng minh rằng biểu thức sau có giá trị là một số nguyên :
$P=\frac{a^3}{(a-b)(a-c)}+\frac{b^3}{(b-a)(b-c)}+\frac{c^3}{(c-a)(c-b)}$

12. Cho $3 y-x=6$. Tính giá trị của biểu thức

$A=\frac{x}{y-2}+\frac{2 x-3 y}{x-6}$

13. Tìm $\mathrm{x}, \mathrm{y}, \mathrm{z}$, biết rằng $\frac{\mathrm{x}^2}{2}+\frac{\mathrm{y}^2}{3}+\frac{\mathrm{z}^2}{4}=\frac{\mathrm{x}^2+\mathrm{y}^2+\mathrm{z}^2}{5}$.

14. Tìm $\mathrm{x}, \mathrm{y}$, biết rằng $\mathrm{x}^2+\mathrm{y}^2+\frac{1}{\mathrm{x}^2}+\frac{1}{\mathrm{y}^2}=4$.

15. Cho biết :

$\frac{1}{\mathrm{a}}+\frac{1}{\mathrm{~b}}+\frac{1}{\mathrm{c}}=2$

$\frac{1}{\mathrm{a}^2}+\frac{1}{\mathrm{~b}^2}+\frac{1}{\mathrm{c}^2}=2 .$

Chứng minh rằng $\mathrm{a}+\mathrm{b}+\mathrm{c}=\mathrm{abc}$.

16. Cho

$\frac{\mathrm{x}}{\mathrm{a}}+\frac{\mathrm{y}}{\mathrm{b}}+\frac{\mathrm{z}}{\mathrm{c}}=0$

và $\quad \frac{\mathrm{a}}{\mathrm{x}}+\frac{\mathrm{b}}{\mathrm{y}}+\frac{\mathrm{c}}{\mathrm{z}}=2$.

Tính giá trị của biểu thức : $\frac{\mathrm{a}^2}{\mathrm{x}^2}+\frac{\mathrm{b}^2}{\mathrm{y}^2}+\frac{\mathrm{c}^2}{\mathrm{z}^2}$.

17. Cho $(a+b+c)^2=a^2+b^2+c^2$ và $a, b, c$ khác 0 . Chứng minh rằng

$\frac{1}{a^3}+\frac{1}{b^3}+\frac{1}{c^3}=\frac{3}{a b c}$

18. Cho

$\frac{\mathrm{a}}{\mathrm{b}}+\frac{\mathrm{b}}{\mathrm{c}}+\frac{\mathrm{c}}{\mathrm{a}}=\frac{\mathrm{b}}{\mathrm{a}}+\frac{\mathrm{a}}{\mathrm{c}}+\frac{\mathrm{c}}{\mathrm{b}}$

Chứng minh rằng trong ba số $\mathrm{a}, \mathrm{b}, \mathrm{c}$, tồn tại hai số bằng nhau.

19. Tìm các giá trị nguyên của $\mathrm{x}$ để phân thức sau có giá trị là số nguyên :

a) $\mathrm{A}=\frac{2 \mathrm{x}^3-6 \mathrm{x}^2+\mathrm{x}-8}{\mathrm{x}-3}$

b) $\mathrm{B}=\frac{\mathrm{x}^4-2 \mathrm{x}^3-3 \mathrm{x}^2+8 \mathrm{x}-1}{\mathrm{x}^2-2 \mathrm{x}+1}$

c) $C=\frac{x^4+3 x^3+2 x^2+6 x-2}{x^2+2}$

20. Rút gọn biểu thức sau với $\mathrm{x}=\frac{\mathrm{a}}{3 \mathrm{a}+2}$ :

$A=\frac{x+3 a}{2-x}+\frac{x-3 a}{2+x}-\frac{2 a}{4-x^2}+a$

21. Rút gọn biểu thức :

$A=\frac{2}{a-b}+\frac{2}{b-c}+\frac{2}{c-a}+\frac{(a-b)^2+(b-c)^2+(c-a)^2}{(a-b)(b-c)(c-a)} .$

  1. Cho biết $\frac{a+b-c}{a b}-\frac{b+c-a}{b c}-\frac{a+c-b}{a c}=0$. Chứng minh rằng trong ba phân thức ở vế trái, có ít nhất một phân thức bằng 0 .

23. Xác định các số a, b, c sao cho :

a) $\frac{1}{x\left(x^2+1\right)}=\frac{a}{x}+\frac{b x+c}{x^2+1}$

b) $\frac{1}{x^2-4}=\frac{a}{x-2}+\frac{b}{x+2}$

c) $\frac{1}{(x+1)^2(x+2)}=\frac{a}{x+1}+\frac{b}{(x+1)^2}+\frac{c}{x+2}$.

24. Rút gọn biểu thức

$\mathrm{B}=(\mathrm{ab}+\mathrm{bc}+\mathrm{ca})\left(\frac{1}{\mathrm{a}}+\frac{1}{\mathrm{~b}}+\frac{1}{\mathrm{c}}\right)-\mathrm{abc}\left(\frac{1}{\mathrm{a}^2}+\frac{1}{\mathrm{~b}^2}+\frac{1}{\mathrm{c}^2}\right)$

25. Cho $\mathrm{a}, \mathrm{b}, \mathrm{c}$ khác nhau đôi một và $\frac{1}{\mathrm{a}}+\frac{1}{\mathrm{~b}}+\frac{1}{\mathrm{c}}=0$. Rút gọn các biểu thức :

a) $M=\frac{1}{a^2+2 b c}+\frac{1}{b^2+2 a c}+\frac{1}{c^2+2 a b}$

b) $\mathrm{N}=\frac{\mathrm{bc}}{\mathrm{a}^2+2 \mathrm{bc}}+\frac{\mathrm{ca}}{\mathrm{b}^2+2 \mathrm{ac}}+\frac{\mathrm{ab}}{\mathrm{c}^2+2 \mathrm{ab}}$;

c) $\mathrm{P}=\frac{\mathrm{a}^2}{\mathrm{a}^2+2 \mathrm{bc}}+\frac{\mathrm{b}^2}{\mathrm{~b}^2+2 \mathrm{ac}}+\frac{\mathrm{c}^2}{\mathrm{c}^2+2 \mathrm{ab}}$.

26. Cho các số $a, b, c$ khác nhau đôi một và $\frac{a+b}{c}=\frac{b+c}{a}=\frac{c+a}{b}$. Tính giá trị của biểu thức

$\mathrm{M}=\left(1+\frac{\mathrm{a}}{\mathrm{b}}\right)\left(1+\frac{\mathrm{b}}{\mathrm{c}}\right)\left(1+\frac{\mathrm{c}}{\mathrm{a}}\right)$

27*. Cho $\mathrm{a}^3+\mathrm{b}^3+\mathrm{c}^3=3 \mathrm{abc}$ và $\mathrm{a}+\mathrm{b}+\mathrm{c} \neq 0$. Tính giá trị của biểu thức :

$N=\frac{a^2+b^2+c^2}{(a+b+c)^2}$

28. Rút gọn các biểu thức :

a) $A=\left(1-\frac{1}{2^2}\right)\left(1-\frac{1}{3^2}\right)\left(1-\frac{1}{4^2}\right) \ldots\left(1-\frac{1}{n^2}\right)$;

b) $\mathrm{B}=\frac{1^2}{2^2-1} \cdot \frac{3^2}{4^2-1} \cdot \frac{5^2}{6^2-1} \cdot \cdots \cdot \frac{(2 n+1)^2}{(2 n+2)^2-1} .$

29. Rút gọn các biểu thức :

a) $\frac{1}{1.2}+\frac{1}{2.3}+\frac{1}{3.4}+\ldots+\frac{1}{(n-1) n}$;

b) $\frac{1}{2.5}+\frac{1}{5.8}+\frac{1}{8.11}+\ldots+\frac{1}{(3 n+2)(3 n+5)}$;

c) $\frac{1}{1.2 .3}+\frac{1}{2.3 .4}+\frac{1}{3.4 .5}+\ldots+\frac{1}{(n-1) n(n+1)}$.

30. Chứng minh rằng với mọi số tự nhiên $\mathrm{n} \geq 1$ :

a) $\frac{1}{2^2}+\frac{1}{4^2}+\frac{1}{6^2}+\ldots+\frac{1}{(2 n)^2}<\frac{1}{2}$

b) $\frac{1}{3^2}+\frac{1}{5^2}+\frac{1}{7^2}+\ldots+\frac{1}{(2 n+1)^2}<\frac{1}{4}$.

31. Chứng minh rằng với mọi số tự nhiện $\mathrm{n} \geq 2$ :

$A=\frac{1}{2^2}+\frac{1}{3^2}+\frac{1}{4^2}+\ldots+\frac{1}{n^2}<\frac{2}{3} .$

32. Chứng minh rằng với mọi số tự nhiên $\mathrm{n} \geq 3$ :

$\mathrm{B}=\frac{1}{3^3}+\frac{1}{4^3}+\frac{1}{5^3}+\ldots+\frac{1}{\mathrm{n}^3}<\frac{1}{12} $

33. Chứng minh rằng với mọi số tự nhiên $\mathrm{n} \geq 1$ :

$A=\left(1+\frac{1}{1.3}\right)\left(1+\frac{1}{2.4}\right)\left(1+\frac{1}{3.5}\right) \cdots\left(1+\frac{1}{n(n+2)}\right)<2$

34. Chứng minh rằng với mọi số tự nhiên $\mathrm{n} \geq 2$ :

$\mathrm{B}=\left(1-\frac{2}{6}\right)\left(1-\frac{2}{12}\right)\left(1-\frac{2}{20}\right) \ldots\left(1-\frac{2}{\mathrm{n}(\mathrm{n}+1)}\right)>\frac{1}{3} \text {. }$

35. Rút gọn biểu thức

$A=\frac{3^2-1}{5^2-1} \cdot \frac{7^2-1}{9^2-1} \cdot \frac{11^2-1}{13^2-1} \cdot \ldots \frac{43^2-1}{45^2-1} .$

36*. Chứng minh rằng :

a) $\mathrm{A}=\frac{2^3+1}{2^3-1} \cdot \frac{3^3+1}{3^3-1} \cdot \frac{4^3+1}{4^3-1} \cdot \ldots \cdot \frac{9^3+1}{9^3-1}<\frac{3}{2}$.

b) $\mathrm{B}=\frac{2^3-1}{2^3+1} \cdot \frac{3^3-1}{3^3+1} \cdot \ldots \cdot \frac{\mathrm{n}^3-1}{\mathrm{n}^3+1}>\frac{2}{3}$.

37*. Rút gọn biểu thức

$P=\frac{\left(1^4+4\right)\left(5^4+4\right)\left(9^4+4\right) \ldots\left(21^4+4\right)}{\left(3^4+4\right)\left(7^4+4\right)\left(11^4+4\right) \ldots\left(23^4+4\right)} .$

38. Rút gọn biểu thức

$M=\frac{1}{a^2-5 a+6}+\frac{1}{a^2-7 a+12}+\frac{1}{a^2-9 a+20}+\frac{1}{a^2-11 a+30}$

39. Rút gọn biểu thức

9.$\left(\frac{\mathrm{n}-1}{1}+\frac{\mathrm{n}-2}{2}+\frac{\mathrm{n}-3}{3}+\ldots+\frac{2}{\mathrm{n}-2}+\frac{1}{\mathrm{n}-1}\right):\left(\frac{1}{2}+\frac{1}{3}+\frac{1}{4}+\ldots+\frac{1}{\mathrm{n}}\right) .$

40. Rút gọn biểu thức

$\frac{A}{B}=\frac{\frac{1}{1(2 n-1)}+\frac{1}{3(2 n-3)}+\frac{1}{5(2 n-5)}+\ldots+\frac{1}{(2 n-3) \cdot 3}+\frac{1}{(2 n-1) .1}}{1+\frac{1}{3}+\frac{1}{5}+\ldots+\frac{1}{2 n-1}} .$

41. Cho

$a b c=1$

và $\quad \mathrm{a}+\mathrm{b}+\mathrm{c}=\frac{1}{\mathrm{a}}+\frac{1}{\mathrm{~b}}+\frac{1}{\mathrm{c}}$.

Chứng minh rằng trong ba số a, b, c, tồn tại một số bằng 1 .

42. Chứng minh rằng nếu $\mathrm{x}+\mathrm{y}+\mathrm{z}=\mathrm{a}$ và $\frac{1}{\dot{\mathrm{x}}}+\frac{1}{\mathrm{y}}+\frac{1}{\mathrm{z}}=\frac{1}{\mathrm{a}}$ thì tồn tại một trong ba số $\mathrm{x}, \mathrm{y}, \mathrm{z}$ bằng $\mathrm{a}$.

43. Các biểu thức $\mathrm{x}+\mathrm{y}+\mathrm{z}$ và $\frac{1}{\mathrm{x}}+\frac{1}{\mathrm{y}}+\frac{1}{\mathrm{z}}$ có thể cùng có giá trị bằng 0 được hay không ?

44. Tính giá trị của biểu thức $\mathrm{M}=\frac{1}{\mathrm{x}+2}+\frac{1}{\mathrm{y}+2}+\frac{1}{\mathrm{z}+2}$, biết rằng $2 a=b y+c z, 2 b=a x+c z, 2 c=a x+b y$ và $a+b+c \neq 0$.

45. a) Cho abc $=2$. Rút gọn biểu thức

$M=\frac{a}{a b+a+2}+\frac{b}{b c+b+1}+\frac{2 c}{a c+2 c+2} .$

b) Cho abc $=1$. Rút gọn biểu thức

$\mathrm{N}=\frac{\mathrm{a}}{\mathrm{ab}+\mathrm{a}+1}+\frac{\mathrm{b}}{\mathrm{bc}+\mathrm{b}+1}+\frac{\mathrm{c}}{\mathrm{ac}+\mathrm{c}+1} .$

46. Cho $\frac{\mathrm{a}}{\mathrm{c}}=\frac{\mathrm{a}-\mathrm{b}}{\mathrm{b}-\mathrm{c}}, \mathrm{a} \neq 0, \mathrm{c} \neq 0, \mathrm{a}-\mathrm{b} \neq 0, \mathrm{~b}-\mathrm{c} \neq 0$. Chứng minh rằng

$\frac{1}{a}+\frac{1}{a-b}=\frac{1}{b-c}-\frac{1}{c}$

47. Cho $\mathrm{a}+\mathrm{b}+\mathrm{c}=0(\mathrm{a} \neq 0, \mathrm{~b} \neq 0, \mathrm{c} \neq 0)$. Rút gọn các biểu thức :

a) $\mathrm{A}=\frac{\mathrm{a}^2}{\mathrm{bc}}+\frac{\mathrm{b}^2}{\mathrm{ca}}+\frac{\mathrm{c}^2}{\mathrm{ab}}$

b) $\mathrm{B}=\frac{\mathrm{a}^2}{\mathrm{a}^2-\mathrm{b}^2-\mathrm{c}^2}+\frac{\mathrm{b}^2}{\mathrm{~b}^2-\mathrm{c}^2-\mathrm{a}^2}+\frac{\mathrm{c}^2}{\mathrm{c}^2-\mathrm{a}^2-\mathrm{b}^2}$.

48*. Tính giá trị của biểu thức sau, biết rằng $\mathrm{a}+\mathrm{b}+\mathrm{c}=0$ :

$A=\left(\frac{a-b}{c}+\frac{b-c}{a}+\frac{c-a}{b}\right)\left(\frac{c}{a-b}+\frac{a}{b-c}+\frac{b}{c-a}\right) \text {. }$

49. Chứng minh rằng nếu $\left(\mathrm{a}^2-\mathrm{bc}\right)(\mathrm{b}-\mathrm{abc})=\left(\mathrm{b}^2-\mathrm{ac}\right)(\mathrm{a}-\mathrm{abc})$ và các số $\mathrm{a}, \mathrm{b}, \mathrm{c}, \mathrm{a}-\mathrm{b}$ khác 0 thì $\frac{1}{\mathrm{a}}+\frac{1}{\mathrm{~b}}+\frac{1}{\mathrm{c}}=\mathrm{a}+\mathrm{b}+\mathrm{c}$.

50*. Cho $a+b+c=0, x+y+z=0, \frac{a}{x}+\frac{b}{y}+\frac{c}{z}=0$. Chứng minh rằng

$a x^2+b y^2+c z^2=0 .$

51. Cho $\frac{x y+1}{y}=\frac{y z+1}{z}=\frac{x z+1}{x}$. Chứng minh rằng $x=y=z$ hoặc $x^2 y^2 z^2=1$.

52. Cho $\frac{a}{b+c}+\frac{b}{c+a}+\frac{c}{a+b}=1$. Chứng minh rằng $\frac{a^2}{b+c}+\frac{b^2}{c+a}+\frac{c^2}{a+b}=0$.

53*. Cho $\frac{\mathrm{a}}{\mathrm{b}-\mathrm{c}}+\frac{\mathrm{b}}{\mathrm{c}-\mathrm{a}}+\frac{\mathrm{c}}{\mathrm{a}-\mathrm{b}}=0$. Chứng minh rằng

$\frac{a}{(b-c)^2}+\frac{b}{(c-a)^2}+\frac{c}{(a-b)^2}=0$

54. Cho $\mathrm{x}+\frac{1}{\mathrm{x}}=\mathrm{a}$. Tính các biểu thức sau theo $\mathrm{a}$ :

a) $x^2+\frac{1}{x^2}$

b) $x^3+\frac{1}{x^3}$

c) $x^4+\frac{1}{x^4}$

d) $x^5+\frac{1}{x^5}$

55. Cho $\left(x^2-\frac{1}{x^2}\right):\left(x^2+\frac{1}{x^2}\right)=a$. Tính biểu thức

$M=\left(x^4-\frac{1}{x^4}\right):\left(x^4+\frac{1}{x^4}\right) \text { theo } a$

  1. Cho $x^2-4 x+1=0$. Tính giá trị của biểu thức $A=\frac{x^4+x^2+1}{x^2}$.

57. Cho $\frac{x}{x^2-x+1}=a$. Tính $M=\frac{x^2}{x^4+x^2+1}$ theo $a$.

58. Cho $x=\frac{b^2+c^2-a^2}{2 b c}, y=\frac{a^2-(b-c)^2}{(b+c)^2-a^2}$.

Tính giá trị của biểu thức $\mathrm{x}+\mathrm{y}+\mathrm{xy}$.

59. Tìm hai số tự nhiên a và b sao cho :

a) $a-b=\frac{a}{b}$;

b) $a-b=\frac{a}{2 b}$

60. Cho hai số nguyên dương $\mathrm{a}$ và $\mathrm{b}$ trong đó $\mathrm{a}>\mathrm{b}$. Tìm số nguyên dương $\mathrm{c}$ khác b sao cho

$\frac{a^3+b^3}{a^3+c^3}=\frac{a+b}{a+c}$

61. Cho dãy số $a_1, a_2, a_3, \ldots$ sao cho :

$a_2=\frac{a_1-1}{a_1+1} ; a_3=\frac{a_2-1}{a_2+1} ; \ldots ; a_n=\frac{a_{n-1}-1}{a_{n-1}+1} .$

a) Chứng minh rằng $\mathrm{a}_1=\mathrm{a}_5$.

b) Xác định năm số đầu của dãy, biết rằng $\mathrm{a}_{101}=3$.

62. Tìm phân số $\frac{\mathrm{m}}{\mathrm{n}}$ khác 0 và số tự nhiên $\mathrm{k}$, biết rằng $\frac{\mathrm{m}}{\mathrm{n}}=\frac{\mathrm{m}+\mathrm{k}}{\mathrm{nk}}$.

63*. Cho hai số tự nhiên a và $\mathrm{b}(\mathrm{a}<\mathrm{b})$. Tìm tổng các phân số tối giản có mẫu bằng 7 , mỗi phân số lớn hơn a nhưng nhỏ hơn b.

64. a) Mức sản xuất của một xí nghiệp năm 2001 tăng a\% so với năm 2000, năm 2002 tăng b\% so với năm 2001. Mức sản xuất của xí nghiệp đó năm 2002 tăng so với năm 2000 là :

A) $(a+b) \%$;

B) $a b \%$

C) $\left(a+b+\frac{a+b}{100}\right) \%$

D) $\left(a+b+\frac{a b}{100}\right) \%$

$\mathrm{E})\left(\frac{\mathrm{a}+\mathrm{b}}{100}+\frac{\mathrm{ab}}{10000}\right) \%$

Hãy chọn câu trả lời đúng.

b) Một số a tăng m\%, sau đó lại giảm đi n\% ( $\mathrm{a}, \mathrm{m}, \mathrm{n}$ là các số dương) thì được số $b$. Tìm liên hệ giữa $m$ và $n$ để $b>a$.

65*. Chứng minh rằng các tổng sau không là số nguyên :

a) $\mathrm{A}=\frac{1}{2}+\frac{1}{3}+\frac{1}{4}+\ldots+\frac{1}{\mathrm{n}}(\mathrm{n} \in \mathrm{N}, \mathrm{n} \geq 2)$

b) $\mathrm{B}=\frac{1}{3}+\frac{1}{5}+\frac{1}{7}+\ldots+\frac{1}{2 \mathrm{n}+1}(\mathrm{n} \in \mathrm{N}, \mathrm{n} \geq 1)$.

 

CHUYÊN ĐỀ: TÍNH CHIA HẾT ĐỐI VỚI ĐA THỨC

Định lý Bezout và áp dụng

1. Đa thức chia có dạng $x-a$ (a là hằng)

Ví dụ 1. Chứng minh rằng số dư khi chia đa thức $\mathrm{f}(\mathrm{x})$ cho nhị thức $\mathrm{x}$ – a bằng giá trị của đa thức $\mathrm{f}(\mathrm{x})$ tại $\mathrm{x}=\mathrm{a}$.

Định lí Bê-du (Bézout, 1730 – 1783, nhà toán học Pháp).

Giải : Do đa thức chia $\mathrm{x}$ – a có bậc nhất nên số dư khi chia $\mathrm{f}(\mathrm{x})$ cho $\mathrm{x}-\mathrm{a}$ là hằng số $\mathrm{r}$.

Ta có $\quad \mathrm{f}(\mathrm{x})=(\mathrm{x}-\mathrm{a}) 、 \mathrm{Q}(\mathrm{x})+\mathrm{r}$.

Đẳng thức trên đúng với mọi $\mathrm{x}$ nên với $\mathrm{x}=\mathrm{a}$ ta có

$f(a)=0 . Q(a)+r \text { hay } f(a)=r \text {. }$

Chú ý : Từ định lí Bê-du ta suy ra :

Đa thức $\mathrm{f}(\mathrm{x})$ chia hết cho $\mathrm{x}-\mathrm{a}$ khi và chỉ khi $\mathrm{f}(\mathrm{a})=0$ (tức là khi và chỉ khi a là nghiệm của đa thức).

Ví dụ 2. Chứng minh rằng nếu đa thức $\mathrm{f}(\mathrm{x})$ có tổng các hệ số bằng 0 thì đa thức ấy chia hết cho $\mathrm{x}-1$ ‘.

Giải : Gọi : $f(x)=a_ox^n+a_1 x^n-1+\ldots+a_n-1x+a_n$.

Theo giả thiết, $\quad a_0+a_1+\ldots+a_{n-1}+a_n=0 $.

Theo định lí Bê-du, số dư khi chia $\mathrm{f}(\mathrm{x})$ cho $\mathrm{x}-1$ là

$r = f(1) = a_\circ + a_1 + \ldots + a_{n-1} + a_n $

Từ (1) và (2) suy ra $r=0$. Vậy $\mathrm{f}(\mathrm{x})$ chia hết cho $\mathrm{x}-1$.

Ví dụ 3. Chứng minh rằng nếu đa thức $\mathrm{f}(\mathrm{x})$ có tổng các hệ số của các hạng tử bậc chẵn bằng tổng các hệ số của các hạng tử bậc lẻ thì đa thức ấy chia hết cho $x+1$.

Giải : Gọi $f(x)=a_0 x^{2 n}+a_1 x^{2 n-1}+a_2 x^{2 n-2}+\ldots+a_{2 n-2} x^2+a_{2 n-1} x+a_{2 n}$, trong đó $\mathrm{a}_0$ có thể bằng 0 .

Theo giả thiết

$a_\circ + a_2 + \ldots + a_{2n} = a_2 + a_3 + \ldots + a_{2n-1}$ nên

$\left(a_0+a_2+\ldots+a_{2 n}\right)-\left(a_1+a_3+\ldots+a_{2 n-1}\right)=0 .$

Theo định lí Bê-du, số dư khi chia $\mathrm{f}(\mathrm{x})$ cho $\mathrm{x}+1$ bằng

$r =f(-1)=a_0-a_1+a_2-\ldots+a_{2 n-2}-a_{2 n-1}+a_{2 n} $

$=\left(a_o+a_2+\ldots+a_{2 n}\right)-\left(a_1+a_3+\ldots+a_{2 n-1}\right) $

Từ (1) và (2) suy ra $\mathrm{r}=0$. Vậy $\mathrm{f}(\mathrm{x})$ chia hết cho $\mathrm{x}+1$.

CHUYÊN ĐỀ: MỘT SỐ PHƯƠNG PHÁP PHÂN TÍCH ĐA THỨC THÀNH NHÂN TỬ

Trong chuyên đề này, ta chỉ phân tích đa thức thành nhân tử với các hệ số nguyên.

 

PHƯƠNG PHÁP TÁCH MỘT HẠNG TỬ THÀNH NHIỀU HẠNG TỬ

 

Ví dụ 1. Phân tích đa thức thành nhân tử :

$\quad\quad\quad\quad\quad\quad\quad\quad3 x^2-8 x+4$

Giải : Đa thức trên không chứa nhân tử chung, không có dạng một hằng đẳng thức đáng nhớ nào, cũng không thể nhóm các hạng tử. Ta biến đổi đa thức ấy thành đa thức có nhiều hạng tử hơn.

Cách 1. (Tách hạng tử thứ hai)

$3 x^2-8 x+4=3 x^2-6 x-2 x+4=3 x(x-2)-2(x-2)=(x-2)(3 x-2) \text {. }$

Cách 2. (Tách hạng tử thứ nhất)

$3 x^2-8 x+4=4 x^2-8 x+4-x^2=(2 x-2)^2-x^2 $

$=(2 x-2+x)(2 x-2-x)=(3 x-2)(x-2) .$

Nhận xét : Trong cách 1 , hạng tử $-8 \mathrm{x}$ được tách thành hai hạng tử $-6 \mathrm{x}$ và $-2 x$. Trong đa thức $3 x^2-6 x-2 x+4$, hệ số của các hạng tử là $3,-6,-2,4$. Các hệ số thứ hai và thứ tư đều gấp $-2$ lần hệ số liền trước, nhờ đó mà xuất hiện nhân tử chung $x-2$.

Một cách tổng quát, để phân tích tam thức bậc hai $\mathrm{ax}^2+\mathrm{bx}+\mathrm{c}$ thành nhân tử, ta tách hạng tử bx thành $b_1 x+b_2 x$ sao cho $\frac{b_1}{a}=\frac{c}{b_2}$, tức là $b_1 b_2=a c$.

Trong thực hành ta làm như sau :

Bước I : Tìm tích ac.

Bước 2 : Phân tích ac ra tích của hai thừa số nguyên bằng mọi cách.

Bước 3 : Chọn hai thừa số mà tổng bằng $\mathrm{b}$.

Trong ví dụ trên, đa thức $3 \mathrm{x}^2-8 \mathrm{x}+4$ có $\mathrm{a}=3, \mathrm{~b}=-8, \mathrm{c}=4$. Tích $\mathrm{ac}=3.4=12$. Phân tích 12 ra tích của hai thừa số, hai thừa số này cùng dấu (vì tích của chúng bằng 12 ), và cùng âm (để tổng của chúng bằng $-8)$ : $(-1)(-12)$, $(-2)(-6),(-3)(-4)$. Chọn hai thừa số mà tổng bằng $-8$, đó là $-2$ và $-6$.

Ví dụ 2. Phân tích đa thức thành nhân tử :

$\quad\quad\quad\quad\quad\quad\quad\quad4 x^2-4 x-3$

Giải :

Cách 1. (Tách hạng tử thứ hai)

$4 \mathrm{x}^2-4 \mathrm{x}-3=4 \mathrm{x}^2+2 \mathrm{x}-6 \mathrm{x}-3=2 \mathrm{x}(2 \mathrm{x}+1)-3(2 \mathrm{x}+1)=(2 \mathrm{x}+1)(2 \mathrm{x}-3)$

Chú ý rằng hệ số $-4$ được tách thành 2 và $-6$ có tích bằng $-12$, bằng tích của $4(-3)$.

Cách 2. (Tách hạng tử thứ ba)

$4 x^2-4 x-3=4 x^2-4 x+1-4=(2 x-1)^2-2^2=(2 x+1)(2 x-3)$

Nhận xét : Qua hai ví dụ trên, ta thấy việc tách một hạng tử thành nhiều hạng tử khác thường nhằm mục đích

  • Làm xuất hiện các hệ số tỉ lệ, nhờ đó mà xuất hiện nhân tử chung (cách 1 ) ;

  • Làm xuất hiện hiệu của hai bình phương (cách 2).

Với các đa thức có bậc từ bậc ba trở lên, để dễ dàng làm xuất hiện các hệ số tỉ lệ, người ta thường dùng cách tìm nghiệm của đa thức.

Ta nhắc lại khái niệm nghiệm của đa thức : số a được gọi là nghiệm của đa thức $\mathrm{f}(\mathrm{x})$ nếu $\mathrm{f}(\mathrm{a})=0$. Như vậy, nếu đa thức $\mathrm{f}(\mathrm{x})$ có nghiệm $\mathrm{x}=\mathrm{a}$ thì nó chứa nhân tử $x-a$.

Ta chứng minh được rằng nghiệm nguyên của đa thức, nếu có, phải là ước của hệ số tự do.

Thật vậy, giả sử đa thức $a_0 x^n+a_1 x^{n-1}+\ldots+a_{n-1} x+a_n$ với các hệ số $\mathrm{a}_{\mathrm{O}}$, $a_1, \ldots, a_n$ nguyên, có nghiệm $x=a(a \in \mathbf{Z})$ . Thế thì

$a_0 x^n+a_1 x^{n-1}+\ldots+a_{n-1} x+a_n=(x-a)\left(b_0 x^{n-1}+b_1 x^{n-2}+\ldots+b_{n-1}\right)$

trong đó $b_0, b_1, \ldots, b_{n-1}$ nguyên. Hạng tử có bậc thấp nhất của tích ở vế phải bằng $-a b_{n-1}$ . Do đó $-a b_{n-1}=a_n$, tức a là ước của $a_n$

Ví dụ 3. Phân tích đa thức thành nhân tử :

$\quad\quad\quad\quad\quad\quad\quad\quad f(x)=x^3-x^2-4$

Giải : Lần lượt kiểm tra với $\mathrm{x}=\pm 1, \pm 2, \pm 4$, ta thấy $\mathrm{f}(2)=2^3-2^2-4=0$. Đa thức có nghiệm $\mathrm{x}=2$, do đó chứa nhân tử $\mathrm{x}-2$.

Ta tách các hạng tử như sau :

Cách 1. $\quad \mathrm{x}^3-\mathrm{x}^2-4=\mathrm{x}^3-2 \mathrm{x}^2+\mathrm{x}^2-2 \mathrm{x}+2 \mathrm{x}-4$

$=x^2(x-2)+x(x-2)+2(x-2)=(x-2)\left(x^2+x+2\right)$

Cách 2. $\quad \mathrm{x}^3-\mathrm{x}^2-4=\mathrm{x}^3-8-\mathrm{x}^2+4$

$=(x-2)\left(x^2+2 x+4\right)-(x+2)(x-2) $

$=(x-2)\left(x^2+2 x+4-x-2\right)=(x-2)\left(x^2+x+2\right)$

Chú ý : Khi xét nghiệm nguyên của đa thức, nên nhớ hai định lí sau :

a) Nếu đa thức $\mathrm{f}(\mathrm{x})$ có tổng các hệ số bằng 0 thì 1 là nghiệm của đa thức, do đó đa thức chứa nhân tử $\mathrm{x}-1$.

Chẳng hạn, đa thức $\mathrm{x}^3-5 \mathrm{x}^2+8 \mathrm{x}-4$ có $1-5+8-4=0$ nên 1 là nghiệm của đa thức, đa thức chứa nhân tử $\mathrm{x}-1$.

b) Nếu đa thức $\mathrm{f}(\mathrm{x})$ có tổng các hệ số của hạng tử bậc chẵn bằng tổng các hệ số của hạng tử bậc lẻ thì $-1$ là nghiệm của đa thức, đa thức chứa nhân tử $\mathrm{x}+1$.

Chẳng hạn, đa thức $x^3-5 x^2+3 x+9$ có $9-5=3+1$ nên $-1$ là nghiệm của đa thức, đa thức chứa nhân tử $\mathrm{x}+1$.

Chú ý : Để nhanh chóng loại trừ các ước của hệ số tự do không là nghiệm của đa thức, có thể dùng nhận xét sau :

Nếu a là nghiệm nguyên của đa thức $\mathrm{f}(\mathrm{x})$ và $\mathrm{f}(1), \mathrm{f}(-1)$ khác 0 thì $\frac{\mathrm{f}(1)}{\mathrm{a}-1}$ và $\frac{\mathrm{f}(-1)}{\mathrm{a}+1}$ đều là số nguyên.

Chứng̉ minh. Số a là nghiệm của $\mathrm{f}(\mathrm{x})$ nên

$f(x)=(x-a) \cdot Q(x) \quad\quad(1)$

Thay $x=1$ vào (1), ta có $f(1)=(1-a) \cdot Q(1)$.

Do $\mathrm{f}(1) \neq 0$ nên $\mathrm{a} \neq 1$, vì thế $\mathrm{Q}(1)=\frac{\mathrm{f}(1)}{1-\mathrm{a}}$, tức là $\frac{\mathrm{f}(1)}{\mathrm{a}-1}$ là số nguyên.

Thay $\mathrm{x}=-1$ vào (1). Chứng minh tương tự, ta cũng có $\frac{\mathrm{f}(-1)}{a+l}$ là số nguyên. Lấy một ví dụ : $\quad \mathrm{f}(\mathrm{x})=4 \mathrm{x}^3-13 \mathrm{x}^2+9 \mathrm{x}-18$.

Các ước của 18 là $\pm 1, \pm 2, \pm 3, \pm 6, \pm 9, \pm 18$.

$f(1)=4-13+9-18=-18, f(-1)=-4-13-9-18=-44 .$

Hiển nhiên $\pm 1$ không là nghiệm của $f(x)$. Ta thấy $\frac{-18}{-3-1}, \frac{-18}{\pm 6-1}, \frac{-18}{\pm 9-1}$, $\frac{-18}{\pm 18-1}$ không nguyên nên $-3, \pm 6, \pm 9, \pm 18$ không là nghiệm của $f(x)$.

Ta thấy $\frac{-44}{2+1}$ không nguyên nên 2 không là nghiệm của $f(x)$. Chỉ còn $-2$ và 3 .

Kiểm tra ta thấy 3 là nghiệm của $\mathrm{f}(\mathrm{x})$. Do đó, ta tách các hạng tử như sau :

$ 4 x^3-13 x^2+9 x-18=4 x^3-12 x^2-x^2+3 x+6 x-18 $

$= 4 x^2(x-3)-x(x-3)+6(x-3)=(x-3)\left(4 x^2-x+6\right)$

Ví dụ 3. Phân tích đa thức thành nhân tử :

$\quad\quad\quad\quad\quad\quad\quad\quad3 x^3-7 x^2+17 x-5$

Giải : Các số $\pm 1, \pm 5$ không là nghiệm của đa thức. Như vậy, đa thức không có nghiệm nguyên. Tuy vậy, đa thức có thể có nghiệm hữu tỉ khác. Ta chứng minh được rằng trong đa thức có các hệ số nguyên, nghiệm hữu tỉ (nếu có) phải có dạng $\frac{\mathrm{p}}{\mathrm{q}}$ trong đó $\mathrm{p}$ là ước của hệ số tự do, $\mathrm{q}$ là ước dương của hệ số cao nhất (*).

Xét các số $\pm \frac{1}{3}, \pm \frac{5}{3}$, ta thấy $\frac{1}{3}$ là nghiệm của đa thức, do đó đa thức chứa thừa số $3 x-1$. Ta tách các hạng tử như sau :

$3 x^3-7 x^2+17 x-5=3 x^3-x^2-6 x^2+2 x+15 x-5 $

$= x^2(3 x-1)-2 x(3 x-1)+5(3 x-1)=(3 x-1)\left(x^2-2 x+5\right)$

(*) $-$ Thật vậy, giả sử đa thức $a_0 x^n+a_1 x^{n-1}+\ldots+a_{n-1} x+a_n$ với các hệ số $a_0, a_1, \ldots, a_n$ nguyên, có nghiệm hữu tỉ $x=\frac{p}{q}$, trong đó $p, q \in \mathbf{Z}, \mathrm{q}>0,(\mathrm{p}, \mathrm{q})=1$. Thế thì

$a_0 x^n+a_1 x^{n-1}+\ldots+a_{n-1} x+a_n=(q x-p)\left(b_0 x^{n-1}+b_1 x^{n-2}+\ldots+b_{n-1}\right)$

Ta có $-pb_n-1=a_n$, $qb_o=a_o$ nên p là ước của $a_n$; còn q là ước dương của $a_o$.

 

PHƯƠNG PHÁP THÊM VÀ BỚT CÙNG MỘT HẠNG TỬ

 

1. Thêm và bớt cùng một hạng tử làm xuất hiện hiệu của hai bình phương

Ví dụ 4. Phân tích đa thức thành nhân tử :

$\quad\quad\quad\quad\quad\quad\quad\quad4 x^4+81$

Giải : Thêm và bớt $36 \mathrm{x}^2$ :

$4 \mathrm{x}^4+81=4 \mathrm{x}^4+36 \mathrm{x}^2+81-36 \mathrm{x}^2$

$=\left(2 x^2+9\right)^2-(6 x)^2=\left(2 x^2+9+6 x\right)\left(2 x^2+9-6 x\right) .$

Ví dụ 5. Phân tích đa thức thành nhân tử :

$\quad\quad\quad\quad\quad\quad\quad\quad 64 x^4+y^4 \text {. }$

Giải : Thêm và bớt $16 \mathrm{x}^2 \mathrm{y}^2$ :

$64 x^4+y^4 =64 x^4+16 x^2 y^2+y^4-16 x^2 y^2=\left(8 x^2+y^2\right)^2-(4 x y)^2 $

$=\left(8 x^2+y^2+4 x y\right)\left(8 x^2+y^2-4 x y\right)$

2. Thêm và bớt cùng một hạng tử làm xuất hiện nhân tử chung

Ví dụ 6. Phân tích đa thức thành nhân tử :

$\quad\quad\quad\quad\quad\quad\quad\quad x^5+x-1$

Giải :

Cách 1:

$x^5+x-1 =x^5-x^4+x^3+x^4-x^3+x^2-x^2+x-1 $

$=x^3\left(x^2-x+1\right)+x^2\left(x^2-x+1\right)-\left(x^2-x+1\right) $

$=\left(x^2-x+1\right)\left(x^3+x^2-1\right)$

Cách 2. Thêm và bớt $\mathrm{x}^2$ :

$x^5+x-1=x^5+x^2-x^2+x-1=x^2\left(x^3+1\right)-\left(x^2-x+1\right) $

$=\left(x^2-x+1\right)\left[x^2(x+1)-1\right]=\left(x^2-x+1\right)\left(x^3+x^2-1\right)$

Ví dụ 7. Phân tích đa thức thành nhân tử :

$\quad\quad\quad\quad\quad\quad\quad\quad x^7+x^2+1$

Giải : Thêm và bớt x :

$x^7+x^2+1 =x^7-x+x^2+x+1 $

$=x\left(x^3+1\right)\left(x^3-1\right)+\left(x^2+x+1\right) $

$=x\left(x^3+1\right)(x-1)\left(x^2+x+1\right)+\left(x^2+x+1\right) $

$=\left(x^2+x+1\right)\left(x^5-x^4+x^2-x+1\right)$

Chú ý : Các đa thức dạng $\mathrm{x}^{3 \mathrm{~m}+1}+\mathrm{x}^{3 \mathrm{n}+2}+1 \mathrm{nhu} \mathrm{x}^7+\mathrm{x}^2+1, \mathrm{x}^7+\mathrm{x}^5+1$, $x+x^5+1, x+x^8+1, \ldots$ đều chứa nhân tử $x^2+x+1$

PHƯƠNG PHÁP ĐỔI BIẾN

Ví dụ 8. Phân tích đa thức thành nhân tử :

$\quad\quad\quad\quad\quad\quad\quad\quad x(x+4)(x+6)(x+10)+128$

Giải :

$x(x+4)(x+6)(x+10)+128=\left(x^2+10 x\right)\left(x^2+10 x+24\right)+128$

Đặt $x^2+10 x+12=y$, đa thức đã cho có dạng :

$(y-12)(y+12)+128=y^2-16=(y+4)(y-4) $

$=\left(x^2+10 x+16\right)\left(x^2+10 x+8\right)=(x+2)(x+8)\left(x^2+10 x+8\right) .$

Nhận xét : Trong ví dụ trên, nhờ phương pháp đổi biến, ta đã đưa đa thức bậc bốn đối với $x$ thành đa thức bậc hai đối với y.

Ví dụ 9. Phân tích đa thức thành nhân tử :

$\quad\quad\quad\quad\quad\quad\quad\quad A=x^4+6 x^3+7 x^2-6 x+1$

Giải : Giả sử $\mathrm{x} \neq 0$. Ta viết đa thức dưới dạng :

$A=x^2\left(x^2+6 x+7-\frac{6}{x}+\frac{1}{x^2}\right)=x^2\left[\left(x^2+\frac{1}{x^2}\right)+6\left(x-\frac{1}{x}\right)+7\right] \text {. }$

Đặt $x-\frac{1}{x}=y$ thì $x^2+\frac{1}{x^2}=y^2+2$. Do đó

$A =x^2\left(y^2+2+6 y+7\right)=x^2(y+3)^2=(x y+3 x)^2 $

$=\left[x\left(x-\frac{1}{\dot{x}}\right)+3 x\right]^2=\left(x^2+3 x-1\right)^2$

Dạng phân tích này cũng đúng với $x=0$.

Chú ý : Có thể trình bày lời giải của ví dụ trên như sau :

$A =x^4+6 x^3-2 x^2+9 x^2-6 x+1 $

$=x^4+2 x^2(3 x-1)+(3 x-1)^2=\left(x^2+3 x-1\right)^2$

 

PHƯƠNG PHÁP HỆ SỐ BẤT ĐỊNH

Ví dụ 10. Phân tích đa thức thành nhân tử :

$\quad\quad\quad\quad\quad\quad\quad\quad x^4-6 x^3+12 x^2-14 x+3$

Giải : Các số $\pm 1, \pm 3$ không là nghiệm của đa thức, đa thức không có nghiệm nguyên, cũng không có nghiệm hữu tỉ. Như vậy nếu đa thức trè̀n phân tích được thành nhân tử thì phải có dạng $\left(x^2+a x+b\right)\left(x^2+c x+d\right)$. Phép nhân. này cho kết quả $\mathrm{x}^4+(\mathrm{a}+\mathrm{c}) \mathrm{x}^3+(\mathrm{ac}+\mathrm{b}+\mathrm{d}) \mathrm{x}^2+(\mathrm{ad}+\mathrm{bc}) \mathrm{x}+\mathrm{bd}$. Đồng nhất đa thức này với đa thức đã cho, ta được hệ điều kiện :

$\quad\quad\quad\quad\quad\quad\quad\quad\left\{\begin{array}{l}a+c=-6 \\ a c+b+d=12 \\ a d+b c=-14 \\ b d=3 .\end{array}\right.$

Xét bd $=3$ với $\mathrm{b}, \mathrm{d} \in \mathbf{Z}, \mathrm{b} \in{\pm 1, \pm 3}$. Với $\mathrm{b}=3$ thì $\mathrm{d}=1$, hệ điều kiện trên trở thành :

$\quad\quad\quad\quad\quad\quad\quad\quad\left\{\begin{array}{l}a+c=-6 \\ a c=8 \\ a+3 c=-14\end{array}\right.$

Suy ra $2 \mathrm{c}=-14-(-6)=-8$. Do đó $\mathrm{c}=-4$, $\mathrm{a}=-2$.

Vậy đa thức đã cho phân tich thành $\left(x^2-2 x+3\right)\left(x^2-4 x+1\right)$.

Chú ý : Ta trình bày lời giải của ví dụ trên như sau :

$x^4-6 x^3+12 x^2-14 x+3=$

$= x^4-4 x^3+x^2-2 x^3+8 x^2-2 x+3 x^2-12 x+3 $

$= x^2\left(x^2-4 x+1\right)-2 x\left(x^2-4 x+1\right)+3\left(x^2-4 x+1\right) $

$=\left(x^2-4 x+1\right)\left(x^2-2 x+3\right)$

PHƯƠNG PHÁP XÉT GIÁ TRỊ RIÊNG

Trong phương pháp này, trước hết ta xác định dạng các nhân tử chứa biến của đa thức, rồi gán cho các biến các giá trị cụ thể để xác định nhân tử còn lại.

Ví dụ 11. Phân tích đa thức thành nhân tử :

$\quad\quad\quad\quad\quad\quad\quad\quad P=x^2(y-z)+y^2(z-x)+z^2(x-y)$

Giải : Thử thay $\mathrm{x}$ bởi $\mathrm{y}$ thì $\mathrm{P}=\mathrm{y}^2(\mathrm{y}-\mathrm{z})+\mathrm{y}^2(\mathrm{z}-\mathrm{y})=0$. Như vậy $\mathrm{P}$ chia hết cho $\mathrm{x}-\mathrm{y}$.

Ta lại thấy nếu thay $\mathrm{x}$ bởi $\mathrm{y}$, thay $\mathrm{y}$ bởi $\mathrm{z}$, thay $\mathrm{z}$ bởi $\mathrm{x}$ thì $\mathrm{P}$ không đổi ( $\mathrm{ta}$ nói đa thức $\mathrm{P}$ có thể hoán vị vòng quanh $\mathrm{x} \rightarrow \mathrm{y} \rightarrow \mathrm{z} \rightarrow \mathrm{x}$ ). Do đó, nếu $\mathrm{P}$ đã chia hết cho $\mathrm{x}-\mathrm{y}$ thì cũng chia hết cho $\mathrm{y}-\mathrm{z}$ và $\mathrm{z}-\mathrm{x}$. Vậy $\mathrm{P}$ có dạng

$\mathrm{k}(\mathrm{x}-\mathrm{y})(\mathrm{y}-\mathrm{z})(\mathrm{z}-\mathrm{x}) \text {. }$

Ta thấy $\mathrm{k}$ phải là hằng số (không chứa biến) vì $\mathrm{P}$ có bậc ba đối với tập hợp các biến $\mathrm{x}, \mathrm{y}, \mathrm{z}$, còn tích $(\mathrm{x}-\mathrm{y})(\mathrm{y}-\mathrm{z})(\mathrm{z}-\mathrm{x})$ cũng có bậc ba đối với tập hợp các biến $\mathrm{x}, \mathrm{y}, \mathrm{z}$.

Vì đẳng thức $\mathrm{x}^2(\mathrm{y}-\mathrm{z})+\mathrm{y}^2(\mathrm{z}-\mathrm{x})+\mathrm{z}^2(\mathrm{x}-\mathrm{y})=\mathrm{k}(\mathrm{x}-\mathrm{y})(\mathrm{y}-\mathrm{z})(\mathrm{z}-\mathrm{x})$ đúng với mọi $\mathrm{x}, \mathrm{y}, \mathrm{z}$ nên ta gán cho các biến $\mathrm{x}, \mathrm{y}, \mathrm{z}$ các giá trị riêng, chẳng hạn $\mathrm{x}=2$, $\mathrm{y}=1$, $\mathrm{z}=0$(*), ta được :

$4 \cdot 1+1 \cdot(-2)+0=\mathrm{k} \cdot 1 \cdot 1 \cdot(-2) \Leftrightarrow 2=-2 \mathrm{k} \Leftrightarrow \mathrm{k}=-1 \text {. }$

Vậy $P=-(x-y)(y-z)(z-x)=(x-y)(y-z)(x-z)$.

(*) Các giá trị của $\mathrm{x}, \mathrm{y}, \mathrm{z}$ có thể chọn tuỳ ý, chỉ cần chúng đôi một khác nhau để

$(x-y)(y-z)(z-x) \neq 0$

 

BÀI TẬP

 

Phân tích các đa thức sau thành nhân tử (từ bài 1 đến bài 14)

1. a) $6 \mathrm{x}^2-11 \mathrm{x}+3$

b) $2 x^2+3 x-27$

c) $2 x^2-5 x y-3 y^2$

2. a) $x^3+2 x-3$;

b) $x^3-7 x+6$

c) $x^3+5 x^2+8 x+4$

d) $x^3-9 x^2+6 x+16$

e) $x^3-x^2-x-2$;

g) $x^3+x^2-x+2$;

h) $x^3-6 x^2-x+30$.

3. $x^3-7 x-6$ (giải bằng nhiều cách).

4. a) $27 \mathrm{x}^3-27 \mathrm{x}^2+18 \mathrm{x}-4$

b) $2 x^3-x^2+5 x+3$;

c) $\left(x^2-3\right)^2+16$.

5. a) $\left(x^2+x\right)^2-2\left(x^2+x\right)-15$;

b) $x^2+2 x y+y^2-x-y-12$

c) $\left(x^2+x+1\right)\left(x^2+x+2\right)-12$;

d) $(x+2)(x+3)(x+4)(x+5)-24$

6. a) $(x+a)(x+2 a)(x+3 a)(x+4 a)+a^4$;

b) $\left(x^2+y^2+z^2\right)(x+y+z)^2+(x y+y z+z x)^2$;

$\left.c^*\right) 2\left(x^4+y^4+z^4\right)-\left(x^2+y^2+z^2\right)^2-2\left(x^2+y^2+z^2\right)(x+y+z)^2+$

$+(x+y+z)^4$

7*. $(\mathrm{a}+\mathrm{b}+\mathrm{c})^3-4\left(\mathrm{a}^3+\mathrm{b}^3+\mathrm{c}^3\right)-12 \mathrm{abc}$ bằng cách đổi biến : đặt $\mathrm{a}+\mathrm{b}=\mathrm{m}$, $a-b=n$.

8. a) $4 x^4-32 x^2+1$;

b) $x^6+27$;

c) $3\left(x^4+x^2+1\right)-\left(x^2+x+1\right)^2$

d) $\left(2 x^2-4\right)^2+9$

9. a) $4 x^4+1$

b) $4 x^4+y^4$;

c) $x^4+324$.

10. a) $x^5+x^4+1$

b) $x^5+x+1$

c) $x^8+x^7+1$

d) $x^5-x^4-1$

e) $x^7+x^5+1$

g) $x^8+x^4+1$.

11. a) $a^6+a^4+a^2 b^2+b^4-b^6$

$\left.b^*\right) x^3+3 x y+y^3-1$

12. Dùng phương pháp hệ số bất định :

a) $4 x^4+4 x^3+5 x^2+2 x+1$

b) $x^4-7 x^3+14 x^2-7 x+1$

c) $x^4-8 x+63$

d) $(x+1)^4+\left(x^2+x+1\right)^2$.

13* *. a) $x^8+14 x^4+1$;

b) $x^8+98 x^4+1$

14. Dùng phương pháp xét giá trị riêng :

$\mathrm{M}=\mathrm{a}(\mathrm{b}+\mathrm{c}-\mathrm{a})^2+\mathrm{b}(\mathrm{c}+\mathrm{a}-\mathrm{b})^2+\mathrm{c}(\mathrm{a}+\mathrm{b}-\mathrm{c})^2+$

$+(\mathrm{a}+\mathrm{b}-\mathrm{c})(\mathrm{b}+\mathrm{c}-\mathrm{a})(\mathrm{c}+\mathrm{a}-\mathrm{b})$

$180(3)$. Chứng minh rằng tích của bốn số tự nhiên liên tiếp cộng thêm 1 là một số chính phương.

15*. Chứng minh rằng số $\mathrm{A}=(\mathrm{n}+1)^4+\mathrm{n}^4+1$ chia hết cho một số chính phương khác 1 với mọi số n nguyên dương.

16. Tìm các số nguyên a, b, c sao cho khi phân tích đa thức $(x+a)(x-4)-7$ thành nhân tử ta được $(\mathrm{x}+\mathrm{b})(\mathrm{x}+\mathrm{c})$.

17. Tìm các số hữu tỉ $a, b, c$ sao cho khi phân tích đa thức $x^3+a x^2+b x+c$ thành nhân tử ta được $(x+a)(x+b)(x+c)$.

18. Số tự nhiên $\mathrm{n}$ có thể nhận bao nhiêu giá trị, biết rằng khi phân tích đa thức $x^2+x-n$ thành nhân tử ta được $(x-a)(x+b)$ với $a, b$ là các số tự nhiên và $1<\mathrm{n}<100$ ?

19. Cho $A=a^2+b^2+c^2$, trong đó $a$ và $b$ là hai số tự nhiên liên tiếp, $\mathrm{c}=\mathrm{ab}$. Chứng minh rằng $\sqrt{\mathrm{A}}$ là một số tự nhiên lẻ.

 

 

 

 

 

 

 

 

 

 

 

 

 

 

 

 

 

PHÂN THỨC ĐẠI SỐ – P.1

TÍNH CHẤT CƠ BẢN CỦA PHÂN THỨC.

RÚT GỌN PHÂN THỨC

Phân thức đại số là một biểu thức có dạng $\frac{\mathrm{A}}{\mathrm{B}}$, trong đó $\mathrm{A}$ và $\mathrm{B}$ là các đa thức, $\mathrm{B} \neq 0$.

Phân thức đại số có các tính chất cơ bản sau :

$-$ Nếu nhân cả tử thức và mẫu thức của một phân thức với cùng một đa thức khác 0 thì được một phân thức bằng phân thức đã cho.

$-$ Nếu chia cả tử thức và mầu thức của một phân thức cho cùng một nhân tử chung của chúng thì được một phân thức bằng phân thức đã cho.

Muốn rút gọn một phân thức đại số, ta có thể :

$-$ Phân tích tử thức và mẫu thức thành nhân tử ;

$-$ Chia cả tử thức và mẫu thức cho nhân tử chung.

Ví dụ 1. Cho phân thức

$M=\frac{\left(a^2+b^2+c^2\right)(a+b+c)^2+(a b+b c+c a)^2}{(a+b+c)^2-(a b+b c+c a)}$

a) Tìm các giá trị của $\mathrm{a}, \mathrm{b}, \mathrm{c}$ để phân thức được xác định (tức là để mẫu . khác 0).

b) Rút gọn phân thức $M$.

Giải : Ta có

$(a+b+c)^2-(a b+b c+c a)=0 \Leftrightarrow a^2+b^2+c^2+a b+b c+c a=0 $

$\Leftrightarrow  2 a^2+2 b^2+2 c^2+2 a b+2 b c+2 c a=0 $

$\Leftrightarrow (a+b)^2+(b+c)^2+(c+a)^2=0 \Leftrightarrow a+b=b+c=c+a=0 $

$\Leftrightarrow  a=b=c=0$

Vậy điều kiện để phân thức $\mathrm{M}$ được xác định là $\mathrm{a}, \mathrm{b}$, $\mathrm{c}$ không đồng thời bằng $0 .$

b) Chú ý rằng $(a+b+c)^2=a^2+b^2+c^2+2(a b+b c+c a)$. Do đó, ta đặt $a^2+b^2+c^2=x, a b+b c+c a=y$. Khi đó $(a+b+c)^2=x+2 y$. Ta có

$M=\frac{x(x+2 y)+y^2}{x+2 y-y}=\frac{x^2+2 x y+y^2}{x+y}=\frac{(x+y)^2}{x+y}=x+y$

$=a^2+b^2+c^2+a b+b c+c a .$

Ví  dụ 2.Rút gọn phân thức

$A=\frac{(b-c)^3+(c-a)^3+(a-b)^3}{a^2(b-c)+b^2(c-a)+c^2(a-b)} .$

Giải : Phân tích mẫu thức thành nhân tử :

$a^2(b-c)+b^2(c-a)+c^2(a-b)=a^2(b-c)+b^2 c-a b^2+a c^2-b c^2 $

$= a^2(b-c)+b c(b-c)-a\left(b^2-c^2\right)=(b-c)\left(a a^2+b c-a b-a c\right) $

$=(b-c)[a(a-b)-c(a-b)]=(b-c)(a-b)(a-c) . $

$\text { Do đó } \quad A=\frac{(b-c)^3+(c-a)^3+(a-b)^3}{-(a-b)(b-c)(c-a)} .$

Ta có nhận xét : Nếu $x+y+z=0$ thì $x^3+y^3+z^3=3 x y z$ (chứng minh : xem bài tập 42). Đặt $b-c=x, c-a=y, a-b=z$ thì $x+y+z=0$. Theo nhận xét trên :

$A=\frac{x^3+y^3+z^3}{-x y z}=\frac{3 x y z}{-x y z}=-3$

Ví dụ 3. Chứng minh rằng với mọi số nguyên n thì phân số $\frac{n^3+2 n}{n^4+3 n^2+1}$ là phân số tối giản.

Giải : Để chứng minh phân số đã cho là tối giản, ta sẽ chứng tỏ rằng tử và mẫu chỉ có ước chung là $\pm 1$.

Gọi d là ước chung của $n^3+2 n$ và $n^4+3 n^2+1$. Ta có :

$n^3+2 n \vdots d \Rightarrow n\left(n^3+2 n\right) \vdots d \Rightarrow n^4+2 n^2 \vdots d $

$n^4+3 n^2+1-\left(n^4+2 n^2\right)=n^2+1 \vdots d \Rightarrow\left(n^2+1\right)^2=n^4+2 n^2+1 \vdots d$

Từ $(1)$ và $(2)$ suy ra

$\left(n^4+2 n^2+1\right)-\left(n^4+2 n^2\right): d \Rightarrow 1: d \Rightarrow d=\pm 1 .$

Vậy $\frac{n^3+2 n}{n^4+3 n^2+1}$ là phân số tối giản.

Ví dụ 4. Chứng minh rằng

$1+x+x^2+x^3+\ldots+x^{31}=(1+x)\left(1+x^2\right)\left(1+x^4\right)\left(1+x^8\right)\left(1+x^{16}\right)\quad(1)$

Giải : Gọi vế trái của đẳng thức (1) là $\mathrm{A}$, vế phải là $\mathrm{B}$.

Ta có $(1-\mathrm{x}) \cdot \mathrm{A}=1-\mathrm{x}^{32}$ theo hằng đẳng thức 8 ,

$(1-x) \cdot B=(1-x)(1+x)\left(1+x^2\right)\left(1+x^4\right)\left(1+x^8\right)\left(1+x^{16}\right)=1-x^{32} \text {. }$

Nếu $\mathrm{x} \neq 1$ thì $\mathrm{A}$ và $\mathrm{B}$ đều bằng phân thức $\frac{1-\mathrm{x}^{32}}{1-\mathrm{x}}$. Do đó $\mathrm{A}=\mathrm{B}$.

Nếu $\mathrm{x}=1$ thì hai vế của (1) đều bằng 32 . Do đó $\mathrm{A}=\mathrm{B}$.

Trong cả hai trường hợp, đẳng thức (1) đều đúng.

 

BÀI TẬP

1. Tìm giá trị của $\mathrm{x}$ để các phân thức sau bằng 0 :

a) $\frac{x^4+x^3+x+1}{x^4-x^3+2 x^2-x+1}$

b) $\frac{x^4-5 x^2+4}{x^4-10 x^2+9}$

2. Rút gọn các phân thức :

a) $\mathrm{A}=\frac{1235.2469-1234}{1234.2469+1235}$;

b) $\mathrm{B}=\frac{4002}{1000.1002-999.1001}$.

3. Rút gọn các phân thức :

a) $\frac{3 x^3-7 x^2+5 x-1}{2 x^3-x^2-4 x+3}$

b) $\frac{(x-y)^3-3 x y(x+y)+y^3}{x-6 y}$

c) $\frac{x^2+y^2+z^2-2 x y+2 x z-2 y z}{x^2-2 x y+y^2-z^2}$.

4. Rút gọn các phân thức với n là số tự nhiên :

a) $\frac{(n+1) !}{n !(n+2)}$

b) $\frac{n !}{(n+1) !-n !}$

c) $\frac{(n+1) !-(n+2) !}{(n+1) !+(n+2) !}$

5. Rút gọn các phân thức :

a) $\frac{a^2(b-c)+b^2(c-a)+c^2(a-b)}{a b^2-a c^2-b^3+b c^2}$;

b) $\frac{2 x^3-7 x^2-12 x+45}{3 x^3-19 x^2+33 x-9}$

c) $\frac{x^3-y^3+z^3+3 x y z}{(x+y)^2+(y+z)^2+(z-x)^2}$

d) $\frac{x^3+y^3+z^3-3 x y z}{(x-y)^2+(y-z)^2+(z-x)^2}$.

6. Chứng minh rằng các phân số sau tối giản với mọi số tự nhiên $\mathrm{n}$ :

a) $\frac{3 n+1}{5 n+2}$;

b) $\frac{12 n+1}{30 n+2}$

$\left.c^*\right) \frac{n^3+2 n}{n^4+3 n^2+1}$

d) $\frac{2 n+1}{2 n^2-1}$.

7. Chứng minh rằng phân số $\frac{n^7+n^2+1}{n^8+n+1}$ không tối giản với mọi số nguyên dương $n$.

8. Viết gọn biểu thức sau dưới dạng một phân thức :

$\left(x^2-x+1\right)\left(x^4-x^2+1\right)\left(x^8-x^4+1\right)\left(x^{16}-x^8+1\right)\left(x^{32}-x^{16}+1\right)$

9. Cho biết $\mathrm{x}, \mathrm{y}, \mathrm{z}$ khác 0 và $\frac{(\mathrm{ax}+\mathrm{by}+\mathrm{cz})^2}{\mathrm{x}^2+\mathrm{y}^2+\mathrm{z}^2}=\mathrm{a}^2+\mathrm{b}^2+\mathrm{c}^2$.

Chứng minh rằng $\frac{\mathrm{a}}{\mathrm{x}}=\frac{\mathrm{b}}{\mathrm{y}}=\frac{\mathrm{c}}{\mathrm{z}}$.

10*. Cho biết $\mathrm{ax}+\mathrm{by}+\mathrm{cz}=0$.

Rút gọn $\mathrm{A}=\frac{\mathrm{bc}(\mathrm{y}-\mathrm{z})^2+\mathrm{ca}(\mathrm{z}-\mathrm{x})^2+\mathrm{ab}(\mathrm{x}-\mathrm{y})^2}{a \mathrm{x}^2+\mathrm{by}^2+c \mathrm{z}^2}$.

11. Rút gọn $\frac{\mathrm{x}^2+\mathrm{y}^2+\mathrm{z}^2}{(\mathrm{y}-\mathrm{z})^2+(\mathrm{z}-\mathrm{x})^2+(\mathrm{x}-\mathrm{y})^2}$, biết rằng $\mathrm{x}+\mathrm{y}+\mathrm{z}=0$.

12. Tính giá trị của biểu thức $\mathrm{A}=\frac{\mathrm{x}-\mathrm{y}}{\mathrm{x}+\mathrm{y}}$, biết $\mathrm{x}^2-2 \mathrm{y}^2=\mathrm{xy}(\mathrm{y} \neq 0 ; \mathrm{x}+\mathrm{y} \neq 0)$.

13. Tính giá trị của phân thức $A=\frac{3 x-2 y}{3 x+2 y}$, biết rằng $9 x^2+4 y^2=20 x y$ và $2 y<3 x<0$

14. Cho $3 \mathrm{x}-\mathrm{y}=3 \mathrm{z}$ và $2 \mathrm{x}+\mathrm{y}=7 \mathrm{z}$. Tính giá trị của biểu thức

$M=\frac{x^2-2 x y}{x^2+y^2}(x \neq 0, y \neq 0)$

15. Tìm số nguyên $x$ để phân thức sau có giá trị là số nguyên :

a) $\frac{3}{2 x-1}$

b) $\frac{5}{x^2+1}$;

c) $\frac{7}{x^2-x+1}$

d) $\frac{x^2-59}{x+8}$

e) $\frac{x+2}{x^2+4}$

16. Tìm số hữu tỉ $x$ để phân thức $\frac{10}{x^2+1}$ có giá trị là số nguyên.

17*. Chứng minh rằng nếu các chữ số $\mathrm{a}, \mathrm{b}, \mathrm{c}$ khác 0 thoả mãn điều kiện $\overline{\mathrm{ab}}: \overline{\mathrm{bc}}=\mathrm{a}: \mathrm{c}$ thì $\overline{\mathrm{abbb}}: \overline{\mathrm{bbbc}}=\mathrm{a}: \mathrm{c} .$

 

 

 

 

 

 

 

 

 

 

 

 

 

 

 

 

 

 

 

 

 

 

 

 

 

 

 

 

 

 

 

 

 

 

 

 

 

 

 

 

 

 

 

 

 

 

 

 

 

 

 

 

 

 

 

PHÉP NHÂN VÀ PHÉP CHIA ĐA THỨC – P.3

PHÂN TÍCH ĐA THỨC THÀNH PHÂN TỬ

 

Để phân tích một đa thức thành nhân tử, ta thường dùng các phương pháp :

  • Đặt nhân tử chung.

  • Dùng các hằng đẳng thức đáng nhớ.

  • Nhóm các hạng tử một cách thích hợp nhằm làm xuất hiện dạng hằng đẳng thức hoặc xuất hiện nhân tử chung mới.

Để phân tích đa thức thành nhân tử, người ta còn dùng các phương pháp khác. Xem chuyên đề Một số phương pháp phân tích đa thức thành nhân tủ̉.

Ví dụ 1. Phân tích đa thức sau thành nhân tử :

$x^4+x^3+2 x^2+x+1$

Giải : $\quad \mathrm{x}^4+\mathrm{x}^3+2 \mathrm{x}^2+\mathrm{x}+1=\left(\mathrm{x}^4+2 \mathrm{x}^2+1\right)+\left(\mathrm{x}^3+\mathrm{x}\right)$

$=\left(x^2+1\right)^2+x\left(x^2+1\right)=\left(x^2+1\right)\left(x^2+x+1\right) .$

Ví dụ 2. Cho $a+b+c=0$. Rút gọn biểu thức

$M=a^3+b^3+c\left(a^2+b^2\right)-a b c .$

Giải :

$M =a^3+b^3+a^2 c+b^2 c-a b c=\left(a^3+a^2 c\right)+\left(b^3+b^2 c\right)-a b c $

$=a^2(a+c)+b^2(b+c)-a b c=a^2(-b)+b^2(-a)-a b c $

$=-a b(a+b+c)=0$

Ví dụ 3.

a) Phân tích đa thức sau thành nhân tử : $a^3+b^3+c^3-3 a b c$.

b) Phân tích đa thức sau thành nhân tử bằng cách áp dụng câu a) :

$(x-y)^3+(y-z)^3+(z-x)^3$.

Giải : $a^3+b^3+c^3-3 a b c=(a+b)^3-3 a^2 b-3 a b^2+c^3-3 a b c $

$= {\left[(a+b)^3+c^3\right]-3 a b(a+b+c) } $

$=(a+b+c)\left[(a+b)^2-c(a+b)+c^2\right]-3 a b(a+b+c) $

$=(a+b+c)\left(a^2+2 a b+b^2-a c-b c+c^2-3 a b\right) $

$=(a+b+c)\left(a^2+b^2+c^2-a b-b c-c a\right)$

b) Đặt $x-y=a, y-z=b, z-x=c$ thì $a+b+c=0$.

Do đó theo câu a) ta có $a^3+b^3+c^3-3 a b c=0 \Rightarrow a^3+b^3+c^3=3 a b c$

$\Rightarrow(x-y)^3+(y-z)^3+(z-x)^3=3(x-y)(y-z)(z-x) .$

Cần nhớ kết quả của câu a) để vận dụng vào giải toán.

Ví dụ 4. Phân tích các đa thức sau thành nhân tử :

a) $(a+b+c)^3-a^3-b^3-c^3$

b) $8(x+y+z)^3-(x+y)^3-(y+z)^3-(z+x)^3$

Giải : a) Áp dụng nhiều lần công thức $(\mathrm{x}+\mathrm{y})^3=\mathrm{x}^3+\mathrm{y}^3+3 \mathrm{xy}(\mathrm{x}+\mathrm{y})$, ta có :

$(a+b+c)^3-a^3-b^3-c^3=[(a+b)+c]^3-a^3-b^3-c^3 $

$=(a+b)^3+c^3+3 c(a+b)(a+b+c)-a^3-b^3-c^3 $

$=a^3+b^3+3 a b(a+b)+c^3+3 c(a+b)(a+b+c)-a^3-b^3-c^3 $

$=3(a+b)\left(a b+a c+b c+c^2\right) $

$=3(a+b)[a(b+c)+c(b+c)] $

$=3(a+b)(b+c)(c+a) .$

b) Đặt $\mathrm{x}+\mathrm{y}=\mathrm{a}, \mathrm{y}+\mathrm{z}=\mathrm{b}, \mathrm{z}+\mathrm{x}=\mathrm{c}$ thì $\mathrm{a}+\mathrm{b}+\mathrm{c}=2(\mathrm{x}+\mathrm{y}+\mathrm{z})$. Đa thức đã cho có dạng $(a+b+c)^3-a^3-b^3-c^3$.

Áp dụng kết quả của câu a), đa thức đã cho bằng :

$3(a+b)(b+c)(c+a)=3(x+2 y+z)(y+2 z+x)(z+2 x+y)$

Chú ý : Cần nhớ kết quả của câu a) để vận dụng vào giải toán.

Ví dụ 5. Phân tích đa thức sau thành nhân tử :

$P=x^2(y-z)+y^2(z-x)+z^2(x-y)$

Giải :

Cách 1 : Khai triển hai hạng tử cuối rồi nhóm các hạng tử làm xuất hiện ṇân tử chung $\mathrm{y}-\mathrm{z}$.

$P =x^2(y-z)+y^2 z-x y^2+x z^2-y z^2 $

$=x^2(y-z)+y z(y-z)-x\left(y^2-z^2\right) $

$=(y-z)\left(x^2+y z-x y-x z\right) $

$=(y-z)[x(x-y)-z(x-y)] $

$=(y-z)(x-y)(x-z)$

Cách 2. Tách $\mathrm{z}-\mathrm{x}$ thành $-[(\mathrm{y}-\mathrm{z})+(\mathrm{x}-\mathrm{y})]$, ta có

$P =x^2(y-z)-y^2[(y-z)+(x-y)]+z^2(x-y) $

$=(y-z)\left(x^2-y^2\right)-(x-y)\left(y^2-z^2\right) $

$=(y-z)(x+y)(x-y)-(x-y)(y+z)(y-z) $

$=(y-z)(x-y)(x+y-y-z) $

$=(y-z)(x-y)(x-z)$

Ví dụ 6. Xét hằng đẳng thức $(x+1)^3=x^3+3 x^2+3 x+1$.

Lần lượt cho $\mathrm{x}$ bằng $1,2,3, \ldots$, n rồi cộng từng vế $\mathrm{n}$ đẳng thức trên để tính giá trị của biểu thức :

$S=1^2+2^2+3^2+\ldots+n^2$

Giải : Từ hằng đẳng thức đã cho ta có :

$2^3=1^3+3.1^2+3.1+1 $

$3^3=2^3+3.2^2+3.2+1 $

$\cdots $

$(n+1)^3=n^3+3 n^2+3 n+1 $

Cộng từng vế $\mathrm{n}$ đẳng thức trên rồi rút gọn, ta được

$(n+1)^3=1^3+3\left(1^2+2^2+\ldots+n^2\right)+3(1+2+\ldots+n)+n$

Do đó

$ 3\left(1^2+2^2+\ldots+n^2\right)=(n+1)^3-\frac{3 n(n+1)}{2}-(n+1)=$

$=(n+1)\left[(n+1)^2-\frac{3 n}{2}-1\right]=(n+1)\left(n^2+\frac{n}{2}\right)=\frac{1}{2} n(n+1)(2 n+1) $

$\text { Vậy } S=\frac{1}{6} n(n+1)(2 n+1) $

BÀI TẬP

55. Phân tích thành nhân tử :

a) $(a b-1)^2+(a+b)^2$

b) $x^3+2 x^2+2 x+1$;

c) $x^3-4 x^2+12 x-27$

d) $x^4-2 x^3+2 x-1$;

e) $x^4+2 x^3+2 x^2+2 x+1$.

56. Phân tích thành nhân tử :

a) $x^2-2 x-4 y^2-4 y$

b) $x^4+2 x^3-4 x-4$;

c) $x^2\left(1-x^2\right)-4-4 x^2$

d) $(1+2 x)(1-2 x)-x(x+2)(x-2)$;

e) $x^2+y^2-x^2 y^2+x y-x-y$.

57. Chứng minh rằng $199^3-199$ chia hết cho 200 .

58. Tính giá trị của biểu thức sau, biết $x^3-x=6$ :

$A=x^6-2 x^4+x^3+x^2-x $

59. Phân tích thành nhân tử :

a) $a\left(b^2+c^2+b c\right)+b\left(c^2+a^2+a c\right)+c\left(a^2+b^2+a b\right)$

b) $(a+b+c)(a b+b c+c a)-a b c$;

$\left.c^*\right) a(a+2 b)^3-b(2 a+b)^3$

60. Phân tích thành nhân tử :

a) $a b(a+b)-b c(b+c)+a c(a-c)$;

b) $a\left(b^2+c^2\right)+b\left(c^2+a^2\right)+c\left(a^2+b^2\right)+2 a b c$;

c) $(a+b)\left(a^2-b^2\right)+(b+c)\left(b^2-c^2\right)+(c+a)\left(c^2-a^2\right)$

d) $a^3(b-c)+b^3(c-a)+c^3(a-b)$;

e) $a^3\left(c-b^2\right)+b^3\left(a-c^2\right)+c^3\left(b-a^2\right)+a b c(a b c-1)$.

61*. Phân tích thành nhân tử :

a) $a(b+c)^2(b-c)+b(c+a)^2(c-a)+c(a+b)^2(a-b)$;

b) $a(b-c)^3+b(c-a)^3+c(a-b)^3$;

c) $a^2 b^2(a-b)+b^2 c^2(b-c)+c^2 a^2(c-a)$

d) $a\left(b^2+c^2\right)+b\left(c^2+a^2\right)+c\left(a^2+b^2\right)-2 a b c-a^3-b^3-c^3$;

e) $a^4(b-c)+b^4(c-a)+c^4(a-b)$.

62. Phân tích thành nhân tử :

a) $(a+b+c)^3-(a+b-c)^3-(b+c-a)^3-(c+a-b)^3$.

b) $a b c-(a b+b c+c a)+(a+b+c)-1$.

63. Chứng minh rằng trong ba số $a, b, c$, tồn tại hąi số bằng nhau, nếu :

$a^2(b-c)+b^2(c-a)+c^2(a-b)=0 $

64. Chứng minh rằng nếu $\mathrm{a}^2+\mathrm{b}^2=2 \mathrm{ab}$ thì $\mathrm{a}=\mathrm{b}$.

65*. Chứng minh rằng nếu $\mathrm{a}^3+\mathrm{b}^3+\mathrm{c}^3=3 \mathrm{abc}$ và $\mathrm{a}, \mathrm{b}, \mathrm{c}$ là các số dương thì $\mathrm{a}=\mathrm{b}=\mathrm{c} .$

66*. Chứng minh rằng nếu $a^4+b^4+c^4+d^4=4 a b c d$ và $a, b, c, d$ là các số dương thì $a=b=c=d$

67. Chứng minh rằng nếu $\mathrm{m}=\mathrm{a}+\mathrm{b}+\mathrm{c}$ thì

$(\mathrm{am}+\mathrm{bc})(\mathrm{bm}+\mathrm{ac})(\mathrm{cm}+\mathrm{ab})=$

$(\mathrm{a}+\mathrm{b})^2(\mathrm{~b}+\mathrm{c})^2(\mathrm{c}+\mathrm{a})^2 $

68. Cho $a^2+b^2=1, c^2+d^2=1, a c+b d=0$. Chứng minh rằng $a b+c d=0$.

69. Xét hằng đẳng thức $(x+1)^2=x^2+2 x+1$.

Lần lượt cho $x$ bằng $1,2,3, \ldots$, n rồi cộng từng vế n đẳng thức trên để tính giá trị của biểu thức $\mathrm{S}_1=1+2+3+\ldots+\mathrm{n}$.

70*. Bằng phương pháp tương tự như ở ví dụ 14 và bài tập 74 , tính giá trị của biểu thức $\mathrm{S}_3=1^3+2^3+3^3+\ldots+\mathrm{n}^3$.

 

PHÉP NHÂN VÀ PHÉP CHIA ĐA THỨC – P.2

CÁC HẰNG ĐẲNG THỨC ĐÁNG NHỚ

 

Thực hiện phép nhân đa thức, ta được các hằng đẳng thức sau :

1. $(a+b)^2=a^2+2 a b+b^2$.

2. $(a-b)^2=a^2-2 a b+b^2$.

3. $(a+b)(a-b)=a^2-b^2$.

4. $(a+b)^3=a^3+3 a^2 b+3 a b^2+b^3$

$(a+b)^3=a^3+b^3+3 a b(a+b) \text {. }$

5. $(a-b)^3=a^3-3 a^2 b+3 a b^2-b^3$

$(a-b)^3=a^3-b^3-3 a b(a-b)$

6. $(a+b)\left(a^2-a b+b^2\right)=a^3+b^3$

7. $(a-b)\left(a^2+a b+b^2\right)=a^3-b^3$.

Ta cũng có :

$(a+b+c)^2=a^2+b^2+c^2+2 a b+2 a c+2 b c .$

Tổng quát của các hằng đẳng thức 3 và 7 , ta có hằng đẳng thức :

8. $a^n-b^n=(a-b)\left(a^{n-1}+a^{n-2} b+a^{n-3} b^2+\ldots+a b^{n-2}+b^{n-1}\right)$

với mọi số nguyên dương $\mathrm{n}$.

Tổng quát của hằng đẳng thức 6 , ta có hằng đẳng thức :

9. $a^n+b^n=(a+b)\left(a^{n-1}-a^{n-2} b+a^{n-3} b^2-\ldots-a b^{n-2}+b^{n-1}\right)$

với mọi số lẻ n.

Tổng quát của các hằng đẳng thức $1,2,4,5$, ta có công thức Niu-tơn (xem chuyên đề Tính chia hết đối với số nguyên).

Ví dụ 1. Chứng minh rằng số 3599 viết được dưới dạng tích của hai số tự nhiên khác 1 .

Giải : $\quad 3599=3600-1=60^2-1=(60+1)(60-1)=61.59$.

Ví dụ 2. Chứng minh rằng biểu thức sau viết được dưới dạng tổng các bình phương của hai biểu thức :

$x^2+2(x+1)^2+3(x+2)^2+4(x+3)^2$

Giải: $\mathrm{x}^2+2(\mathrm{x}+1)^2+3(\mathrm{x}+2)^2+4(\mathrm{x}+3)^2=$

$=x^2+2\left(x^2+2 x+1\right)+3\left(x^2+4 x+4\right)+4\left(x^2+6 x+9\right) $

$=x^2+2 x^2+4 x+2+3 x^2+12 x+12+4 x^2+24 x+36 $

$=10 x^2+40 x+50 $

$=\left(x^2+10 x+25\right)+\left(9 x^2+30 x+25\right) $

$=(x+5)^2+(3 x+5)^2$

Ví dụ 3. Cho

$x+y+z=0 $

$4x y+y z+z x=0$

Chứng minh rằng $\mathrm{x}=\mathrm{y}=\mathrm{z}$.

Giải : Ta có $(\mathrm{x}+\mathrm{y}+\mathrm{z})^2=\mathrm{x}^2+\mathrm{y}^2+\mathrm{z}^2+2(\mathrm{xy}+\mathrm{yz}+\mathrm{zx})$.

Suy ra

$0=x^2+y^2+z^2+2.0$

hay

$\text { Vậy } x=y=z(=0) \text {. }$

Ví dụ 4 :

a) Tính $A=-1^2+2^2-3^2+4^2-\ldots-99^2+100^2$.

b) Tính $\mathrm{A}=-1^2+2^2-3^2+4^2-\ldots+(-1)^{\mathrm{n}} \cdot \mathrm{n}^2$.

Giải: a) $\mathrm{A}=\left(2^2-1^2\right)+\left(4^2-3^2\right)+\ldots+\left(100^2-99^2\right)$

$=(2-1)(1+2)+(4-3)(3+4)+\ldots+(100-99)(99+100) $

$=1+2+3+4+\ldots+99+100 $

$=\frac{100.101}{2}=5050 .$

b) Xét hai trường hợp :

Nếu n chẵn thì $\mathrm{A}=\left(2^2-1^2\right)+\left(4^2-3^2\right)+\ldots+\left[\mathrm{n}^2-(\mathrm{n}-1)^2\right]$

$=1+2+3+4+\ldots+(n-1)+n$

$=\frac{\mathrm{n}(\mathrm{n}+1)}{2} \text {. }$

Nếu n lẻ thì $\mathrm{A}=\left(2^2-1^2\right)+\left(4^2-3^2\right)+\ldots+\left[(\mathrm{n}-1)^2-(\mathrm{n}-2)^2\right]-\mathrm{n}^2$

$=1+2+3+4+\ldots+(n-1)-n^2 $

$=\frac{n(n-1)}{2}-n^2=-\frac{n(n+1)}{2}$

Chú ý : Hai kết quả trên có thể viết chung trong một công thức

$(-1)^{\mathrm{n}} \cdot \frac{\mathrm{n}(\mathrm{n}+1)}{2}$

Ví dụ 5. Cho

$x+y=a+b\quad(1)$

$x^2+y^2=a^2+b^2\quad(2)$

Chứng minh rằng $x^3+y^3=a^3+b^3$.

Giải : Ta có : $\quad \mathrm{x}^3+\mathrm{y}^3=(\mathrm{x}+\mathrm{y})\left(\mathrm{x}^2-\mathrm{xy}+\mathrm{y}^2\right)\quad(3)$.

Từ (1) suy ra : $\quad(x+y)^2=(a+b)^2$,

tức là $\quad x^2+2 x y+y^2=a^2+2 a b+b^2$.

Do $x^2+y^2=a^2+b^2$ nên $2 x y=2 a b$, suy ra $x y=a b\quad(4)$

Thay các kết quả (1), (2), (4) vào (3), ta được

$x^3+y^3=(x+y)\left(x^2+y^2-x y\right)=(a+b)\left(a^2+b^2-a b\right)=a^3+b^3 .$

Ví dụ 6. Cho $a+b=m, a-b=n$. Tính $a b$ và $a^3-b^3$ theo $m$ và $n$.

Giải :

Cách 1. Từ $\mathrm{a}+\mathrm{b}=\mathrm{m}, \mathrm{a}-\mathrm{b}=\mathrm{n}$, ta tính được $\mathrm{b}=\frac{\mathrm{m}-\mathrm{n}}{2}, \mathrm{a}=\frac{\mathrm{m}+\mathrm{n}}{2}$.

Do đó $\quad \mathrm{ab}=\frac{\mathrm{m}+\mathrm{n}}{2} \cdot \frac{\mathrm{m}-\mathrm{n}}{2}=\frac{\mathrm{m}^2-\mathrm{n}^2}{4} ;$

$a^3-b^3=\left(\frac{m+n}{2}\right)^3-\left(\frac{m-n}{2}\right)^3=\frac{(m+n)^3-(m-n)^3}{8}$

Rút gọn biểu thức trên, ta được $\frac{3 \mathrm{~m}^2 \mathrm{n}+\mathrm{n}^3}{4}$.

Cách 2. Ta có

$4 a b =(a+b)^2-(a-b)^2=m^2-n^2 \text { nên } a b=\frac{m^2-n^2}{4} . $

$\text { Ta có } a^3-b^3 =(a-b)\left(a^2+a b+b^2\right)=(a-b)\left[(a+b)^2-a b\right] $

$=n\left(m^2-\frac{m^2-n^2}{4}\right)=\frac{n\left(3 m^2+n^2\right)}{4}=\frac{3 m^2 n+n^3}{4} .$

BÀI TẬP

16. Tính giá trị của các biểu thức :

a) $\frac{63^2-47^2}{215^2-105^2}$

b) $\frac{437^2-363^2}{537^2-463^2}$

17. So sánh $\mathrm{A}=26^2-24^2$ và $\mathrm{B}=27^2-25^2$.

18. Tìm $\mathrm{x}$, biết :

$4(x+1)^2+(2 x-1)^2-8(x-1)(x+1)=11$

19. Rút gọn các biểu thức :

a) $2 x(2 x-1)^2-3 x(x+3)(x-3)-4 x(x+1)^2$;

b) $(a-b+c)^2-(b-c)^2+2 a b-2 a c$;

c) $(3 x+1)^2-2(3 x+1)(3 x+5)+(3 x+5)^2$;

d) $(3+1)\left(3^2+1\right)\left(3^4+1\right)\left(3^8+1\right)\left(3^{16}+1\right)\left(3^{32}+1\right)$;

e) $(a+b-c)^2+(a-b+c)^2-2(b-c)^2$

g) $(a+b+c)^2+(a-b-c)^2+(b-c-a)^2+(c-a-b)^2$;

h) $(a+b+c+d)^2+(a+b-c-d)^2+(a+c-b-d)^2+(a+d-b-c)^2$.

20. Cho $x+y=3$. Tính giá trị của biểu thức

$A=x^2+2 x y+y^2-4 x-4 y+1 $

21. Cho $\mathrm{a}^2+\mathrm{b}^2+\mathrm{c}^2=\mathrm{m}$. Tính giá trị của biểu thức sau theo $\mathrm{m}$ :

$A=(2 a+2 b-c)^2+(2 b+2 c-a)^2+(2 c+2 a-b)^2 .$

22. Hãy viết các số sau đây dưới dạng tích của hai số tự nhiên khác 1 :

a) $899$

b) $9991$

23. Chứng minh rằng hiệu sau đây là một số gồm toàn các chữ số như nhau :

$7778^2-2223^2$

24. Chứng minh các hằng đẳng thức :

a) $(a+b+c)^2+a^2+b^2+c^2=(a+b)^2+(b+c)^2+(c+a)^2$

b) $x^4+y^4+(x+y)^4=2\left(x^2+x y+y^2\right)^2$

25. Cho $\mathrm{a}^2-\mathrm{b}^2=4 \mathrm{c}^2$. Chứng minh hằng đẳng thức

$(5 a-3 b+8 c)(5 a-3 b-8 c)=(3 a-5 b)^2$

26. Chứng minh rằng nếu $\left(a^2+b^2\right)\left(x^2+y^2\right)=(a x+b y)^2$ với $x, y$ khác 0 thì $\frac{\mathrm{a}}{\mathrm{x}}=\frac{\mathrm{b}}{\mathrm{y}}$

27. Chứng minh rằng nếu $\left(\mathrm{a}^2+\mathrm{b}^2+\mathrm{c}^2\right)\left(\mathrm{x}^2+\mathrm{y}^2+\mathrm{z}^2\right)=(\mathrm{ax}+\mathrm{by}+\mathrm{cz})^2$ với $x, y, z$ khác 0 thì $\frac{a}{x}=\frac{b}{y}=\frac{c}{z}$.

28. Cho $(a+b)^2=2\left(a^2+b^2\right)$. Chứng minh rằng $a=b$.

29. Chứng minh rằng $\mathrm{a}=\mathrm{b}=\mathrm{c}$ nếu có một trong các điều kiện sau :

a) $a^2+b^2+c^2=a b+b c+c a$

b) $(a+b+c)^2=3\left(a^2+b^2+c^2\right)$

c) $(a+b+c)^2=3(a b+b c+c a)$.

  1. Hãy viết các biểu thức sau dưới dạng tổng của ba bình phương :

a) $(a+b+c)^2+a^2+b^2+c^2$

b) $2(a-b)(c-b)+2(b-a)(c-a)+2(b-c)(a-c)$

31. Tính giá trị của biểu thức $\mathrm{a}^4+\mathrm{b}^4+\mathrm{c}^4$, biết rằng $\mathrm{a}+\mathrm{b}+\mathrm{c}=0$ và :

a) $a^2+b^2+c^2=2$;

b) $a^2+b^2+c^2=1$.

32. Cho $\mathrm{a}+\mathrm{b}+\mathrm{c}=0$. Chứng minh $\mathrm{a}^4+\mathrm{b}^4+\mathrm{c}^4$ bằng mỗi biểu thức :

a) $2\left(a^2 b^2+b^2 c^2+c^2 a^2\right)$;

b) $2(a b+b c+c a)^2$

c) $\frac{\left(a^2+b^2+c^2\right)^2}{2}$

33. Chứng minh rằng các biểu thức sau luôn luôn có giá trị dương với mọi giá trị của biến :

a) $9 x^2-6 x+2$

b) $\mathrm{x}^2+\mathrm{x}+1$

c) $2 x^2+2 x+1$.

34. Tìm giá trị nhỏ nhất của các biểu thức :

a) $A=x^2-3 x+5 ;$

b) $B=(2 x-1)^2+(x+2)^2$

35. Tìm giá trị lớn nhất của các biểu thức :

a) $A=4-x^2+2 x$

b) $B=4 x-x^2$

36. Chứng minh rằng :

a) Nếu $\mathrm{p}$ và $\mathrm{p}^2+8$ là các số nguyên tố thì $\mathrm{p}^2+2$ cũng là số nguyên tố.

b) Nếu $\mathrm{p}$ và $8 \mathrm{p}^2+1$ là các số nguyên tố thì $2 \mathrm{p}+1$ cũng là số nguyên tố.

37. Chứng minh rằng các số sau là hợp số :

a) 999991 ;

b) 1000027 .

38. Thực hiện phép tính :

a) $(x-2)^3-x(x+1)(x-1)+6 x(x-3)$

b) $(x-2)\left(x^2-2 x+4\right)(x+2)\left(x^2+2 x+4\right)$.

39. Tìm $x$, biết :

a) $(x-3)\left(x^2+3 x+9\right)+x(x+2)(2-x)=1$

b) $(x+1)^3-(x-1)^3-6(x-1)^2=-10$

40. Rút gọn các biểu thức :

a) $(a+b+c)^3-(b+c-a)^3-(a+c-b)^3-(a+b-c)^3$

b) $(a+b)^3+(b+c)^3+(c+a)^3-3(a+b)(b+c)(c+a)$

41. Chứng minh các hằng đẳng thức :

a) $(a+b+c)^3-a^3-b^3-c^3=3(a+b)(b+c)(c+a)$.

b) $a^3+b^3+c^3-3 a b c=(a+b+c)\left(a^2+b^2+c^2-a b-b c-c a\right)$.

42. Cho $a+b+c=0$. Chứng minh rằng $a^3+b^3+c^3=3 a b c$.

43. Cho $\mathrm{x}+\mathrm{y}=\mathrm{a}$ và $\mathrm{xy}=\mathrm{b}$. Tính giá trị của các biểu thức sau theo $\mathrm{a}$ và $\mathrm{b}$ :

a) $x^2+y^2$

b) $x^3+y^3$

c) $x^4+y^4$;

d) $x^5+y^5$.

44. a) Cho $x+y=1$. Tính giá trị của biểu thức $x^3+y^3+3 x y$.

b) Cho $\mathrm{x}-\mathrm{y}=1$. Tính giá trị của biểu thức $\mathrm{x}^3-\mathrm{y}^3-3 \mathrm{xy}$.

45. Cho $\mathrm{a}+\mathrm{b}=1$. Tính giá trị của biểu thức

$M=a^3+b^3+3 a b\left(a^2+b^2\right)+6 a^2 b^2(a+b)$

46. a) Cho $x+y=2$ và $x^2+y^2=10$. Tính giá trị của biểu thức $x^3+y^3$.

b) Cho $x+y=a$ và $x^2+y^2=b$. Tính $x^3+y^3$ theo a và $b$.

47. Chứng minh rằng :

a) Nếu số n’ là tổng của hai số chính phương thì 2 n cũng là tổng của hai số chính phương.

b) Nếu số $2 \mathrm{n}$ là tổng của hai số chính phương thì n cũng là tổng của hai số chính phương.

c) Nếu số $\mathrm{n}$ là tổng của hai số chính phương thì $\mathrm{n}^2$ cũng là tổng của hai số chính phương.

d) Nếu mỗi số m và $\mathrm{n}$ đều là tổng của hai số chính phương thì tích mn cũng là tổng của hai số chính phương.

48. Chứng minh rằng với mọi số tự nhiên $\mathrm{a}$, tồn tại số tự nhiên $\mathrm{b}$ sao cho $\mathrm{ab}+4$ là số chính phương.

49. Cho a là số gồm $2 \mathrm{n}$ chữ số $1, \mathrm{~b}$ là số gồm $\mathrm{n}+1$ chữ số $1, \mathrm{c}$ là số gồm $\mathrm{n}$ chữ số 6. Chứng minh rằng $\mathrm{a}+\mathrm{b}+\mathrm{c}+8$ là số chính phương.

50. Chứng minh rằng biểu thức sau không là lập phương của một số tự nhiên :

$10^{150}+5.10^{50}+1 .$

51. Chứng minh rằng tích ba số nguyền dương liên tiếp không là lập phương của một số tự nhiên.

52. Chia 27 quả cân có khối lượng $10,20,30, \ldots, 270$ gam thành ba nhóm có khối lượng bằng nhau.

53*. Chia 18 quả cân có khối lượng $1^2, 2^2, 3^2, \ldots, 18^2$ gam thành ba nhóm có khối lượng bằng nhau.

54*. Chia 27 quả cân có khối lượng $1^2, 2^2, 3^2, \ldots, 27^2$ gam thành ba nhóm có khối lượng bằng nhau.